Download as pdf or txt
Download as pdf or txt
You are on page 1of 124

SOLUTIONS

INDEX
OF SELF-PRACTICE Q’S

If you want to go to the answer of any Self-practice Q’s and you


know it comes under Objective Q's, then just click on Objective Q's link
FOR EXAMPLE
within the chapter to navigate quickly.

1 : Real Numbers Pg. 4 : Quadratic Equations Pg.


Objective Q's 01 Objective Q's 21

CBQ 01 CBQ 23

VSA 02 VSA 24

SA-I 02 SA-I 25

SA-II 03 SA-II 25

LA 04 LA 28

2 : Polynomials 5 : Arithmetic Progressions


Objective Q's 05 Objective Q's 31

CBQ 07 CBQ 32

VSA 08 VSA 33

SA-I 08 SA-I 34

SA-II 08 SA-II 35

LA 09 LA 38

3 : Pair of Linear Equations


in Two Variables 6 : Triangles
Objective Q's 10 Objective Q's 43

CBQ 12 CBQ 44

VSA 14 VSA 46

SA-I 14 SA-I 46

SA-II 15 SA-II 48

LA 18 LA 48
7 : Coordinate Geometry Pg. 11 : Areas Related to Circles Pg.
Objective Q's 51 Objective Q's 89

CBQ 54 CBQ 90

VSA 56 VSA 91

SA-I 56 SA-I 92

SA-II 57 SA-II 92

LA 59 LA 94

12 : Surface Areas and Volumes


8 : Introduction to Objective Q's 97
Trigonometry
CBQ 98
Objective Q's 62

CBQ 64 VSA 99

SA-I 100
VSA 65
SA-II 100
SA-I 65
LA 102
SA-II 66

9 : Some Applications of 13 : Statistics


Trigonometry Objective Q's 105
Objective Q's 67
VSA 109
CBQ 69
SA-I 109
VSA 70
SA-II 109
SA-I 71
LA 112
SA-II 71

LA 72 14 : Probability
Objective Q's 115
10 : Circles
CBQ 116
Objective Q's 77

CBQ 79 VSA 116

VSA 80 SA-II 118

SA-I 81 LA 121

SA-II 84

LA 86
 ANSWER SHEET

SELF PRACTICE
3. (c) 20 [CBSE Marking Scheme SQP 2019] 25. (b) 2
Explanation: Explanation: Finding the HCF of 65, 117 by
Smallest two-digit composite number = 10 factorisation method:
= 2 × 5
Smallest composite number = 4 = 22 65 = 13 × 5
\ LCM (10, 4) = 22 × 5 117 = 13 × 3 × 3
= 20 So, HCF of 65 and 117 = 13  ...(i)
6. (c) 500 Also, given that,
Explanation: HCF of two numbers is always a HCF (65, 117) = 65m – 117 ...(ii)
factor of their LCM. From equations (i) and (ii), we get
Here, 300, 400 and 600 are the factors of 2400,
65m − 117 = 13
but 500 is not a factor of 2400.
Þ 65m = 130
10. (c) 3 Þ m=2
Explanation:
27. 81
144 = 2 × 2 × 2 × 2 × 3 × 3
Explanation:
= 24 × 32
Let the other number be x.
180 = 2 × 2 × 3 × 3 × 5
We know,
= 22 × 32 × 5
Product of two numbers
HCF (144, 180) = 22 × 32
= HCF × LCM = 27 × 162
= 36
⇒ 13 m – 3 = 36 Þ 54x = 27 × 162
13 m = 39 Þ x = 81
39 31. 1
m= =3
13
Explanation: We know,
12. (d) 675 HCF (a, b) × LCM (a, b) = ab
Explanation: We know that, HCF ( a, b ) × LCM ( a, b )
Þ =1
a × b ab
LCM (a, b) =
HCF( a, b )
32. 2
30375
So, LCM (a, b) = = 675 Explanation: All even numbers are divisible by
45 2. Therefore, HCF of any two consecutive even
even numbers is 2.
15. (a) prime, co-prime
Explanation: Prime numbers are those 36. (c) Assertion (A) is true but reason (R) is false.
numbers which have only two factors i.e., 1 Explanation: Product of (3 + 3 ) and (2 – 3 ) is
and itself. E.g., 3, 5, 11 etc.
Co-prime numbers are the set of those numbers (3 – 3 ), which is irrational.
that have only 1 as a common factor.
But product of two irrational numbers
E.g. 35 and 39 also gives a rational number, for example,
35 = 1 × 5 × 7, 39 = 1 × 3 × 13
3 × 3 = 3.
Here, common factor is 1.
18. (b) x = 21; y = 84 Hence, assertion is true but the reason is false.
Explanation:
39. (C) (a) 3

Here, x = 7 × 3 = 21
Explanation: To calculate the number of
and y = 4 × x = 4 × 21 = 84 times the buses on lines A and B leave

Real Numbers 1

Click here to buy other Educart books on Amazon - https://amzn.to/40U8Txx


together between 8 a.m. and 11 a.m., if 45. Given, product of numbers,
they first leave at 8 a.m., we first need to xy = 180
calculate the LCM of 15 and 20. HCF (x, y) = 3
15 = 3 × 5
Also, we know,,
20 = 22 × 5
LCM(x, y) × HCF(x, y) = Product of x and y
LCM = 22 × 3 × 5 = 60
So, the buses will leave together after 180
\ LCM(x, y) = = 60
every 60 minutes. They will leave together 3
at 9 a.m., 10 a.m. and 11 a.m.
Hence, LCM of x and y is 60.

(E) (b) 156
49. Given, HCF (a, b) = 5 and LCM (a, b) = 200.
Explanation: The relation between the
numbers a and b and their HCF (a, b), LCM We know,
(a, b) is given by: HCF (a, b) × LCM (a, b)
HCF (a, b) × LCM (a, b) = a × b = a × b
So, 13 × LCM = 2028 Þ a × b = 5 × 200
2028 Þ ab = 1000
Þ LCM = = 156
13 Hence, the product ab is 1000.
40. (C) Since, there should be no wastage, the
edge of the towel should exactly divide
50. No.
the length & breadth of the piece of cloth.
The dimension of the towel is that highest We know that:
number which completely divides 16 and “The HCF of any two numbers must be a factor
20. So, here we have to find the HCF of the LCM of those numbers.”
(16, 20).
 16 = 24 So, two numbers cannot have their HCF 18 and
and, 20 = 22 × 5 LCM 380, as 18 does not divide 380 completely.
\ HCF (16, 20) = 22 = 4
So, the dimensions of one small towel 54. Rational number between 2 (1.41 approx)
that can be cut are 4 m × 4 m.
and 3 (1.73 approx) can be 1.5, 1.6, 1.63 etc.
The minimum number of towels possible
16  20
So, a required rational number may be 1.5.
is = 20.
44
57. Since, HCF (a, b) × LCM (a, b) = a × b
41. (B) Here, 2783 = 253 × y
45 × 105
2783 Then, LCM (45, 105) = = 315
=y 15
253
Hence, the LCM (45, 105) is 315.
2783
y =
253 59. Let the two natural numbers be p and q.
y = 11 According to question,
Similarly, p × q = 24 ...(i)
253 = 11 × z
and p–q=2
253
=z Þ p=2+q ...(ii)
11
Substituting the value of p from equation (ii) to
253
z = equation (i), we get
11
(q + 2) × q = 24
z = 23
q2 + 2q – 24 = 0
44. HCF × LCM = product of two numbers q2 + 6q – 4q – 24 = 0
96 × 404 (q + 6) (q – 4) = 0
\ LCM (96,404) =
HCF (96,404) q = – 6, 4
96 × 404 q=4
=  [Since –6 is not a natural number]
4
p=2+4=6
Þ LCM = 9696.
[CBSE Marking Scheme SQP 2020] So, the numbers are 4, 6.

2 Mathematics Class X

Click here to buy other Educart books on Amazon - https://amzn.to/40U8Txx


60. Here, we need to find the LCM of 72 and 50. 74. Let 5 + 2 7 is a rational number.
 72 = 2 × 2 × 2 × 3 × 3
a
50 = 2 × 5 × 5 i.e., 5+2 7 = ,
\ LCM of 72 and 50 = 23 × 32 × 52 = 1800
b
 1800 sec = 30 min where a and b are co-prime numbers and
Hence, alarm clocks will beep again for the b ¹ 0.
second time at 12:30 pm. a
Þ 2 7 = −5
b
63. We have,
612 = 2 × 2 × 3 × 3 × 17 a − 5b
Þ 7 =
1314 = 2 × 3 × 3 × 73 2b
\ HCF (612, 1314) = 2 × 3 × 3 a − 5b
Since, is a rational number, so 7
= 18 2b be a rational number which is
is must also
Hence, the HCF of 612 and 1314 is 18. contradiction to the given condition.
66. Consider two irrationals as, 5 − 2 2 and It means our assumption is wrong.
5 + 2 2. Hence, 5 + 2 7 is irrational.
Here,
76. We know that the LCM is the product of the
(5 − 2 2 ) (5 + 2 2 ) = 52 (
− 2 2 )2 greatest power of each prime factor of the
given numbers.
= 25 – 8 = 17
which is a rational number. We have to find the LCM of 40, 42 and 45 to
get the required minimum distance.
70. To make all the plates identical and to find the
greatest number of plates, we have to find the For this, we find prime factorisation of given
greatest number which can divide 12 and 18 numbers.
exactly. i.e., H.C.F. of 12 and 16. 40 = 2 × 2 × 2 × 5
H.C.F. of (12, 16) = 4 42 = 2 × 3 × 7
That is, 12 pieces of chicken would be served in 45 = 3 × 3 × 5
4 plates at the rate of 3 pieces per plate. LCM (40, 42, 45) = 2 × 2 × 2 × 3 × 3 × 5 × 7
And 16 rotis would be served in 4 plates at the = 2520
rate of 4 rotis per plate.
Hence, each person should walk a minimum
72. Let 2 – 3 be a rational number. distance of 2520 cm, so that each of them can
Then, we can find co-primes a and b (b ≠ 0) cover the same distance in complete steps.
such that,
a 79. Let us suppose that p + q is rational.
2– 3=
b Let p + q = a, where a is a rational number.
a

Þ 2− = 3 Þ p =a– q
b
On squaring both sides, we get
(2b − a )
So we get, = 3 Þ p = a2 + q − 2a q
b
(2b − a ) [Using (a − b)2 = a2 + b2 − 2ab]
Since a and b are integers, we get
b a2 + q – p
Þ q =
is rational and so 3 is rational. But it 2a
The above statement is a contradiction as the
is already given that 3 is an irrational
right hand side is a rational number, while the
number.
left hand side i.e., q is irrational, since p and q
This contradicts our assumption.
are prime numbers.
Hence, our assumption is wrong. So, our assumption is wrong.
Therefore, 2 − 3 is an irrational number. Hence, p + q is irrational.

Real Numbers 3

Click here to buy other Educart books on Amazon - https://amzn.to/40U8Txx


2
5  5  5 2
81. Let is a rational number. Therefore, we can
7  49 k  = b
49

find two integers a, b (b ¹ 0)
5 2
5 a b2 = k .
such that, = 49
7 b
5
Let, a and b have a common factor other than This means that b2 is divisible by and
49
1. Then we can divide them by the common
factor, and assume that a and b are co-prime. 5
hence, b is divisible by .
49
5
a= b 5
7 This implies that a and b have as a
49
5 2
a2 = b common factor. And this is a contradiction to
49 the fact that a and b are co-prime.
5 p
Therefore, a2 is divisible by and it can be 5
49 Thus, cannot be expressed as . It means
7 q
5 5 our assumption is wrong.
said that a is divisible by . Let a = k,
49 49 5

Hence, is an irrational number.
where k is an integer 7

4 Mathematics Class X

Click here to buy other Educart books on Amazon - https://amzn.to/40U8Txx


 ANSWER SHEET

SELF PRACTICE
1. (b) 3 If the product of the two numbers is positive,
Explanation: Since the curve y = p(x) cuts or then either both are negative or both are
meets the x-axis at three points, the function positive. But the sum of these numbers is
p(x) has three zeroes. negative, so the numbers must be negative.
Þ Both the zeroes are negative.
5. (a) k[x2 – 8x + 5] Hence, in both the cases, both the zeroes
Explanation:
cannot be positive.
We know that, a quadratic polynomial with sum
(S) and product (P) of zeroes is given as, 10. (a) has no linear term and the constant term is
k[x2 – Sx + P] negative.
i.e., k[x2 – 8x + 5] Explanation: Let, p(x) = x2 + ax + b
6. (a) x2 + 5x + 6 And let a be one of the zeroes.
Explanation: Since, the quadratic polynomial Then, –a is the other zero of the polynomial p(x).
is:  [Given]
x – (sum of roots) x + product of roots = 0
2
Now,
Thus a polynomial, in which sum of zeroes is Constant term
Product of the zeroes =
– 5 and product of zeroes is 6, is : Coefficient of x 2
x2 + 5x + 6 b
= 1
8. (a) both cannot be positive
Þ a(–a) = b
Explanation: Let p(x) = x2 + kx + k, where k ≠ 0.
Þ –a2 = b i.e., b<0
On comparing p(x) with ax2 + bx + c,
i.e., the constant term is negative.
we get, a = 1, b = k and c = k.
Also,
Let a and b be the zeroes of the polynomial p(x).
Coefficient of x
We know that, Sum of the zeroes = −
Coefficient of x 2
Sum of the zeroes
a
= – 1
b
a+b = − a
a Þ a–a = – 1
k 0 =–a
Þ a+b = − = −k ...(i)
1 Þ a =0
And product of the zeroes
Hence, it has no linear term and the constant
c term is negative.
ab =
a 11. (d)
y

k
Þ ab =− = k ...(ii)
1
x!
Case 1: k is negative
x

If k is negative,
Then, ab [from equation (ii)] is negative. y!
It means a and b are of the opposite sign.
Explanation: From the given options, only
Þ Both the zeroes are of the opposite signs. option (d) has more than two roots, so it cannot
Case 2: k is positive be the graph of a quadratic polynomial.
If k is positive, For any quadratic polynomial ax2 + bx + c,
Then, ab [(from equation (ii)] is positive. But a ≠ 0, the graph of the corresponding
a + b is negative. polynomial ax2 + bx + c, has one of the two

Polynomials 5

Click here to buy other Educart books on Amazon - https://amzn.to/40U8Txx


shapes: either open upwards like È (parabolic Therefore, p(2) = 0 and p(–3) = 0
shape) or open downwards like Ç (parabolic p(2) = (2)2 + (a + 1)(2) + b = 0
shape), depending on whether a > 0 or a < 0 Þ 4 + 2a + 2 + b = 0
respectively. These curves are called parabolas.
Þ 2a + b + 6 = 0 ...(i)
So, option (d) cannot be possible.
Also, p(–3) = (–3)2 + (a + 1)(–3) + b = 0
Alternate Method:
Þ 9 – 3a – 3 + b = 0
Also, the curve of a quadratic polynomial
crosses the x-axis at atmost two points but in Þ –3a + b + 6 = 0 ...(ii)
option (d), the curve crosses the x–axis at three From (i) and (ii), we get
points, so it does not represent a quadratic 2a + b + 6 = –3a + b + 6
polynomial.
Þ 5a = 0
Hence, (d) is not the graph of a quadratic
Þ a =0
polynomial.
Putting the value of ‘a’ in (i), we get
16. (b) p = r = –2
2(0) + b + 6 = 0
Sum of zeroes = 2 + 1 = 5 Þ b = –6
2 2
Hence, if the zeroes of the quadratic polynomial
5 5 x2 + (a + 1)x + b are 2 and –3, then the required
i.e., = − . So, p = –2
2 p values of a and b are 0 and – 6 respectively.
Alternate Method:
Product of zeroes = 2 × 1 = 1
2 It is given that 2 and –3 are the zeroes of the
given quadratic polynomial.
r
i.e., = 1 or r = p = –2 \ Sum of the zeroes = 2 + (–3) = –1 ...(i)
p
Product of the zeroes = 2(–3) = –6 ...(ii)
[CBSE Marking Scheme Term-1 SQP 2021]
The equation of a quadratic polynomial is
given by
1 p(x) = k{x2 – (sum of the zeroes)x
Explanation: Since 2 and are zeroes of
2 + (product of the zeroes)},
polynomial px2 + 5x + r where, k is a constant.
Coefficient of x Here, p(x) = x2 + (a + 1)x + b.

\ Sum of zeroes = –
2 Comparing the two equations we get:
Coefficient of x
− Coefficient of x
1 5 Sum of the zeroes =

Þ 2+ = − Coefficient of x 2
2 p
Þ Sum of the zeroes = – (a + 1)
5 5 Þ –1 = –a – 1 [Using (i)]
Þ = −
2 p
Þ –1 + 1 = –a
Þ p = –2 ...(i) Þ –a = 0
Also,
Þ a=0
Constant term
Product of zeroes = Constant term
Coefficient of x 2 Product of the zeroes =
Coefficient of x 2
1 r ÞProduct of the zeroes = b

Þ 2× =
2 p –6 = b [Using (ii)]
r Þ b = –6

Þ =1 Hence, if the zeroes of the quadratic polynomial
p
x2 + (a + 1)x + b are 2 and –3, then the required

Þ r = p = –2 [From (i)] values of a and b are a = 0 and b = –6.
17. (d) a = 0, b = –6
24. 3 and – 1
Explanation: Let p(x) = x2 + (a + 1)x + b.
Explanation: We have,
We know that, if a is a zero of a polynomial p(x) x2 – 2x – 3 = x2 – 3x + x – 3
then, p(a) = 0. = x (x – 3) + 1 (x – 3)
It is given that 2 and –3 are the zeroes of the = (x – 3) (x + 1)
given quadratic polynomial. \ Zeroes are 3 and –1.

6 Mathematics Class X

Click here to buy other Educart books on Amazon - https://amzn.to/40U8Txx


−2 36. (D) (c) neither touches nor intersects x-axis.
26.
3 Explanation: We have,
Explanation: Let x2 + 1 = 0
p(x) = kx2 + 2x + 3k ⇒ x2 = –1

−2 ⇒ x = ± −1
\ Sum of zeroes =
k Since, ± −1 are imaginary values, so no
And, real value of x exists.
3k Therefore, graph of x2 + 1 = 0 neither
Product of zeroes = =3
k touches nor intersects x-axis.
According to question,  1
Sum of zeroes = Product of zeroes (E) (c) k  x 2 + px − 
 p
−2
Þ =3 Explanation: We know, a quadratic
k
polynomial with S and P as sum and
−2 product of zeroes respectively, is given as
Þ k=
3 k(x2 – Sx + P), where k is any constant.
29. (a) Both assertion (A) and reason (R) are true 1
and reason (R) is the correct explanation of Here, S = –p and P = −
p
assertion (A). \The required quadratic polynomial is:
Explanation: The given quadratic equation is   1 
x2 + 7x + 12 k  x2 − ( − p)x +  −  
  p 
x2+ 4x + 3x +12 = 0

Þ x(x + 4) + 3(x + 4) = 0  2 1
⇒ k  x + px − 
Þ (x + 4)(x + 3) = 0  p
Þ (x + 4) = 0 or (x + 3) = 0 5
Þ x = –4 or x = –3 37. (B) (c) −
2
Therefore, x2 + 7x + 12 has two real zeroes.
Explanation: Given a and b are the roots of
Hence, both assertion and reason are true and f(x) = 2x2 + 4x + k
the reason is the correct explanation of the
21
assertion. Anda2 + b2 + ab =
4
33. (C) To find the zeroes of the given polynomial,
let us factorise the polynomial by splitting b 4
We know,a + b = − =−
the middle term. a 2
p(t) = 24t2 – 41t + 12 c k
= 24t2 – 32t – 9t + 12 ab = =
a 2
= 8t (3t – 4) – 3(3t – 4)
= (8t – 3)(3t – 4) (a + b)2 = a2 + b2 + 2ab
2
3 4  4
Therefore, zeroes are t = ,
8 3  − 2  = a2 + b2 + ab + ab

34. (A) The graph of a quadratic polynomial is 2
a parabola. As the shape of the bridge  4 21 k
 − 2  = +
resembles a parabola, it can be said that 4 2
it represents the graph of a quadratic
polynomial. 16 21 k
= +
35. (A) (d) Parabola 4 4 2
(E) (d) 0
k 16 21
= −
Explanation: We have, 2 4 4
p(x) = x2 – 2x – 3 k −5
=
So, for x = – 1, 2 4

p(–1) = (– 1)2 – 2(–1) – 3
−5
= 1 + 2 – 3 k=
2
= 0

Polynomials 7

Click here to buy other Educart books on Amazon - https://amzn.to/40U8Txx


1 −5
(D) (b) a, Then, a = and b = –1
a 2
−5  −5 
Explanation: The quadratic polynomial a + b + ab =

2
+ (–1) +   −1
 2
( )
a(x2 + 1) – x(a2 + 1) can be written as
ax2 – (a2 + 1) x + a by rearranging the terms.
−5 5
\ ax2 – (a2 + 1) x + a = −1 + = –1
= ax2 – a2x – x + a 2 2
= ax (x – a) – 1(x – a) Hence, the value of a + b + ab is –1.
= (ax – 1)(x – a)
1 51. Here, zeroes of polynomial are 15 and − 15 .
So, the zeroes are a ,
a Let, a = 15 , b = − 15

(E) (b) –10
Sum of roots, a + b = 15 − 15 = 0
Explanation: Let, p(x) = x2 + 3x + a
Since, 2 is one of the zeroes of p(x).
\ p(2) = 0
Product of roots, ab = ( 15 )(− 15 ) = –15
22 + 3(2) + a = 0 Then, quadratic polynomial is :
4+6+a=0 x2 – (sum of roots) x + product of roots
a = –10 ⇒ x2 – (0) x + (–15)
⇒ x2 – 15
40. The general form of a quadratic polynomial is Hence, the required quadratic polynomial is
ax2 + bx + c. x2 – 15.
b c
Here, a + b = − = – 3 and αβ = = 2 52. f(x) = 4x2 – 8kx – 9
a a
where, a and b are the zeroes of the polynomial. Let one of the zeroes of the polynomial be a
Now, a quadratic polynomial with zeroes a and and the other zero be – a.
b is given as,  b  8k
x2 – (a + b) x + ab \ Sum of zeroes =  −  =
 a 4
So, the required polynomial is x2 + 3x + 2.
8k
⇒ a + (– a) =
42. Given, polynomial is 3x2 – 10x + k, whose 4
zeroes are reciprocals of each other.
1 8k
Let the zeroes be a and . So, 0= Þ k=0
α 4
c
\ Product of zeroes =
a 58. Let a and b are be the zeroes of 2x2 – 5x – 3.
1 k b 5
Þ a. = \ a+b= − = ...(i)
α 3 a 2
k c 3
Þ 1= and, ab = = − ...(ii)
3 a 2
Þ k= 3 According to the question,
Hence, the value of k is 3. 2a and 2b are zeroes of x2 + px + q.
45. Since, (–3) is a zero of p(x), we have, \ 2a + 2b = –p
p(–3) = 0 Þ  2(a + b) = –p
⇒ (k – 1)(–3)2 – k(–3) + 1 = 0 5
Þ 2 ×   = –p [from eqn. (i)]
⇒ 9k – 9 + 3k + 1 = 0 2
⇒ 12k = 8 Þ p = –5
2 Also, 2a × 2b = q
⇒ k=
3 Þ  4ab = q
48. Let,
p(y) = 2y2 + 7y + 5  3
= 2y2 + 2y + 5y + 5 Þ 4× −  = q [from eqn. (ii)]
 2
= 2y (y + 1) + 5 (y + 1)
Þ q = –6
= (2y + 5) (y + 1)
Hence, p = –5 and q = –6.
−5
⇒ y= , −1 [CBSE Marking Scheme SQP Std. 2022]
2

8 Mathematics Class X

Click here to buy other Educart books on Amazon - https://amzn.to/40U8Txx


59. Given polynomial is: =
1
[(7y + 1) (3y – 2)]
p(x) = x2 – (k + 6) x + 2(2k – 1) 3

Here, −1 2
y= ,
a = 1, b = – (k + 6), c = 2(2k – 1) 7 3
b −1 2
\ Sum of zeroes = − = k + 6 ...(i) \ Zeroes of polyomial p(y) are ,
a 7 3
c Verification:

Product of zeroes = = 2(2k – 1) ...(ii)
a −1 2 −3 + 14
Sum of zeroes= + =
According to the given condition: 7 3 21
1 11
Sum of the zeroes = × Product of zeroes =
2 21

1  −11 
Þ k+6 = × 2 (2k – 1)
2 −b  3  11
Þ k + 6 = 2k – 1 and =– =
a 7 21
Þ 2k – k = 6 + 1 = Sum of zeroes
Þ k =7 −1 2
Product of zeroes = ×
Hence, the value of k is 7. 7 3
63. Let a and 7a be the two zeroes of the polynomial −2
=
3x – 8x + (2k + 1).
2 21
8  −2 
Then, a + 7a = 8a = .....(i)  3 
3 c −2
and = =
2k + 1 a 7 21
and a.(7a) = 7a2 = .....(ii)
3 = Product of zeroes
Hence, verified.
1
From (i), a= . 68. Since, a and b are the zeroes of the given
3 polynomial
1 7
So, the zeroes are and .
3 3 −Coefficient of x
1
\ Sum of zeroes, a + b =
Using a = in (ii), we get Coefficient of x 2
3
2 ( −p)
 1 2k + 1 =– =p
7  = 1
 3 3
7 Constant term
⇒ 2k + 1 = And, product of zeroes, ab =
3 Coefficient of x 2
4 q
⇒ 2k = = =q
3 1
2
⇒ k = α β
3 Now, LHS = +
β α
11 2
65. Let, p(y) = 7y2 − y −
3 3 ( α + β )2 − 2αβ
=
1 αβ
= (21y2 – 11y – 2)
3 On putting a + b = p and ab = q, we get
1
= (21y2 – 14y + 3y – 2) p2 − q p2
3
\ = − 1 = RHS
q q
1
= [7y(3y – 2) + 1(3y – 2)] Hence, proved.
3

Polynomials 9

Click here to buy other Educart books on Amazon - https://amzn.to/40U8Txx


 ANSWER SHEET

SELF PRACTICE
4. (d) no solution a1 b1 c1
= ≠
Explanation: We know that equation of the a2 b2 c2

form ‘y = a’ is a line parallel to the x-axis at a
distance ‘a’ from it. Here, a1 = 3, b1 = –2, c1 = –5

The given pair of equations are y = 0 and (a) For 9x + 8y = 7


y = –7. a2 = 9, b2 = 8, c2 = –7
y = 0 is the equation of x-axis and y = –7 is
a1 3 1 b1 −2 1
the equation of the line parallel to the x-axis. So, = = ; = =−
a2 9 3 b 8 4
So, these two equations represent two parallel 2
lines.
a1 b1
We know that parallel lines never intersect. So, i.e., ¹
there is no solution for these lines. a2 b2

7. (c) x = 18 m, y = 12 m (b) For –12x – 8y = 7



Explanation: Length of the rectangular garden = x a2 = –12, b2 = –8, c2 = –7
Width of rectangular = y a1 3 1 b −2 1
So, = =− ; 1 = =
Perimeter of rectangular garden a2 −12 4 b2 −8 4
= 2x + 2y = 60
a1 b1
x + y = 30 ...(i) i.e., ¹
a2 b2
and x =y+6
x – y = 6 ...(ii) (c) For –12x + 8y = 7

Adding eq. (i) and eq. (ii), a2 = –12, b2 = 8, c2 = –7

2x = 36 a1 3 1 b −2 −1 c1 −5 5
So, = =− ; 1 = = ; = =
a2 −12 4 b2 8 4 c2 −7 7
36
x= = 18
2
a1 b1 c1
i.e., = ¹
Substituting value of x in eq. 1, a2 b2 c2
18 + y = 30,
Condition of two parallel lines is satisfied.
y = 30 – 18 = 12
Hence, the equation of second line is,
8. (b) no solution –12x + 8y = 7
Explanation: Since the lines represented by
12. (d) x – 4y – 14 = 0 and 5x – y – 13 = 0
the given pair of equations are parallel to each
other, so, it has no solution. Explanation: If x = 2 and y = –3 is a unique
solution of any pair of equations, then these
values must satisfy that pair of equations.
x= 0 Putting the values in the equations for every
X´ X
O option and checking it -
x= – 4 For option (a):
Y´ The given equations are
x + y = –1
11. (c) –12x + 8y = 7 and 2x – 3y = –5
Explanation: Given equation of line is 3x – 2y = 5 Putting x = 2, y = –3 in the LHS of the equation
x + y = –1, we get
For two lines to be parallel,
2 – 3 = –1 = RHS

10 Mathematics Class X

Click here to buy other Educart books on Amazon - https://amzn.to/40U8Txx


Putting x = 2, y = –3 in the LHS of the equation 17. (c) a – b
2x – 3y = – 5, we get Explanation: Given, equations are :
2 × 2 – 3 × (–3) = 4 + 9 = 13 ≠ –5 ≠ RHS ax + by = a2 – b2...(i)

Since x = 2, y = –3 is satisfying only one of the and bx + ay = 0  ...(ii)
two equations, option (a) is incorrect. Multiply equation (i) by b and equation (ii) by
Now, for option (b): ‘a’ and subtracting the resulting equations, we

It is a pair of dependent linear equations and get
abx + b2y = a2­b – b3
hence, has infinitely many solutions (not a
abx + a2y = 0
unique solution). So, option (b) is incorrect.
– – –

Now, for option (c): (b2 – a2) y = a2 b – b3

The given equations are, Þ (b2 – a2)y = b(a2 – b2)
2x – y = 1 − b( b2 − a2 )
Þ y=
and 3x + 2y = 0 b2 − a2
Putting x = 2, y = –3 in the LHS of the equation Þ y = –b
2x – y = 1, we get −ay
And x=  (from equation (ii))
2 × 2 – (–3) = 4 + 3 = 7 ≠ 1 ≠ RHS b
Since, x = 2, y = –3 is not satisfying this Þ x=a
equation, option (c) is incorrect. Then, x+y=a–b

Now, for option (d): 21. (b) 1


The given equations are 3x + y = 243 = 35
So, x+y=5 ...(i)
x – 4y –14 = 0
243x – y = 3
and 5x – y – 13 = 0 35(x – y) = 31
Putting x = 2, y = –3 in the LHS of the equation So, 5x – 5y = 1 ...(ii)
x – 4y –14 = 0, we get a1 b1
Since, ≠ , so unique solution
2 – 4 × (–3) – 14 = 2 + 12 – 14 = 0 = RHS a2 b2
Putting x = 2, y = –3 in the LHS of the equation [CBSE Marking Scheme Term-1 SQP 2021]
5x – y – 13 = 0, we get
Explanation: We have,
5 × 2 – (–3) – 13 = 10 + 3 – 13 = 0 = RHS
Þ 3x + y= 243; 243x – y = 3
Since x = 2, y = –3 is satisfying both the Þ 3x + y= 35; 35(x – y) = 31
equations, option (d) is correct. Þ x + y= 5; 5(x – y) = 1
Hence, x = 2, y = –3 is the unique solution for x + y – 5= 0; 5x – 5y – 1 = 0
these equations. [ Base is same, so powers will also be same]
a b c
1 = 1 ; 1 = 1 ; 1 = −5
14. (c) 3 and 1 Now,
a 5 b −5 c −1
2 2 2
Explanation: Since x = a, y = b is the solution
of the equations x – y = 2 and x + y = 4, these a b c

i.e., 1 ≠ 1 ≠ 1
values must satisfy the given pair of equations. a b c
2 2 2

Putting the values in the equations, we have

\ Two equation of lines represent intersecting
a – b = 2  ...(i) lines, which have a unique solution, i.e., number

and a + b = 4  ...(ii) of solutions is one.


Adding equations (i) and (ii), we get 2
25. not equals to −
3
2a = 6
Explanation: For unique solution, we must have
or  a=3
3 2


Putting the value of a in equation (ii), we get 1 −k
3+b =4 2
Þ k¹ −
or  b=1 3

Pair of Linear Equations in Two Variables 11

Click here to buy other Educart books on Amazon - https://amzn.to/40U8Txx


27. (0, –8) Put k = 14 in equation (ii), we get
Explanation: On y-axis, x = 0 6x – 8y = 14
\ Substitute x = 0 in the equation x – y = 8.
Now, divide eq (ii) by 2, we get
Þ 0 – y = 8 Þ y = –8
3x – 4y = 7 ...(iii)
∴ Point = (0, –8)
Therefore, the lines represented by (i) and (iii)
31. 6 shows that the lines are coincident and there
Explanation: Given equations are are infinitely many solutions.
(p – 3) x + 3y = p and px + py = 12 The graph of linear equations a1x + b1y + c1 = 0
For infinitely many solutions, we have and a2x + b2y + c2 = 0 gives a pair of intersecting
p −3 3 p a1 b1
= = lines if ≠ .
p p 12 a2 b2
p −3 3 3 p
Þ = and = Hence, the assertion is true but the reason is
p p p 12
false.
Þ p – 3 = 3 and p2 = 36 40. (B) (c) x + 4y = 22
Þ p = 6 and p = ± 6 [CBSE Marking Scheme Term-1 SQP 2021]
Common value of p is 6. Explanation: Since, Amruta kept book for
So, p =6 6 days and paid `22.
34. False, the given pair of equations will not have \ x + (6 – 2)y = 22
a unique solution for any value of c. or, x + 4y = 22
Explanation: The given pair of linear equations
(D) (d) `3
are: [CBSE Marking Scheme Term-1 SQP 2021]
Explanation: Additional charges for each
x – 2y – 8 = 0
subsequent day = `y = `3 [From ‘C’]
and 5x – 10y – c = 0
41. (B) (a) ` 2 and ` 4
Comparing with a1x + b1y + c1 = 0 and
Explanation: Solving the equations
a2x + b2y + c2 = 0, we have
4x – 3y = –4 and 4x + 3y = 20 by the
a1 = 1, b1 = –2, c1 = –8; method of substitution:
a2 = 5, b2 = –10, c2 = –c. From (i): 4x = 3y – 4 ….(iii)
Substituting in equation (ii),
a1 1 b1 −2 1 c1 −8 8
= ; = = ; = = (3y – 4) + 3y = 20
a2 5 b2 −10 5 c2 −c c 6y – 4 = 20
Now, for c = 40 (any real value), 6y = 24
c1 8 1 y = 4.
= = Substituting y = 4 in equation (iii),
c2 40 5
4x = 3 × 4 – 4 = 8
a1 b1 c1 1 x =2
\ = = =
a2 b2 c2 5 Therefore, cost of one ring game = ` 2 and
Thus, the given pair of linear equations will cost of one balloon game = ` 4
have infinitely many solutions for c = 40. (D) (d) 12 sq. units
Explanation: Let us plot the graph of the
Also, when c ≠ 40, then
lines given by the equations 4x – 3y = –4
a1 b1 c1 and 4x + 3y = 20.
= ≠
a2 b2 c2 For equation 4x – 3y = – 4 :

Thus, the given pair of linear equations will x –1 2 5
have no solution for c ≠ 40. y 0 4 8

Hence, for any value of c, the system of linear
For equation 4x + 3y = 20:
equations does not have a unique solution.
x –1 2 5
37. (c) Assertion (A) is true but reason (R) is false.
y 8 4 0
Explanation: Let 3x – 4y = 7 ...(i)
Plotting these points on the same graph
6x – 8y = k ...(ii) paper, we get the following graph:

12 Mathematics Class X

Click here to buy other Educart books on Amazon - https://amzn.to/40U8Txx


y

P (– 1, 8) C (5, 8)
8

0
=
4
+
6

–3y
4x
5
B (2, 4)
4

4x
3
+
3y
2 –2
0
=0

1 Q(5, 0)
A(–1, 0)
x! x
–1 O 1 2 3 4 5 6 7 8

–1
y!

The vertices of the triangle formed by the 2000 – y + 2y = 2800


two lines and the x-axis are (–1, 0), (2, 4) Or, 2000 + y = 2800
and (5, 0). y = 2800 – 2000 = 800
Area of triangle formed by the two lines Therefore, x = 2000 – 800 = 1200
and the x-axis is shaded and given by
We find that 1,200 children and 800 adults
1 bought tickets to the park on that day.
= × AQ × h
2
43. (A) (c) 4x – 3y = 6; 7x + 3y = 27
1
= ×6×4 Explanation: Here, the number of text
2 books and story books bought by Amrita
= 12 sq. units. are x and y respectively.
42. (A) We have, x = the number of children and \ According to the question,
y = the number of adults who visited the
amusement park on that particular day. 4x = 3y + 6
The total attendance = 2,000. or 4x – 3y = 6 ...(i)
Therefore, x + y = 2000 ...(i)
And, 7x + 3y = 27 ...(ii)
As the ticket for a child is `25 and for an
adult is `50, the total revenue is: (C) (c) 16.5 sq. units
25x + 50y = 70000 Explanation: In order to find the area of
Dividing both sides by 25, we get the triangle formed by the graphs of the
x + 2y = 2800  ...(ii) two equations obtained and the y-axis, we
Hence, the system of linear equations
have to first plot the graphs of the lines
describing the given situation is x + y = 2000
and x + 2y = 2800. represented by the two equations:
For equation 4x – 3y = 6
(B) From equation (i),
x = 2000 – y  ...(iii) x 0 3 1.5
Substituting this value of x in equation (ii), we y –2 2 0
get,

Pair of Linear Equations in Two Variables 13

Click here to buy other Educart books on Amazon - https://amzn.to/40U8Txx


For equation 7x + 3y = 27 7x + 6(3 – x) = 2
x 0 3 6 ⇒ 7x + 18 – 6x = 2
y 9 2 –5 i.e. x = –16
Plotting these points on the same graph From (i), y = 3 + 16 = 19
paper, we get the following graph:
Thus, x = –16 and y = 19 is the required solution.
Y
55. Let, the smaller angle be ‘x’ and the larger
9 (0, 9)
angle be ‘y’.
8

According to the given condition
7x
7 +3

6
y = x + 18º

=
y=

3y
6
or – x + y = 18º  ...(i)


27
5

4x
and x + y = 180º  ...(ii)
4
3
[ Sum of the supplementary angles is 180º]
2 (3, 2)
Now, on adding equations (i) and (ii), we get:
1 – x + y = 18º
X! X x + y = 180º
–5 –4 –3 –2 –1 0 1 2 3 4 5 6
–1 2y = 198
(1.5, 0)
(0, –2) –2 Þ y = 99º
–3
Put the value of y in equation (i), we get
–4
– x + 99° = 18º
–5 (6, –5)
Þ x = 99° – 18º
Y! = 81º

Hence, the two supplementary angles are 81º
The vertices of the triangle formed by the and 99º.
two lines and the y-axis are: (0, –2), (3, 2)
57. Adding the two equations and dividing by 10,
and (0, 9).
we get x + y = 10
1 33
Area of the triangle = × 11 × 3 = = 16.5 Subtracting the two equations and dividing by
= 16.5 sq. units 2 2 –2, we get x – y =1
44. (C) Let number of questions he answered Solving these two new equations, we get
correctly be x. 11
x=
Then, number of questions he answered by 2
guessing = (120 – x).
9
 1 y=
So, x × 1 + (120 – x) ×  −  = 95 2
 4
[CBSE Marking Scheme SQP Std. 2022]
Þ 4x – (120 – x) = 380
Detailed Answer:
Þ 5x = 380 + 120 = 500
The given equations are:
Þ x = 100
Hence, he answered 100 questions 49x + 51y = 499 ...(i)
correctly. 51x + 49y = 501 ...(ii)
On adding the equations (i) and (ii), we get
48. A pair of linear pair has unique solution only 100x + 100y = 1000
when, Dividing by 100
a1 b1
¹ Þ x + y = 10 ....(iii)
a2 b2
On subtracting the equations (ii) from (i), we get
k –1 –2x + 2y = –2
i.e., ≠
6 –2
Dividing by 2
i.e., k ≠ 3.
Þ –x + y = –1 ...(iv)
51. x+y =3 Now, solving equation (iii) and equation (iv), we
get
gives, y =3 – x ...(i)
x + y = 10
So, 7x + 6y = 2 gives –x + y = –1

14 Mathematics Class X

Click here to buy other Educart books on Amazon - https://amzn.to/40U8Txx


On adding, y 9
and x –2 = 7 –2
2y = 9
5
9
=–7
Þ y=
2 y 
Hence, the values of (5y – 2x) and  − 2 are
9 x 
Substituting y = in equation (iii)
2 5
31 and – 7 respectively.
9
Þ x+ = 10 64. The given pair of linear equations is
2
lx + y – l2 = 0
9
Þ x = 10 –
and x + ly – 1 = 0
2
Comparing with a1x + b1y + c1 = 0 and
11 a2x + b2y + c2 = 0, we have
Þ x=
2 a1 = l, b1 = 1, c1 = –l2;
11 9 a2 = 1, b2 = l, c2 = –1;
Therefore, the value of x is and that of y is .
2 2 a1 λ b1 1 c1 − λ2 λ2
= ; = ; = =
a2 1 b2 λ c2 −1 1
Caution
 Students while making the coefficients equal first (A) For no solution,
check their signs carefully to decide whether to add
a1 b1 c1
or subtract the two equations. = ≠
a2 b2 c2
59. The given system of linear equations are:
2x + 3y = 7 2
λ 1 λ
(k – 1)x + (k + 2)y = 3k = ≠

1 λ 1
For infinitely many solutions:
2
a1 b1 c1 λ 1 λ λ
= = = and ≠
a2 b2 c2
1 λ 1 1
l2 – 1 = 0 and l2 ≠ l
Here, a1 = 2, b1 = 3, c1 = –7 (l – 1)(l + 1) = 0 and (l2 – l) ≠ 0
and a2 = (k – 1), b2 = (k + 2), c2 = –3k (l – 1)(l + 1) = 0 and l(l – 1) ≠ 0
2 3 −7 l = 1, –1 and l ≠ 0, 1
Þ = =
k −1 k +2 −3k Here, we take only l = –1.
Hence, for l = –1, the pair of linear
Þ 2(k + 2) = 3(k – 1); 3(3k) = 7(k + 2)
equations has no solution.
Þ 2k – 3k = –3 – 4; 9k – 7k = 14
(B) For infinitely many solutions,
Þ –k = –7; 2k = 14
Þ k = 7; k = 7 a1 b1 c1
= =
Hence, the value of k is 7. a2 b2 c2

60. The given equations are: λ 1 λ2


2x + y = 23 ...(i) = =
1 λ 1

4x – y = 19 ...(ii)
On adding both equations, we get λ 1 λ λ2
= and =
Þ 6x = 42
1 λ 1 1
Þ x=7
Putting the value of x in eq. (i), we get l2 – 1 = 0 and l2 = l
Þ 2(7) + y = 23 (l – 1)(l + 1) = 0 and (l2 – l) = 0
Þ y = 23 – 14 (l – 1)(l + 1) = 0 and l(l – 1) = 0
Þ y=9
l = 1, –1 and l = 0, 1
\ x = 7 and y = 9
Now, 5y – 2x = 5(9) – 2(7) l = 1 satisfies both the equations.
= 45 – 14 Hence, for l = 1, the pair of linear equations
= 31 has infinitely many solutions.

Pair of Linear Equations in Two Variables 15

Click here to buy other Educart books on Amazon - https://amzn.to/40U8Txx


(C) For a unique solution,
Hence, the required values of a and b are 3 and
a1 b 1 respectively for which the given pair of linear
≠ 1 equations has infinitely many solutions.
a2 b2

69. The given pair of equations are:
λ 1
≠ x y
1 λ + − 1 =0
l2 – 1 ≠ 0 10 5
(l – 1)(l + 1) ≠ 0 Þ x + 2y – 10 = 0
l ≠ 1, –1
[Multiplying by 10 on both sides]
Hence, for all real values of l except ±1,
the given pair of equations has a unique Þ x + 2y = 10 ...(i)
solution. x y

and + = 15
66. The given pair of linear equations are: 8 6
x + 2y = 1 Þ 3x + 4y = 360 ...(ii)

and (a - b)x + (a + b)y = a + b – 2 [Multiplying by 24 on both sides]

Þ x + 2y – 1 = 0
Multiplying eq. (i) by 2 and then subtracting it

and (a – b)x + (a + b)y – (a + b – 2) = 0 from eq. (ii), we get

Comparing with a1x + b1y + c1 = 0 and (3x + 4y) – (2x + 4y) = 360 – 20
a2x + b2y + c2 = 0, we have
Þ x = 340
a1 = 1, b1 = 2, c1 = –1

Putting the value of x in eq. (i), we get
a2 = (a – b), b2 = (a + b), c2 = –(a + b – 2)
340 + 2y = 10
a1 1 b1 2

\ = ; = ; Þ 2y = –330
a2 a − b b2 a + b
Þ y = –165
c1 −1 1
= =
\ x = 340 and y = –165.
c2 −( a + b − 2) a + b −2

For infinitely many solutions,
It is given that, y = lx + 5

a1 b1 c1
Putting the values of x and y in the above
= = equation, we get
a2 b2 c2
Þ –165 = l(340) + 5
1 2 1
Þ = = Þ –l(340) = 5 + 165
a−b a+b a + b −2
1 2 Þ –l(340) = 170

Taking, =
a−b a+b 170 1
Þ l= −
=−
Þ a + b = 2(a – b) 340 2
Þ 2a – a = 2b + b
Hence, the solution of the pair of equations is
x = 340, y = –165 and the required value of l is
Þ a = 3b ...(i)
1
− .
2 1 2

Taking, =
a+b a + b −2
72. The given pair of equations is,
Þ 2(a + b – 2) = (a + b)
x + y = 3 and 3x + 3y = 9
Þ 2a + 2b – 4 = a + b
Now, x+y = 3
Þ a+b=4 ...(ii)
Þ y = 3–x

Putting the value of a from eq. (i) in eq. (ii), we get
If x = 0, then y = 3
Þ 3b + b = 4
If x = 3, then y = 0
Þ 4b = 4 If x = 2, then y = 1
Þ b=1 So, the table for x + y = 3 is:

Putting the value of b in eq. (i), we get
x 0 3 2
a = 3(1) = 3
y 3 0 1

The values (a, b) = (3, 1) satisfies all the parts.

16 Mathematics Class X

Click here to buy other Educart books on Amazon - https://amzn.to/40U8Txx


and 3x + 3y = 9 75. The line intersects at two points i.e., (4, 0) and
( 9 − 3x ) (0, – 8)
Þ y= Given, equation is ax + y + 8 = 0
3
If x = 0 then y = 3 Y
if x = 1, then y = 2
if x = 3, then y = 0.
So, the table for 3x + 3y = 9 is:

x 0 1 3
y 3 2 0

Plotting the points A(0, 3) and B(3, 0), we get X

the line AB.


And, plotting the points A(0, 3) and C(1, 2) and
B(3, 0), we get the line ADB.

6
5
4
(A) Now, substitute (4,0) in the given equation
3 A(0, 3)
a(4) + 0 + 8 = 0
2 C(1, 2)
1 D(2, 1) 4a = – 8
X
B(3, 0)
X a=–2
0 1 2 3 4 5 6
–7 –6 –5 –4 –3 –2 –1
–1 (B) The given equation is 4x – 3y – 14 = 0
–2 ⇒ 4x = 3y + 14
–3 3y + 14
–4 x=
4
–5
–6 x 5 6
Y′ y 2 8
Since, the two lines are coincident, the pair of 79. Let the number of students in halls A and B be
equations is consistent with infinitely many x and y respectively.
solutions.

By the given condition, to make the number of
74. Let Meena has received x no. of ` 50 notes and students equal in each hall, 10 students are
y no. of ` 100 notes. sent from A to B.
So, 50x + 100y = 2000 ...(i) i.e.,
x – 10 = y + 10
x + y = 25 ...(ii) Þ x – y = 20 ...(i)
Multiply (ii) by 50 and subtract from (i)
Also, it is given that if 20 students are sent from
50x + 100y = 2000 B to A, the number of students in A becomes
students in B.
50x + 50y = 1250
Þ (x + 20) = 2(y – 20)
– –

Þ x – 2y = –60 ...(ii)
50y = 750

Subtracting eq. (ii) from eq. (i), we get
y = 15 (x – y) – (x –2y) = 20 – (–60)
Putting value of y = 15 in equation (ii) Þ y = 80
x + 15 = 25
Putting the value of y in eq. (i), we get
x = 10 x – 80 = 20
Meena has received 10 notes of ` 50 and 15 Þ x = 100
notes of ` 100.
Hence, 100 students are in hall A and 80
[CBSE Marking Scheme SQP 2020] students are in hall B.

Pair of Linear Equations in Two Variables 17

Click here to buy other Educart books on Amazon - https://amzn.to/40U8Txx



or – 3x + y = 0 ...(i)
1 1
82. Let = a and =b
Five years later:
x y
Þ 21a + 47b = 110
Sumit’s son’s age = (x + 5) years
and 47a + 21b = 162
Sumit’s age = (y + 5) years
Adding and subtracting the two equations, 1
\ (y + 5) = 2 ( x + 5)
we get 2
a + b = 4 and a − b = 2 5
Solving the above two equations, we get
Þ (y + 5) = ( x + 5)
2
a = 3 and b = 1 Þ 2(y + 5) = 5(x + 5)
1 Þ 2y + 10 = 5x + 25
\ x = and y = 1
3
[CBSE Marking Scheme 2019] Þ – 5x + 2y = 15 ...(ii)
84. (A) –3s + h = 1
Multiply equation (i) by 2 and then subtract
equation (ii) from it, we get
s 0 1 2 – 6x + 2y = 0
h 1 4 7 – 5x + 2y = 15
+ – –
8s – 3h = –9
– x = – 15
s 0 3 6
or x = 15

h 3 11 19
Put the value of x in equation (i), we get
– 3 × 15 + y = 0
h Þ y = 45
20
Hence, Sumit’s present age is 45 years.
19 (6, 19)
18 92. Let number of mangoes with A be x and the
17
number of mangoes with B be y.
16
15
According to the question,
14
13 x + 30 = 2 (y – 30)
12

Þ x + 30 = 2y – 60
1
+h=

11 (3,11)

Þ x – 2y = –90 ...(i)
9

10
h=–

–3s

9

and y + 10 = 3(x – 10)
3

8
8s –

7 (2, 7)

Þ y + 10 = 3x – 30
6
5
Þ 3x – y = 40 ...(ii)
(1, 4)

4
(0, 3)
3
Multiplying equation (ii) by 2 and then
2 subtracting it from equation (i), we get
(0, 1)
x – 2y = –90
1
s
0 1 2 3 4 5 6 7 8 9 10 11 12 13 14
6x – 2y = 80
– + –
– 5x = – 170

Þ x = 34
(B) From the graph, the two lines intersect at a
point. So, cat will catch the mouse.
Putting x = 34 in equation (i), we get
(C) Since, the two lines intersect at a point 34 – 2y = –90
(6, 19). – 2y = –90 – 34
\ s = 6 and h = 19
Hence, after 6 seconds at a height of 19 m, – 2y = –124
the cat will catch the mouse. y = 62
90. Let the present age of Sumit’s son be x years So, number of mangoes with A is 34 and
and the present age of Sumit be y years. number of mangoes with B is 62.

According to the given conditions: 94. The given equations of lines are:
y = 3x 3x – y = 3  ...(i)

18 Mathematics Class X

Click here to buy other Educart books on Amazon - https://amzn.to/40U8Txx


2x – 3y = 2  ...(ii)
x 0 2 4
x + 2y = 8  ...(iii)
Let lines (i), (ii) and (iii) represent the sides of a y –4 0 4
∆ABC i.e., AB, BC and CA respectively.
On solving lines (i) and (ii), we will get the Plotting x = 3, x = 5 and 2x – y – 4 = 0, we
obtain three lines I, II and III respectively,
intersection point B.
forming a quadrilateral ABCD with the x-axis
So, multiplying eq. (i) by 3 and then subtracting
as shown below:
eq. (ii) from it, we get
Þ (9x – 3y) – (2x – 3y) = 9 – 2
Y
Þ 7x = 7 8
7
Þ x=1

=0
6 C(5, 6)

x= 3
Putting the value of x in eq. (i), we get

–4
5

–y
4

x= 5
Þ 3×1–y = 3

2x
3
Þ y=0 2 D(3, 2)
Q(2, 0) A (3, 0) B(5, 0)
Hence, the coordinates of point/vertex B are
1
X′ X
(1, 0). –9 –8 –7 –6 –5 –4 –3 –2 –1 1 2 3 4
–1
5 6 7 8

On solving lines (ii) and (iii), we will get the –2


intersection point C. –3
P(0, –4) –4
So, multiplying eq. (iii) by 2 and then subtracting –5
eq. (ii) from it, we get –6
I II

(2x + 4y) – (2x – 3y) = 16 – 2 –7


–8
Þ 7y = 14 III
Y′
Þ y=2
Putting the value of y in eq. (iii), we get From the graph, we get
AB = OB – OA = 5 – 3 = 2
Þ x + 2(2) = 8
AD = 2
Þ x=4 BC = 6
Hence, the coordinates of point/vertex C is (4, 2). We know that the quadrilateral ABCD is a
On solving lines (iii) and (i), we will get the trapezium.
intersection point A. Area of quadrilateral ABCD
So, multiplying eq. (i) by 2 and then adding eq. 1

= × (distance between
(iii) to it, we get 2
parallel lines)
(6x – 2y) + (x + 2y) = 6 + 8 × (sum of parallel sides)
Þ 7x = 14 1
Þ x=2 = × (AB) × (AD + BC)
2
Putting the value of x in eq. (i), we get 1

= × 2 × (2 + 6)
Þ 3×2–y = 3 2
Þ y=3
= 8 sq. units
Hence, the coordinates of point/vertex A are Hence, the area of the required quadrilateral is
(2, 3). 8 square units.
Hence, the vertices of the ∆ABC formed by the 97. Let the money invested in scheme A = ₹ x
given lines are A(2, 3), B(1, 0) and C(4, 2). and the money invested in scheme B = ₹ y
* 95. The given equation of the lines are x = 3, x = 5 Case I:
and 2x – y – 4 = 0 Ruhi invested ₹ x at 8% p.a. + Ruhi invested ₹
For line 2x – y – 4 = 0, we have y at 9% p.a. and received ₹1860 as annual
interest.
y= 2x – 4
We know that simple interest,
If x = 0, y= –4
Principle×Rate×Time
If x = 2, y= 0 SI =
100
If x = 4, y = 4 Interest earned when ₹ x invested at 8% per
So, the table for 2x – y – 4 = 0 is: annum on scheme A,
* Topics and Questions which are a part of latest CBSE Syllabus but have been removed by NCERT.

Pair of Linear Equations in Two Variables 19

Click here to buy other Educart books on Amazon - https://amzn.to/40U8Txx


x × 8 ×1 8x Now,
SI1 = = S.P. of 3 bananas of lot A = `2
100 100
Interest earned when ₹ y invested at 9% per 2
annum on scheme B, Þ S.P. of 1 banana of lot A = `
3
y × 9 ×1 9y
SI2 = 2x
100 = 100 Þ S.P. of x bananas of lot A = `
Interest at 8% per annum on scheme A + Interest 3
at 9% per annum on scheme B = ₹ 1860 Similarly,

8x 9y S.P. of 1 banana of lot B = `1


Þ + = 1860
100 100 Þ S.P. of y bananas of lot B = `y
Þ 8x + 9y = 186000 ...(i) As per given condition
Case II:
2x
Ruhi invested ₹ y at 8% p.a. + Ruhi invested ₹ x + y = 400
3
at 9% p.a. and received ₹ (1860 + 20) as annual Þ 2x + 3y = 1200  ...(i)
interest.
Case II:
Interest earned when ₹ y invested at 8% per
annum on scheme A, Sold the first lot at the rate of `1 per banana
+ Sold the second lot at the rate of `4 for 5
y × 8 ×1 8y bananas and received `460.
SI1 =
100 = 100 S.P. of 1 banana of lot A = `1
Interest earned when ₹ x invested at 9% per
annum on scheme B, Þ S.P. of x bananas of lot A = `x
Similarly,
x × 9 ×1 9x
SI2 = = 100 S.P. of 5 bananas of lot B = ` 4
100
Interest at 8% per annum on scheme A + 4
Þ S.P. of 1 banana of lot B = `
Interest at 9% per annum on scheme B = ₹ 1880 5
8y 9x 4y
Þ + = 1880 Þ S.P. of y bananas of lot B = `
100 100 5
Þ 9x + 8y = 188000  ...(ii) As per the given condition
Multiplying eq. (i) by 9 and eq. (ii) by 8 and then 4y
x + 5 = 460
subtracting them, we get
(72x + 81y) – (72x + 64y) Þ 5x + 4y = 2300  ...(ii)
= 1674000 – 1504000 Multiplying eq. (i) by 4 and eq. (ii) by 3 and then
subtracting them, we get
Þ 81y – 64y = 170000
(8x + 12y) – (15x + 12y) = 4800 – 6900
Þ 17y = 170000
Þ –7x = –2100
Þ y = 10000
Þ x = 300
Putting the value of y in eq. (i), we get
Putting the value of x in eq. (i), we get
Þ 8x + 9(10000) = 186000
2(300) + 3y = 1200
Þ 8x = 186000 – 90000
Þ 3y = 1200 – 600
Þ 8x = 96000
Þ 3y = 600
Þ x = 12000
Þ y = 200
Hence, Ruhi invested ₹ 12000 and ₹ 10000 in
schemes A and B respectively. Total number of bananas
= Number of bananas in lot A
98. Let the number of bananas in lot A = x + Number of bananas in lot B
and the number of bananas in lot B = y. = (x + y)
Case I: = (300 + 200)
Sold the first lot at the rate of ₹2 for 3 bananas = 500
+ Sold the second lot at the rate of ₹1 per Hence, the total number of bananas he had is
banana and received `400. 500.

20 Mathematics Class X

Click here to buy other Educart books on Amazon - https://amzn.to/40U8Txx


 ANSWER SHEET

SELF PRACTICE
3. (d) x3 – x2 = (x – 1)3 (a) 2x2 – 3x + 6 = 0
Explanation: Simplify the given equations. a = 2, b = –3, c = 6
(a) x2 + 2x + 1 = (4 – x)2 + 3
Þ x2 + 2x + 1 = 16 + x2 – 8x + 3 b −3 3
Þ Sum of its roots = – a = – =
Þ 10x = 18 2 2
(b) –x + 3x – 3 = 0
2
Þ 10x – 18 = 0
a = –1, b = 3, c = –3
This equation is not of the form ax2 + bx + c = 0,
a ≠ 0. b  3
Þ Sum of its roots = – a = –   = 3
Thus, it is not a quadratic equation.  −1
3
(c) 2 x –
2
x+1=0
 2 2
(b) –2x2 = (5 – x)  2 x − 
 5 3
Þa= 2,b=– ,c=1
2
2x  −3 
Þ –2x2 = 10x – 2x2 – 2 +
5   3
b
Þ 50x + 2x – 10 = 0 Þ Sum of its roots = – a = – 2 =
2 2
Þ 52x – 10 = 0
This is also not a quadratic equation as it is (d) 3x2 – 3x + 3 = 0
also not of the form ax2+ bx + c = 0, a ≠ 0. Þ a = 3, b = –3 and c = 3
3 b −3
(c) (k + 1)x2 + x = 7, where k = – 1 Þ Sum of its roots = – a = – =1
2 3
3x Concept applied
Þ (–1 + 1)x2 + =7
2  For a quadratic polynomial ax2 + bx + c, or for a
quadratic equation ax2 + bx + c, a ¹ 0, we have
3x
Þ 0+ =7 Coefficient of x
2 Sum of its zeroes/roots = −

2
Þ 3x – 14 = 0 Coefficient of x
This is also not a quadratic equation as it is
also not of the form ax2+ bx + c = 0, a ≠ 0. b
= −
(d) x – x = (x – 1)
3 2 3 a

x3 – x2 = x3 – 3x2(1) + 3x(1)2 – (1)3 9. (a) – 1 ,1


[ (a – b)3 = a3 – 3a2b +3b2a – b3] 2
1 1
Þ x – x = x – 3x + 3x – 1
3 2 3 2 Explanation: − x 2 + x + =0
2 2
Þ –x2 + 3x2 – 3x + 1 = 0
– 2x2 + x + 1 =0
Þ 2x2 – 3x + 1 = 0 Þ – 2x + 2x – x + 1
2
=0
This represents a quadratic equation Þ – 2x (x – 1) – 1 (x – 1) =0
because it is of the form ax2 + bx + c = 0,
Þ (x – 1) (– 2x – 1) =0
a ≠ 0.
1
Þ x = 1, x = –
7. (b) –x2 + 3x – 3 = 0 2
Explanation: We know that the sum of the
b 10. (b) x2 + x – 5 = 0
roots = – a

Explanation: We know that, if D = b2 – 4ac > 0
On comparing the given equations with
for a quadratic equation ax2 + bx + c = 0, then
ax2 + bx + c = 0: its roots are real and distinct.

Quadratic Equations 21

Click here to buy other Educart books on Amazon - https://amzn.to/40U8Txx


(a) The given equation is: 16. (c) no real roots
9 Explanation: The given equation is:
2x2 – 3 2x + 4 = 0
9 (x2 + 1)2 – x2 = 0
Þ a = 2, b = –3 2 and c = 4
Þ x + 1 + 2x2 – x2 = 0
4
Þ D = b2 – 4ac
 9 [(a + b)2 = a2 + b2 + 2ab]
= (–3 2) – 4(2)   2
 4 Þ x4 + x2 + 1 = 0
= 18 – 18 = 0

Let, x2 = y
So, the equation has real and equal roots.
Then, (x2)2 + x2 + 1 = 0
(b) The given equation is:
x2 + x – 5 = 0 Þ y2 + y + 1 = 0
Þ a = 1, b = 1 and c = –5
On comparing with ay2 + by + c = 0, we get
Þ D = b2 – 4ac

a = 1, b = 1, c = 1
= (1)2 – 4(1)(–5)
= 1 + 20 = 21 > 0 Þ D = b2 – 4ac
So, the equation has two distinct real roots. = (1)2 – 4(1)(1)
(c) The given equation is: = –3 < 0
x2 + 3x + 2 2 = 0

As D < 0, thus, we can say equation has no real
Þ a = 1, b = 3 and c = 2 2 roots.
Þ D = b2 – 4ac
= (3)2 – 4(1)(2 2) 21. a ≠ 0.
= 9 – 8 2 < 0 Explanation:
So, the equation has no real roots. In case a = 0, the equation reduces to bx + c = 0,
(d) The given equation is: which is a linear equation.
5x2 – 3x + 1 = 0
23. no.
Þ a = 5, b = –3 and c = 1
Þ D = b2 – 4ac Explanation:
= (–3)2 – 4(5)(1) Here, b2 – 4ac = (1)2 – 4(2)(4) = 1 – 32, which is
= 9 – 20 less than 0.
= –11 < 0 27. (a) Both assertion (A) and reason (R) are true
So, the equation has no real roots. and reason (R) is the correct explanation of
assertion (A).
Concept Applied
Explanation: The given equation is,
 A quadratic equation ax2 + bx + c = 0, a ¹ 0 has:
(1) two distinct real roots if b2 – 4ac > 0. 2x2 – 3x + 5 = 0
(2) two equal roots if b2 – 4ac = 0.
−b
(3) no real roots if b2 – 4ac < 0. a+b =
a
15. (c) no real roots
Explanation: The given equation is: −( −3) 3
= =
2 2
2x2 – 5x+1=0

On comparing it with ax2 + bx + c = 0, we get c 5
ab = =
a = 2, b = – 5 , c = 1 a 2
Discriminant, D = b2 – 4ac And we knew that, if a and b are the roots of
= (– 5 ) – 4(2)(1) 2 a2 + bx + c = 0, a ≠ 0, then sum of roots, a + b is

D = 5 – 8 = –3 −b
equal to and product of roots, ab is equal
a
Þ D = –3 < 0
We know that if D = b2 – 4ac < 0 for a quadratic c
to .
equation ax2 + bx + c = 0, then roots are not a
real. Hence, both assertion and reason are true and

Here, the discriminant is negative, hence, the the reason is the correct explanation of the
given equation has no real roots. assertion.

22 Mathematics Class X

Click here to buy other Educart books on Amazon - https://amzn.to/40U8Txx


30. (a) Both assertion (A) and reason (R) are true b 2b
(D) (d) ,
and reason (R) is the correct explanation of a a
assertion (A). Explanation: Let us factorise the quadratic
Explanation: 8x2 + 3kx + 2 =0 equation.
Discriminant, D = b2 – 4ac a2x2 – 3abx + 2b2 = 0
Þ a x – abx – 2abx + 2b2 = 0
2 2
D = (3k)2 – 4 × 8 × 2
Þ ax(ax – b) – 2b(ax – b) = 0
= 9k2 – 64 Þ (ax – 2b)(ax – b) = 0
For equal roots,
The roots are x = b , 2b
D=0 a a
9k2 – 64 = 0
9k2 = 64 33. (B) Let the length of each side of the chess
board = x cm.
64
k2 =  Side of inner square = x – 3 × 2
9
=x–6
8
k= ± As the width of the border is 3 cm, we can
3
write the equation as (x – 6)2 = 400.
In the quadratic equation ax2 + bx + c = 0; If
Therefore,
D = b2 – 4ac = 0 then the roots are real and
equal. x2 – 12x + 36 = 400
Þ x – 12x – 364 = 0
2
Hence, both assertion and reason are true
and reason is the correct explanation of the Þ x – 26x + 14x – 364 = 0
2

assertion. Þ x(x – 26) + 14(x – 26) = 0

31. (B) To find the length of the shortest side, we Þ (x – 26)(x + 14) = 0
have to find the roots of the quadratic equation Þ x = 26, – 14
x2 – 6x – 7 = 0 As length cannot be negative, length of
each side of the chess board = 26 cm.
Þ x – 7x + x – 7 = 0
2

Þ x(x – 7) + 1(x – 7) = 0 2
(C) As x = and x = –3 are roots of the
3
Þ (x – 7) (x + 1) = 0 quadratic equation ax2 + 7x + b = 0,
The roots of the quadratic equation are 7, –1. 2
 2  2
As length cannot be negative, the length of a   + 7  + b = 0
 3  3
the shortest side = x = 7 m.
4 14
(C) From (B) Þ a+ +b = 0
9 3
Length of shortest side = 7 m Þ 4a + 9b = –42 ...(i)
Þ Length of third side = 3x + 3 = 3(7) + 3 Similarly, a(–3)2 + 7(–3) + b = 0
= 24 m Þ 9a – 21 + b = 0
32. (B) (a) 3 cm and 11 cm Þ 9a + b = 21
Explanation: To find the radii of the two Þ b = 21 – 9a...(ii)
circles, we have to find the roots of the Substituting this value of b in equation (i),
quadratic equation x2 – 14x + 33 = 0. we get
x2 – 14x + 33 = 0 4a + 9(21 – 9a) = –42
Þ x – 11x – 3x + 33 = 0
2 Þ 4a + 189 – 81a = –42
Þ x(x – 11) – 3(x – 11) = 0 Or, –77a = –231
Þ (x – 3)(x – 11) = 0 Þ a = 3.
Therefore, x = 3 or 11. Substituting the value of a = 3 in equation
If radius of first circle = 3 cm, then radius of (ii), we get
second circle = (14 – 3) cm = 11 cm b = 21 – 9 × 3
If radius of first circle = 11 cm, then radius = 21 – 27
of second circle = 3 cm. = – 6

Quadratic Equations 23

Click here to buy other Educart books on Amazon - https://amzn.to/40U8Txx


34. (C) (c) x2 + 30x – 400 = 0 According to the question,
Explanation: We have, 1500 1500 1
− =
Speed of boat in upstream = (20 – x) km/h x x + 250 2
and, speed of boat in downstream 1 1  1
= (20 + x) km/h 1500  −  =
 x x + 250  2
∴ Time taken to cover 15 km upstream.
 x + 250 − x  1
15 Þ 1500   =
t1 = hr x ( x + 250 ) 2
20 − x  
And time taken to cover 15 km downstream, Þ 750000 = x(x + 250)
Þ x2 + 250x – 750000 = 0,
15 which is the required quadratic equation.
t2 =
20 + x
38. Given equation is: 4x2 – 12x – k = 0
According to the statement. For equation to have no real roots,
Þ t1 = t2 + 1
D<0
t 1 – t2 = 1
or b2 – 4ac < 0
15 15 Here, a = 4, b = – 12, c = – k
Þ – = 1
20 − x 20 + x Þ (– 12)2 – 4 × 4 × (– k) < 0
Þ 144 + 16k < 0
 20 + x − 20 + x 
Þ 15   = 1 Þ 16k < –144
 (20 − x ) (20 + x )  Þ k < –9
Þ 15 (2x) = 400 – x2
Hence, for all values of k less than –9, the given
Þ 30x = 400 – x2 equation has no real roots.
Þ x2 + 30x – 400 =0
(E) (c) 30 minutes 42. Given quadratic equation is:
6x2 – x – k = 0.
Explanation: From part (C) we have 2
Its one of its root is .
Time taken to cover 15 km downstream 3
If x = 2 is root of the given equation, then it
15 3
t2 =
20 + x will satisfy the given equation.
2
15 2 2

\ 6  − − k =0
t2 = 3 3
20 +10
4 2
[Q x = 10 using part (D)] Þ 6 × − − k =0
9 3
15 8 2
= Þ − − k =0
30 3 3
1 Þ k =2
= hour Hence, the value of k is 2.
2
= 30 minutes 46. Given, quadratic equation is
35. (A) We have, px2 – 2 5 px + 15 = 0
Usual speed of plane = x km/hr
Here, a = p, b = – 2 5 p, c = 15
Since, speed of plane was increased by
250 km/hr, Now, D = b2 – 4ac
So, increased speed of plane
( )
2
= (x + 250) km/hr = −2 5p – 4 × p × 15
Now, we know,
= 20p – 60p 2

Distance
Time = Since, roots are equal.
Speed
Then, D=0
So, time taken to cover 1500 km with usual

Þ 20p – 60p = 0 2

1500
speed = hr
Þ 20p (p – 3) = 0
x

Þ p = 0 or p = 3
and, time taken to cover 1500 km with
But p cannot be 0.
1500
increased speed = hr
\ p=3
x + 250

24 Mathematics Class X

Click here to buy other Educart books on Amazon - https://amzn.to/40U8Txx


52. 8x2 – 2x – 3 = 0 67. (A) The given equation is: 5x2 + 13x + 8 = 0
⇒ 8x2 – 6x + 4x – 3 = 0 On comparing with ax2 + bx + c = 0, we get
⇒ 2x(4x – 3) + 1(4x – 3) = 0 a = 5, b =13, c = 8
⇒ (4x – 3) (2x + 1) = 0 By quadratic formula:

⇒ 4x – 3 = 0 or 2x + 1 = 0 −b ± b2 − 4ac
x=
3 −1 2a
i.e., x= or
4 2
− ( + 13) ± (13)2 − 4 (5) ( 8 )
=
59. Given, quadratic equation is 2 (5)
−13 ± 169 − 160
2x2 + 7x + 5 2 = 0 =
10
On comparing the above equation with
ax2 + bx + c = 0, we get −13 ± 9
=
10
a= 2 , b = 7, c = 5 2
−13 ± 3
Then, =
10
discriminant, D = b2 – 4ac
–13 + 3 –13 – 3
2 Þ x= 10 or x = 10
= (7) − 4 × 2×5 2
10 16 8
= 49 – 40 = 9 Þ x = – 10 = –1 or x = – 10 = – 5
Now, roots of the quadratic equation are given
Hence, roots of the given equation are –1
by, 8
and – 5 .
−b ± D
x= (B) The given equation is: –3x2 + 5x + 12 = 0
2a
On comparing with ax2 + bx + c = 0, we get
−7 ± 9
= a = –3, b = 5 and c = 12
2 2
By quadratic formula,
−7 ± 3
=
2 2 −b ± b2 − 4ac
x=
2a
−4 −10
∴ x= and
2 3 2 3 −5 ± (5)2 − 4( −3)(12)
=
2( −3)
−2 −5
= and −5 ± 25 + 144
33 =
−2 −6
Hence, the roots of the given equation is
3 −5 ± 169
−5 =
and .
3 −6
61. Given, quadratic equation is: −5 ± 13
=
x2 + 2xk + (k2 – k + 2) = 0 −6
On comparing the quadratic equation with –5 + 13 –5 – 13
Þ x= –6 or x = –6
ax2 + bx + c = 0, we get
a = 1, b = 2k, c = k2 – k + 2 8 = 4 –18
Þ x = –6 –3 or x = – –6 = 3
Since, the roots of the above equation are real
Hence, the roots of the given equation are
and equal. 4
– 3 and 3.
∴ Discriminant, D = 0
i.e., b2 – 4ac = 0 (C) The given equation is: –x2 + 7x – 10 = 0
(2k) – 4 × 1 × (k – k + 2) = 0
2 2 On comparing with ax2 + bx + c = 0, we get
⇒ 4k2 – 4k2 + 4k – 8 = 0 a = –1, b = 7, c = –10
By quadratic formula,
⇒ 4k – 8 = 0
⇒ k =2 − b ± b2 − 4ac
x=
Hence, the value of k is 2. 2a

Quadratic Equations 25

Click here to buy other Educart books on Amazon - https://amzn.to/40U8Txx


−7 ± (7)2 − 4( −1)( −10 ) 3 5 ± 5
= =
2( −1) 2

−7 ± 49 − 40 3 5+ 5 3 5– 5
Þ x= 2 or x= 2
=
−2
4 5 2 5
−7 ± 9 Þ x= 2 = 2 5 or x = 2 = 5
=
−2 Hence, the roots of the given equation are
−7 ± 3 2 5 and 5 .
= 1
−2 (F) The given equation is: 2 x2 – 11 x + 1 = 0
–7 + 3 –7 – 3
Þ x= –2 or x = –2 On comparing with ax2 + bx + c = 0, we get
–4 –10 1
Þ x = –2 = 2 or x = –2 = 5 a = 2 , b = – 11 , c = 1

Hence, roots of the given equation are 2 By quadratic formula,


and 5.
− b ± b2 − 4ac
(D) The given equation is: x2 + 2 2x – 6 = 0 x=
2a
On comparing with ax2 + bx + c = 0, we get

( ) (− 11) − 4  12 (1)


2
a = 1, b = 2 2, c = –6 − − 11 ±
By quadratic formula,
=
 1
− b ± b2 − 4ac 2 
x=  2
2a
11 ± 11 − 2
−2 2 ± (2 2 )2 − 4(1)( −6 ) =
=  1
2(1) 2 
 2
−2 2 ± 8 + 24
=
2 11 ± 9
=
–2 2 ± 32 1
= 2
11 ± 3
=
−2 2 ± 4 2 1
=
2 Þ x = 11 + 3 or x = 11 – 3
Hence, roots of the given equation are
2( − 2 ± 2 2 )
= 11 + 3 and 11 – 3.
2
= − 2 ±2 2 Concept Applied
 Compare the given equation with ax2 + bx + c = 0,
to get a, b and c. Then, use the following quadratic
Þ x=– 2 +2 2 or x = – 2 – 2 2
formula for finding roots of the equation.
Þ x= 2 or  x = –3 2
− b ± b2 − 4ac
Hence, the roots of the given equation are x=
2a
2 and –3 2.
68. (B) The given equation is: 2 3
5 x –x– 5 =0
2
(E) The given equation is: x2 – 3 5 x + 10 = 0
Multiplying both the sides by 5, we get
On comparing with ax2 + bx + c = 0, we get
Þ 2x2 – 5x – 3 = 0
a = 1, b = –3 5 , c = 10
Splitting the middle term, we have
By quadratic formula,
Þ 2x2 – 6x + x – 3 = 0
2
− b ± b − 4ac Þ 2x(x – 3) + 1(x – 3) = 0
x=
2a Þ (2x + 1)(x – 3) = 0

(−3 5 ) − 4(1)(10)
2
−( −3 5 ) ± Þ 2x + 1 = 0 or x – 3 = 0
= 1
2(1) Þ x = – 2 or x = 3
3 5 ± 45 − 40 1
= Hence, the roots of the equation are – 2
2 and 3.

26 Mathematics Class X

Click here to buy other Educart books on Amazon - https://amzn.to/40U8Txx


(D) The given equation is: 3x2 + 5 5 x – 10 = 0 ⇒ (x – 6) (x – 11) = 0
Splitting the middle term, we have ∴ x = 6 or 11
When, x = 6, then y = 11 and when, x = 11, then
3x2 + 6 5 x – 5 x – 10 = 0
y=6
Þ 3x2 + 6 5 x – 5 x – (2 5 )( 5 ) = 0
Hence, the sides of the two squares are 6 m
Þ 3x(x + 2 5 ) – 5 (x + 2 5 ) = 0 and 11m.
Þ (x + 2 5 )(3x – 5 ) = 0 77. Let ABCD be rectangular lawn and EFGH be
Þ (x + 2 5 )= 0 or (3x – 5 ) = 0 rectangular pound. Let x m be the width of
5 grass area, which is same around the pound.
Þ x = –2 5 or x = 3 D C
Hence, roots of the given equation are –2 5 x
H G
5
and 3 .
1
(E) The given equation is: 21x2 – 2x + 21 = 0 x x
Multiplying both the sides by 21, we get
1
(21)(21)x2 – (21)(2x) + 21 (21) = 0 E F
x
Þ 441x2 – 42x + 1 = 0 A B
Splitting the middle term, we have
Given, Length of lawn = 50 m
441x2 – 21x – 21x + 1 = 0
Width of lawn = 40 m
21x(21x – 1) – 1(21x – 1) = 0

Þ Length of pond = (50 – 2x) m
(21x – 1)(21x – 1) = 0
1 1 Breadth of pond = (40 – 2x) m
Þ  x = 21 or x = 21
Also given,
Hence, the roots of the given equation are Area of grass surrounding the pond = 1184 m2
1 1
21 and 21 . Area of rectangular lawn – Area of Pond
= 1184 m2
Concept Applied

Þ 50 × 40 – {(50 – 2x) × (40 – 2x) = 1184
 If any coeffiicient of quadratic equation ax2 + bx + c = 0
is in fractional form, make all coefficients in integral
Þ 2000 – (2000 – 80x – 100x + 4x2) = 1184
form.
Þ 2000 – 2000 + 180x – 4x2 = 1184
75. Let ‘x’ and ‘y’ be the lengths of the sides of the
Þ 4x2 – 180x + 1184 = 0
two squares.
Þ x2 – 45x + 296 = 0
Then, area of first square = (side)2 = x2

Þ x2 – 37x – 8x + 296 = 0
And, area of second square = (side)2 = y2

Þ x(x – 37) – 8(x – 37) = 0
According to the question,

Þ (x – 37) (x – 8) = 0
x2 + y2 = 157 ...(i)
Now, perimeter of first square = 4 × side = 4x
Þ x – 37 = 0 or x – 8 = 0
Perimeter of second square = 4 × side = 4y
Þ x = 37 or x = 8
According to the question,
x = 37 is not possible as in this case length of
4x + 4y = 68 pond becomes 50 – 2 × 37 = –24 (not possible)
or x + y = 17 Hence, x = 8 is acceptable
⇒ y = 17 – x ...(ii)
\ Length of pond = 2 × 8 = 34 m
Substituting the value of ‘y’ from equation (ii), in Breadth of pond = 40 – 2 × 8 = 24 m
equation (i), we get
78. Let the length of AC be x
x2 + (17 – x)2 = 157
Then, BC = 2 – x
⇒ x + 289 + x – 34x – 157 = 0
2 2
Now, AC2 = AB × CB (given)
⇒ 2x2 – 34x + 132 = 0
∴ x2 = 2 × (2 – x)
⇒ x2 – 17x + 66 = 0 [ AB = 2 m (given)]
⇒ x – 11x – 6x + 66 = 0
2
⇒ x2 = 4 – 2x
⇒ x(x – 11) – 6(x –11) = 0 ⇒ x2 + 2x – 4 = 0

Quadratic Equations 27

Click here to buy other Educart books on Amazon - https://amzn.to/40U8Txx


Now, if we compare the above equation with 9( a + b ) ± 3( a − b )
ax2 + bx + c = 0. =
18
Then, a = 1, b = 2, c = –4
3( a + b ) ± (a − b )
Root of the equation are, =
6
−b ± b2 − 4ac 3a + 3b + a − b
x = =
2a 6

or 3a + 3b − a + b
2
−2 ± (2) − 4 × 1 × ( −4)
= 6
2 × 1
4a + 2b 2a + 4b
−2 ± 20 = or
= 6 6
2
2a + b a + 2b
= or
−2 ± 2 5 3 3
=
2 Hence, the roots of the given equation are
= −1 ± 5 2a + b a + 2b
and .
3 3
= − 1 + 5 or − 1 − 5 85. The given equation can be rewritten as
When x = − 1 + 5, (k + 1)x2 – 2(k – 1)x + 1 = 0

So, it will have equal roots, when
Then, BC = 2 − −1 + 5 = 3 − ( ) 5
Discriminant = 0
= 3 – 2.24 = 0.76 ⇒ b2 – 4ac = 0
[–2(k – 1)] – 4(k + 1) (1) = 0
2
When x = −1 − 5, ⇒ 4(k – 1)2 – 4(k + 1) = 0
⇒ 4k2 – 8k + 4 – 4k – 4 = 0
BC = 2 − −1 − ( )
5 =3 + 5 ⇒ 4k2 – 12k = 0
= 3 + 2.24 = 5.24 ⇒ 4k(k – 3) = 0
(which is not possible) ⇒ k = 0 or k = 3
For k = 3, the equation is
Hence, the length of BC is 3 − 5 or 0.76 m. (3 + 1)x2 + 1 – 2(3 – 1)x = 0
82. 9x2 – 9(a + b)x + (2a2 + 5ab + 2b2) = 0 ⇒ 4x2 – 4x + 1 = 0
⇒ (2x) – 2 × 2x × 1 + (1)2 = 0
2

On comparing the given equation with
⇒ (2x – 1)2 = 0

Ax2 + Bx + C = 0,
1 1

we get A = 9, B = – 9(a + b), C = 2a2 + 5ab + 2b2 ⇒ x= ,
2 2

Discriminant, 89. Let n be the required natural number.
D = B2 – 4AC Number when increased by 12 is n + 12.
= [– 9(a + b)]2 – 4 × 9 × (2a2 + 5ab + 2b2)  1  160

160 times number’s reciprocal = 160   = n

= 81(a + b)2 – 36(2a2 + 5ab + 2b2)  n

Now, by the given condition
= 81(a2 + b2 + 2ab) – 72a2 – 180ab – 72b2
160
n + 12 = n
= 81a2 + 81b2 + 162ab – 72a2 – 180ab – 72b2
= 9a2 + 9b2 – 18ab Þ n(n + 12) = 160
Þ n2 + 12n – 160 = 0
= 9(a2 + b2 – 2ab)
Splitting the middle term, we have
= 9(a – b)2
n2 + 20n – 8n – 160 = 0
= [3(a – b)]2
Þ n(n + 20) – 8(n + 20) = 0
−b ± D Þ (n + 20)(n – 8) = 0

Then, x=
2a
Þ n = –20 or 8
2 2 But n ≠ –20 as n is a natural number.
9( a + b ) ± 3 ( a − b )
=
2×9
Hence, the required natural number is 8.

28 Mathematics Class X

Click here to buy other Educart books on Amazon - https://amzn.to/40U8Txx


90. Let ‘x’ km / hr be the usual speed of the plane. Hence, the present age of Nisha = 5 years and
\ Increased speed = (x + 250) km/hr the present age of Asha = x2 + 2
 1500  = 52 + 2
Time taken at the usual speed =  hr
 x  = 25 + 2 = 27 years.
 1500 
94. Let, the marks of Arun in Hindi be x.
Time taken at increased speed =   hr Then, marks in English = 30 – x
x + 250 
According to the given condition,
Difference in the duration of two flights = 30
Þ (x + 2) (30 – x – 3) = 210
1 Þ (x + 2) (27 – x) = 210
minutes = hr
2 Þ 27x + 54 – x2 – 2x = 210
1500 1500 1 Þ x2 – 25x + 156 = 0
∴ − =
x x + 250 2 Þ x − 13x −12x + 156 = 0
2

1 1 1 Þ x (x − 13) − 12(x − 13) = 0


− =
⇒ x x + 250 3000 Þ (x – 13) (x – 12) = 0
Þ x = 13 or 12
x + 250 − x 1
= When x = 13, then
⇒ (
x x + 250 ) 3000
Marks in Hindi = 13
⇒ x2 + 250 x = 250 × 3000
Marks in English = 30 – 13 = 17
⇒ x + 250 x – 750000 = 0
2
When x = 12, then
⇒ x + 1000 x – 750 x – 750000 = 0
2
Marks in Hindi = 12
⇒ x (x + 1000) – 750 (x + 1000) = 0
Marks in English = 30 – 12 = 18
⇒ (x – 750) (x + 1000) = 0
Hence, the marks obtained in the two subjects
⇒ x = 750 or – 1000 are (13, 17) or (12, 18).
⇒ x = 750 [ speed cannot be negative]
Hence, the usual speed of the plane is 750 km/hr. 98. Let, the two consecutive odd natural numbers
be x and (x + 2).
93. Let Nisha’s present age be x years.

According to the given condition,

Then, Asha’s present age = (2 + x2)
x2 + (x + 2)2 = 394
[By the given condition]
Þ x2 + x2 + 4 + 4x = 394

Now, when Nisha grows to her mother’s
Þ 2x2 + 4x – 390 = 0
present age i.e. after {(x2 + 2) – x} years,
Þ x2 + 2x – 195 = 0

Then, Asha’s age will become
Þ x2 + 15x – 13x – 195 = 0
[{(x2 + 2) – x} + (2 + x2)] years.
Þ x(x + 15) – 13 (x + 15) = 0

Now by the given condition, Þ (x – 13) (x + 15) = 0
Asha’s age = 1 year less than 10 times the Þ x = 13, – 15
present age of Nisha.
But x ¹ –15

(2 + x2) + {(x2 + 2) – x} = 10x – 1 [natural numbers are not negative]
Þ 2 + x2 + x2 + 2 – x = 10x – 1 Þ x = 13
Þ 2x2 – 11x + 5 = 0
Hence, the two consecutive odd natural
Splitting the middle term, we have numbers are 13 and 15.
Þ 2x2 – 10x – x + 5 = 0 a b 2c
100. + =
Þ 2x(x – 5) – 1(x – 5) = 0 x −a x −b x −c
Þ (x – 5)(2x – 1) = 0 a( x − b ) + b( x − a ) 2c
Þ =
Þ (x – 5)(2x – 1) = 0 ( x − a )( x − b ) x −c
1 ax − ab + bx − ab 2c
Þ x = 5 or x = 2

Þ =
1 1
2
x − ax − bx + ab x −c

But x ≠ 2 as then Nisha’s age = 2 . This
Þ (ax – 2ab + bx) (x – c) = (x2 – ax – bx + ab) (2c)

means that her mother Asha’s age = (x2 + 2)
Þ ax2 – 2abx + bx2 – acx + 2abc – bcx

1 
=  + 2 = 2 1 years which is not possible. = 2cx2 – 2acx – 2bcx + 2abc
4  4

Quadratic Equations 29

Click here to buy other Educart books on Amazon - https://amzn.to/40U8Txx


Þ ax2 + bx2 – 2cx2 – 2abx – acx + 2acx – bcx +
⇒ x(x – 200) = 2 × 12000
2bcx = 0 ⇒ x(x – 200) – 240000 = 0
Þ (a + b – 2c)x2 + (ac + bc – 2ab)x = 0
⇒ x2 – 200x – 240000 = 0
Þ x[(a + b – 2c)x + (ac + bc – 2ab)] = 0
⇒ (x – 600) (x + 400) = 0
Þ (a + b – 2c)x + (ac + bc – 2ab)] = 0, or x = 0
⇒ x – 600x + 400x – 240000 = 0
2

Þ (a + b – 2c) x + (ac + bc – 2ab) = 0


⇒ x(x – 600) + 400(x – 600) = 0

Þ (a + b – 2c)x = – (ac + bc – 2ab) ⇒ x – 600 = 0
−( ac + bc − 2ab ) ( x + 400 ≠ 0)

Þ x=
a + b − 2c ⇒ x = 600
2ab − ac − bc Thus, the speed of the aircraft originally was
=
( a + b − 2c ) 600 km/hr.
= 2ab
108. Let the smaller tap fills the tank in x hrs.
− c (a + b) \ The larger tap fills the tank in (x – 2) hrs.

Hence, the ‘x’ in terms of a, b, c is
( a + b − 2c )
Time taken by both the taps together:
103. Let the sides of two squares be ‘x’ metres and ‘y’ 15
metres such that x > y. = hrs.
8
Then, x2 + y2 = 544 So, in one hour, amount of water filled in the
and 4x – 4y = 32, or x – y = 8  1  8
tank = =
 15  15
⇒ x2 + (x – 8)2 = 544 [ y = x – 8]  
 8 
⇒ x + x – 16x + 64 = 544
2 2

⇒ 2x2 – 16x – 480 = 0 Now, work done by the smaller tap in an hour:
⇒ x2 – 8x – 240 = 0 1
=
⇒ x2 – 20x + 12x – 240 = 0 x
Work done by the larger tap in an hour:
⇒ (x – 20) + 12(x – 20) = 0
1
⇒ (x – 20) (x + 12) = 0 =
x −2
⇒ x – 20 = 0 ( x + 12 ≠ 0)
Now, according to the given condition:
⇒ x = 20
1 1 8
So, y = 20 – 8 = 12 + =
x x − 2 15
Thus, the sides of two squares are 20 m and
12 m.  ( x − 2) + x 8
Þ =
x ( x − 2) 15
104. Let the speed of the aircraft originally be ‘x’
km/h. 2x − 2 8

Then, time of journey for a distance of 600 km Þ =
x2 − 2x 15
600  Distance  Þ 8x2 – 16x = 30x – 30
is hours. ∵Time = 
x  Speed  Þ 8x – 46x + 30
2
= 0

and time of journey for a distance of 600 km Þ 4x2 – 23x + 15 = 0
600 Þ 4x – 20x – 3x + 15
2
= 0
with speed (x – 200) km/h is hours.
x − 200 Þ 4x(x – 5) – 3(x – 5) = 0
As per the question, Þ (4x – 3) (x – 5) = 0
600 600 1 3
− = x ¹ (not possible)
x − 200 x 2 4
 1  \ x= 5
∵30 minutes = 2 hours
Hence, the smaller tap will fill the tank in
5 hours.
 x − ( x − 200 )  1
⇒ 600   = And the larger tap will fill the tank in (5 – 2)
 x ( x − 200 )  2 = 3 hours.

30 Mathematics Class X

Click here to buy other Educart books on Amazon - https://amzn.to/40U8Txx


 ANSWER SHEET

SELF PRACTICE
4. (b) (2n – 1)a 6
⇒ S6 = [2(–5)+ (6 – 1)(2)]
Explanation: Here, first term = a 2
and common difference, d= 2a ⇒ S6 = 3[–10 + 5(2)]
So, an = a + (n – 1) d = 3[–10 + 10]
= a + (n – 1)2a = 3(0) = 0

= 2an – a ⇒ S6 = 0

= a (2n – 1) 17. (d) 2


Explanation: The terms (2p + 1), 10 and
7. (b) 137
(5p + 5) are consecutive terms of A.P., when
Explanation: It is given that
2 × 10 = (2p + 1) + (5p + 5)
a1= –3 and a2 = 4 ⇒ 20 = 7p + 6
We know that ⇒ 7p = 14
an = a + (n – 1)d ⇒ p =2
⇒ a1 = a + (1 – 1)d = a 21. (d) 8
⇒ a2 = a + (2 – 1)d = a + d Explanation:
⇒ a1 = –3 Here, d = –2, n = 5 and an = 0
and a2 = a + d = 4 We know, an = a + (n – 1)d
⇒ –3 + d = 4
Þ 0 = a + (5 – 1)(–2)
⇒ d =4 + 3 = 7
Þ 0 = a + (– 8)
∴ a21 = a + (21 – 1)d = –3 + (20)7
Þ a =8


= –3 + 140 = 137 22. (c) 15
⇒ a21 = 137 Explanation:

9. (c) 25 Here, common difference, d = 3


Now, a20 – a15 = (a + 19d) – (a + 14d)
Explanation: It is given that common
difference, d = 5. = 19d – 14d

We know, an = a + (n – 1)d = 5d = 5 × 3 = 15

\ a18 = a + (18 – 1)d = a + 17d 26. (c) 38


and, a13 = a + (13 – 1)d = a + 12d Explanation: It is given that

Now, a18 – a13 = (a + 17d) – (a + 12d) first term, a= 1
= a + 17d – a – 12d n term, an = 20
th


= 5d = 5 × 5 = 25 [As d = 5]
Sum of n terms, Sn = 399
⇒ a18 – a13 = 25
We know that,
an = a + (n – 1)d
13. (a) 0
⇒ 20 = 1 + (n – 1)d
Explanation: It is given that the first term,
a = –5 and common difference, d = 2. ⇒ (n – 1)d = 19 ...(i)
We know that the sum of n terms of an A.P. is
We also know that,
n n
Sn = [2a + (n – 1)d] Sn = [2a + (n – 1)d]
2 2

Arithmetic Progressions 31

Click here to buy other Educart books on Amazon - https://amzn.to/40U8Txx


n (C) The maximum distance Arzoo travelled
⇒ 399 = [2(1) + (n – 1)d]
2 carrying a flag = distance travelled by
Arzoo during placing the 14th flag in her left
⇒ 798 = n [2 + (n – 1)d]
position or 27th flag in her right position.
⇒ 798 = 2n + n(n – 1)d ...(ii)
= (3 + 3 + 3 + . . . . . + 13 times)
Using equations (i) and (ii), we get
= 3 × 13 = 39 m
798 = 2n + 19n
Hence, the maximum distance she travelled
⇒ 798 = 21n carrying a flag is 39 m.
798
⇒ n= = 38
21 41. (C) Let the spacecraft passed Braille after
N months. Then number of months,

i.e., n = 38
N = n + 15.
32. 19 The average monthly velocity when it
Explanation: Let n terms of the given A.P. passed Borelly is given by,
make the sum zero. Then, ⇒ aN = 1 + (N – 1)
n = (n + 15 – 1) d
[2a + ( n − 1)d ] = 0
2 = a + (n – 1)d + 15d
For the given A.P., we have ⇒ aN = an + 15d ...(i)
a = 18 = 80
and, d = 16 – 18 = –2 It is given that the velocity when it passed
n Braille, an = 55800 km/hr.
So, [18 × 2 + (n – 1)(–2)] = 0
2 ⇒ aN = 55800 + 15 × 3555
⇒ 36 – 2 (n – 1) = 0 = 1091125 km/hr.
⇒ 36 − 2n + 2 = 0
OR
⇒ 2n = 38
Let the spacecraft passed Braille after n
⇒ n = 19 months.
33. 5 It is given that an = 55800
Explanation: ⇒ a + (n – 1)d = 55800
We know, 27360 + (n – 1) 3555 = 55800
an = Sn – Sn – 1 3555n = 31995
\ a2 = S2 – S1
31995
= (22 + 2 × 2) – (12 + 2 × 1) n=
3555
= (4 + 4) – (1 + 2)
=9
= 5
Thus, after 9 months spacecraft pass
40. (A) All the flags are kept at the position of the Braille.
middle most flag or 14th flag. There are 13
flags on either side of the middle most flag 42. (A) (b) 52 minutes
at a distance of 3 m from each other. The Explanation: The jogging minutes are 10
first flag is at a distance of 3 m, the second minutes for the first week, 16 minutes for
at a distance of 6 m, the third at a distance the second week, 22 minutes for the third
of 9 m and so on, and this is on both sides week and so on.
of the middle most flag. As the jogging minutes increases
As Arzoo has to come back to the position uniformly by 6 minutes per week, these
of the middle most flag each time, the are in A.P. with a = 10 and d = 6.
distance travelled for placing the first To find his jogging time after 8 weeks, we
flag and coming back is 3 + 3 or 6 m, the
will use the formula for finding the general
distance travelled for placing the second
term of an A.P.,
flag and back is 6 + 6 or 12 m and so on.
an = a + (n – 1)d for n = 8.
The distances covered by Arzoo in placing
Therefore, a8 = 10 + 7 × 6
flags on one side of the middle most flag
are 6 m, 12 m, 18 m, … = 52
This forms an A.P. with a = 6 m and Therefore, jogging time after 8 weeks will
d = 6 m. be 52 minutes.

32 Mathematics Class X

Click here to buy other Educart books on Amazon - https://amzn.to/40U8Txx



(C) (d) 2n2 + 7 45. (B) (c) 11
Explanation: In an A.P., the difference Explanation: Let his goal time be
between two consecutive terms should be represented by an
constant. So, an = 31
Let us find out the terms of the given ⇒ a + (n – 1)d = 31
expressions by putting n = 1, 2, 3, … to
⇒ 51 + (n – 1) (–2) = 31 [Using part (A)]
check if they form an A.P. or not.
⇒ 51 – 2n + 2 = 31
(a) By putting n = 1, 2, 3,… in 5 + n, we get
the terms as 6, 7, 8, … which forms an ⇒ 2n = 53 – 31 = 22
A.P. ⇒ n = 11
(b) Here, we get the terms as 5, 7, 9, … Hence, it took him minimum 11 days to
which forms an A.P. achieve his goal.
(c) Here, we get the terms as 1, 6, 11, … (C) (b) 30
which forms an A.P. Explanation: To find the incorrect term, we
(d) Here, we get the terms as 9, 15, 25, … have to put the values of all the four option
which do not form an AP as difference in the formula an = a + (n – 1)d to find the
between any two consecutive terms is value of n.
not same. (a) an = 41
43. (A) Since each row is increasing by 10 seats, ⇒ a + (n – 1)d = 41
so it is an AP with first term a = 30, and ⇒ 51 + (n – 1)(–2) = 41
common difference d = 10.
⇒ 51 – 2n + 2 = 41
So number of seats in 10th row
⇒ 53 – 2n = 41
a10 = a + 9d
⇒ 2n = 53 – 41 = 12
= 30 + 9 × 10 = 120
⇒ n=6
44. (C) (d) ` 48 Since, n is an integer value, so 41 is a term
Explanation: To find the money saved in of the A.P. formed in part (A).
the 13th month, we will put n = 13, a = 12
(b) an = 30
and d = 3 in the formula
⇒ 53 – 2n = 30
an = a + (n – 1)d.
⇒ 2n = 23
Therefore, a13 = 12 + (13 – 1)3
= 12 + 36 = 48 ⇒ n = 10.5
Kaashvi saved ` 48 in the 13th month. Here, the value of n is not an integer value.
(E) (b) 10 : 3 So, 30 cannot be a term of the A.P. formed
in part (A).
Explanation: It is given that
a7 12 49. Given, for an A.P., a = 15, d = – 3 and an = 0
=
a3 5 We know,
a + 6d 12 an = a + (n – 1)d
Þ =
a + 2d 5 Þ 0 = 15 + (n – 1) × (– 3)

Þ ⇒ 5( a + 6d ) = 12( a + 2d ) Þ n =6
Hence, the value of n is 6.
Solving further, we get
6 52. Two digits numbers divisible by 3 are 12, 15,
–7a = –6d or a = d 18 ..., 99.
7

The above series forms an A.P. with first term,
a13 a + 12d
Now, = a = 12
a4 a + 3d
Common difference, d = 15 – 12 = 18 – 15 = 3
6 and last term, an = 99.
d + 12d
6d + 84d 90d 10
= 7 = = =
Now, n term, th
an = a + (n – 1)d,
6 6d + 21d 27d 3
d + 3d
where, ‘n’ is the number of the terms
7
Therefore, the ratio of the 13th to 4th terms Þ 99 = 12 + (n – 1) 3
is 10 : 3.

Arithmetic Progressions 33

Click here to buy other Educart books on Amazon - https://amzn.to/40U8Txx


Þ 99 = 12 + 3n – 3 Difference of their 10th terms

Þ 3n = 90 = (7 + 9d) – (2 + 9d)
Þ n = 30 = 7 + 9d – 2 – 9d

Hence, the number of 2 – digit numbers which =5
are divisible by 3, is 30. Similarly, 21 term of first A.P. = 2 + 20d
st

54. Given, A.P. is –1, 4, 9, 14, ...... and 21st term of second A.P. = 7 + 20d
Here, first term, a = –1
Difference of their 21st terms
Common difference, d = 4 – (–1) = 5
= (7 + 20d) – (2 + 20d)
Now, nth term, an = 129
= 7 + 20d – 2 – 20d = 5
∴ a + (n – 1)d = 129

Thus, we can say that if an and bn are nth terms
⇒ – 1 + (n – 1)5 = 129 of first and second A.P. respectively, then
⇒ – 1 + 5n – 5 = 129
bn – an = [7 + (n – 1)d] – [2 + (n – 1)d]
⇒ 5n – 6 = 129
= 7 + (n – 1)d – 2 – (n – 1)d = 5
⇒ 5n = 135
⇒ bn – an = 5
⇒ n = 27
Hence, the value of n is 27. Hence, the difference between any two
corresponding terms of such A.P.’s is the same
59. Here, Sp = ap2 + bp as the difference between two corresponding

\ S1 = a(1)2 + b (1) = a + b first terms.

Þ S2 = a(2)2 + b(2) = 4a + 2b
69. Given nth term of an A.P. an = 5 – 2n
Now, first term, A = S1 = a + b
And, S2 = A1 + A2 = A + (A + D) First term of A.P. a1 = 5 – 2 × 1 = 3
= 4a + 2b Second term of A.P. a2 = 5 – 2 × 2 = 1
 (Where, D is the common difference)
Third term of A.P. a3 = 5 – 2 × 3 = –1

Þa + b + a + b + D = 4a + 2b

Þ D = 4a + 2b – 2a – 2b Common difference of A.P., d = a2 – a1 = a3 – a1

Þ D = 2a
=1–3=–1–1=–2
Hence, the common difference is 2a.
Sum of first 20 terms:
64. Given: a17 = a10 + 7 ...(i)
Let ‘a’ be the first term of A.P. and ‘d’ be its S20 = [2 × 3 + (20 – 1(– 2)]
common difference. = 10 [6 – 38]
Then, a10 = a + (10 – 1)d = a + 9d
= 10 × – 32
and a17 = a + (17 – 1)d = a + 16d
Put the values in (i), we get = – 320
a + 16d = a + 9d + 7 Sum of first 20 terms of AP is – 320.
⇒ 7d = 7
⇒ d=1 70. Let the first term of the A.P. be ‘a’ and its
common difference be ‘d’.
Hence, the common difference of the given A.P.
is 1. Given, 7a7 = 11a11

66. Let d be the common difference of two A.P.’s Then, 7(a + 6d) = 11(a + 10d)

First term of first A.P. is 2. [Q an + a + (n – 1)d]

So, first A.P. will be 2, 2 + d, 2 + 2d ...


Þ 7a + 42d = 11a + 110d

Þ 7a – 11a = 110d – 42d
Also, first term of second A.P. is 7

Þ – 4a = 68d
So, second A.P. will be 7, 7 + d, 7 + 2d ...
We know that, an = a + (n – 1)d
Þ a = – 17d ...(i)


\ 10th term of first A.P. = 2 + 9d Now, 18th term of A.P.

and 10th term of second A.P. = 7 + 9d a18 = a + (18 – 1)d

34 Mathematics Class X

Click here to buy other Educart books on Amazon - https://amzn.to/40U8Txx


= a + 17d and n = 15.
= –17d + 17d [Using (i)] n
We know, Sn = (2a + (n – 1)d)
2

Þ a18 = 0
15
Hence, the 18th term of the A.P. is 0. \ S15= [2 × 8 + (15 – 1) × 8]
2
73. Given, third term of A.P., a3 = 5 15
= (16 + 14 × 8)
Seventh term of A.P., a7 = 9 2
Let the first term of A.P. be ‘a’ and its common =
15
[16 + 112]
difference be ‘d’. 2
Now, a3 = a + (3 – 1)d 15
= × 128 = 960
or 5 = a + 2d ...(i) 2
and a7 = a + (7 – 1)d Hence, the sum of first 15 multiples of 8 is 960.
or 9 = a + 6d ...(ii) 84. In the given series, we have
On solving equations (i) and (ii), we get 5 – 1 = 4, 9 – 5 = 4, 13 – 9 = 4 and so on i.e. the
4d = 4 difference between any two consecutive terms
⇒ d =1 is same.
Put the value of ‘d’ in equation (i), we get So, this series is an A.P. with a = 1, d = 4,
5 =a+2×1 an = x and Sn = 1326.
⇒ a =3 We know,
Hence, the A.P. is 3, 4, 5, 6. an = a + (n – 1)d
⇒ x = 1 + (n – 1) (4)
1 2 3
76. Given A.P. is 17, 16 , 15 ,14 ,...... ⇒ x = 4n – 3
5 5 5
81 77 73 x +3
or, 17, , , ,..... or n=
5 5 5 4
Here, a = 17 n
Further, Sn = [a + an]
81 4 2
and, d= − 17 = −
5 5
x +3
Let nth term of the A.P. be its first negative term.
Þ 1326 = [1 + x ]
8
So, an < 0 ⇒ (1 + x) (x + 3) – 10608 = 0

⇒ a + (n – 1)d < 0 ⇒ x2 + 4x – 10605 = 0
 4 ⇒ x + 105x – 101x – 10605 = 0
2

⇒ 17 + (n – 1) ×  −  < 0
 5
x(x + 105) – 101(x + 105) = 0

⇒ 85 + (n – 1) (–4) < 0 ⇒ (x + 105) (x – 101) = 0

⇒ 89 – 4n < 0 ⇒ x – 101 = 0

⇒ –4n < –89 ( x + 105 ≠ 0, as this A.P. is an
increasing series, so x cannot be
or, 4n > 89
negative)
89

⇒ n> ⇒ x = 101
4
Thus, the value of x is 101.
Since, n is a natural number, and natural number
89 86. (A) Yes, (2n – 3) is the nth term of an A.P.
just greater than is 23.
4 It is given that
Hence, 23rd term is the first negative term of the an = 2n – 3
given A.P.
Put n = 1, a1 = 2(1) – 3 = 2 – 3 = –1
81. Multiples of 8 are 8, 16, 24, ............
Put n = 2, a2 = 2(2) – 3 = 4 – 3 = 1

Since, in the above series, the difference
Put n = 3, a3 = 2(3) – 3 = 6 – 3 = 3
between any two consecutive numbers is

Put n = 4, a4 = 2(4) – 3 = 8 – 3 = 5
constant, so it forms an A.P. with a = 8, d = 8

Arithmetic Progressions 35

Click here to buy other Educart books on Amazon - https://amzn.to/40U8Txx


List of numbers becomes –1, 1, 3, 5...
So, the required A.P. will be:
Here, a2 – a1 = 1 – (–1) = 2 a, a + d, a + 2d, a + 3d ...
a 3 – a2 = 3 – 1 = 2
i.e., 3, 3 + 4, 3 + 2(4), 3 + 3(4) ...
a 4 – a3 = 5 – 3 = 2
i.e., 3, 7, 11, 15 ...
Clearly, a2 – a1 = a3 – a2 = a4 – a3
92. Let (a – d), a and (a + d) be the three parts of
Hence, 2n – 3 is the nth term of an A.P. 207 such that these are in A.P.
(B) No, (3n2 + 5) is not the nth term of an A.P.

It is given that sum of these numbers is 207.
It is given that an = 3n2 + 5
⇒(a – d) + a + (a + d) = 207
Put n = 1, a1 = 3(1)2 + 5 = 3 + 5 = 8
⇒ 3a = 207
Put n = 2, a2 = 3(2)2 + 5 = 12 + 5 = 17
⇒ a = 69
Put n = 3, a3 = 3(3)2 + 5 = 27 + 5 = 32
List of numbers becomes 8, 17, 32 ... It is also given that product of two smaller
parts is 4623.
Here, a2 – a1 = 17 – 8 = 9,
⇒ a(a – d) = 4623
and, a3 – a2 = 32 – 17 = 15
⇒ 69(69 – d) = 4623
Clearly, a2 – a1 ≠ a3 – a2
Hence, (3n2 + 5) is not the nth term of an A.P. 4623
⇒ 69 – d =
69
(C) No, (1 + n + n2) is not the nth term of an A.P.
It is given that an = 1 + n + n2 ⇒ 69 – d = 67
Put n = 1, a1 = 1 + (1) + (1)2 ⇒ d = 69 – 67 = 2
= 1 + 1 + 1 = 3 So, first part = a – d = 69 – 2 = 67,
Put n = 2, a2 = 1 + (2) + (2)2 second part = a = 69
= 1 + 2 + 4 = 7 and third part = a + d = 69 + 2 = 71
Put n = 3, a3 = 1 + (3) + (3)2
Hence, the three parts are 67, 69 and 71.
= 1 + 3 + 9 = 13 −4 −2 1
94. The given A.P. is −1 , .......4
List of numbers becomes 3, 7, 13 ... 3 3 3
Here, a2 – a1 = 7 – 3 = 4 −4

Here, first term, a =
and, a3 – a2 = 13 – 7 = 6 3
Clearly, a2 – a1 ≠ a3 – a2
 −4 
Hence, (1 + n + n2) is not the nth term of an Common difference, d = −1 − 

 3 
A.P.
4 1
88. Let the first term of an AP be a and common = −1 + =
3 3
difference be d.
1 13

It is given that a5 = 19 and a13 – a8 = 20.
Last term, l = 4 =
3 3

We know that, an = a + (n – 1) d

We know that, an = a + (n – 1)d

\ a5 = a + (5 – 1) d
= a + 4d = 19  ...(i)
If an is the last term, then
l = a + (n – 1)d

Also, a13 – a8 = 20
13 4  1
⇒ (a + 12d) – (a + 7d) = 20
⇒ = − + ( n − 1)  
3 3  3
⇒ a + 12d – a – 7d = 20
⇒ 13 = – 4 + (n – 1)
⇒ 5d = 20
⇒ (n – 1) = 17

⇒ d = 4
⇒ n = 18

Putting d = 4 in equation (i), we get
n th
a + 4d = 19 So, the two middle most terms are   and
 2
th
⇒ a + 4(4) = 19  n  as total numbers of terms are even.
 + 1
⇒ a + 16 = 19 2 th
 18 
⇒ a = 3
The two middle most terms are   and
 2

36 Mathematics Class X

Click here to buy other Educart books on Amazon - https://amzn.to/40U8Txx


th ⇒ 2a + 15d = 32 ...(ii)
 18 
 + 1 i.e., 9th and 10th terms. Subtracting equation (i) from equation (ii), we
2 
get
Now, a9 = a + (9 – 1)d
(2a + 15d) – (2a + 5d) = 32 – 12
= a + 8d
⇒ 2a + 15d – 2a – 5d = 20
4  1  ( −4 + 8) 4 ⇒ 10d = 20
= − + 8   =
=
3 3 3 3 ⇒ d=2
Putting the value of d in equation (i), we get
and, a10 = a + (10 – 1)d
2a + 5(2) = 12
= a + 9d
⇒ 2a + 10 = 12
4  1  ( −4 + 9) 5 ⇒ 2a = 2
= − + 9   =
=
3 3 3 3 ⇒  a=1
10
So, the sum of the two middle most terms Now, S10 =
[2a + (10 – 1)d]
2
= a9 + a10
= 5 [2(1) + 9(2)]
4 5 9 = 5 [2 + 18]

= + = =3
3 3 3
= 5 × 20
97. Let a be the first term, d be the common S10 = 100
difference, l be the last term and n be the
Hence, the required sum of the first 10 terms is
number of terms of the given A.P.
100.

So, a = –5, l = 45 and Sn = 120  [Given]
101. Let, the first term of the A.P. be ‘a’ and its
We know that, if the last term of an A.P. is
common difference be ‘d’.
known, then the sum of n terms of an A.P. is
n Now, am = a + (m – 1)d
Sn = (a + l)
2 1
⇒ = a + (m – 1)d ...(i) (given)
n n
⇒ 120 = ( −5 + 45)
2 and an = a + (n – 1)d
⇒ 120 × 2 = 40 × n 1
⇒ = a + (n – 1)d ...(ii) (given)
⇒ n=6 m
Also, l = a + (n – 1)d On subtracting equation (ii) from equation (i),
we get:
⇒ 45 = –5 + (6 – 1)d
1 1
⇒ 50 = 5d − = [(m – 1) – (n – 1)]d
n m

⇒ d = 10

Hence, number of terms = 6 m − n
⇒ = (m – n)d

and common difference = 10 mn
1
99. Let a be the first term and d be the common ⇒ d=  ...(iii)
mn
difference of the A.P.
Put the value of d in equation (i), we get
We know that sum of n terms of an A.P. is given
by, 1 1
= a + (m – 1) ×
n n mn
Sn = [2a + (n – 1)d]
2 1  m − 1
⇒ a= − 
Now, S6 = 36  (Given) n  mn 
6 m − m +1 1
⇒ [2a + (6 – 1)d] = 36
2 = =  ...(iv)
mn mn
⇒ 3 [2a + (6 – 1)d = 36
Now, sum of first mn terms is given by
⇒ 2a + 5d = 12 ...(i)
mn  1 1 
Also, S16 = 256 [Given] Smn = 2 × + ( mn − 1) × 
2  mn mn 
16
⇒ [2a + (16 – 1)d] = 256  n 
2  ∵Sn = [2a + ( n − 1)d ]
⇒ 8[2a + 15d] = 256  2 

Arithmetic Progressions 37

Click here to buy other Educart books on Amazon - https://amzn.to/40U8Txx


1[ mn + 1 n
Þ Smn = 2 + mn − 1] = = [2 × 1 + (n – 1) × 2]
2 2 2

( mn +1) n
Hence, the sum of first mn terms is . = [2 + 2n – 2]
2 2
104. The numbers which when divided by 6 gives 1 = n2
as remainder are : Hence, the sum of the first ‘n’ terms of A.P. is n2.
7, 13, 19, 25, 31, 37, ...
109. Let a be the first term and d be the common
Clearly, the above series forms an A.P., as their difference of the A.P.
common difference is the same, i.e. d = 6.
Now, a14 = 2a8 [Given]
Here, first term, a = 7, common difference, d = 6
and n = 40. Þ a + 13d = 2(a + 7d)
We know,
Þ a + 13d = 2a + 14d
n Þ –a = d
Sn = [2a + ( n − 1) d ]
2
or a = –d ...(i)
40 Also, a6 = –8
S40 = [2 × 7 + (40 – 1) × 6]
2

Þ a + 5d = –8
= 20[14 + 39 × 6]

Þ –d + 5d = –8 [Using (i)
= 20 × 248
= 4,960
Þ 4d = –8
Hence, the required sum is 4,960.
Þ d = –2

\ a = –d = 2
106. Let the first term of the A.P be ‘a’ and its
common difference be ‘d’. Now, sum of first 20 terms is given by

Given: S7 = 49 20
2a + (20 − 1)d 
S20 =

and S17 = 289 2  
7 = 10[2 × 2 + 19 × (–2)]

Then, S7 = [2a + (7 – 1) × d]
2 = 10[4 – 38]
 n  = – 340
∵Sn = [2a + ( n − 1)d ]
 2  Hence, the required sum is (–340).
7 112. Since, child puts ` 5 on 1st day, ` 10 (2 × 5) on
Þ 49 = [2a + 6d]
2 2nd day, ` 15(3 × 5) on 3rd day and so on.
Þ 7 = a + 3d  ...(i) Total savings = 190 coins = 190 × 5
17

and S17 = [2a + (17 – 1) × d] = ` 950
2
So, the series of her daily savings is,
17
Þ 289 = × [2a + 16 × d] ` 5, ` 10, ` 15, .....
2
Þ 17 = a + 8d  ...(ii) Clearly, this series is an A.P.
On subtracting equation (i) from equation (ii), So, first term, a = 5
we get Common difference, d = 5
a + 8d = 17 Sum of total savings, Sn = 950
a + 3d = 7 Let, n be the last day when piggy bank becomes
– – – full.
5d = 10

\ Sn = n [2a + (n – 1)d]
Þ d=2 2
Put the value of ‘d’ in equation (i), we get n
a + 3 × (2) = 7 Þ 950 = [2 × 5 + ( n − 1) × 5]
2
Þ a=7–6 =1
Þ 1900 = n[10 + 5n – 5]
\ Sum of the first ‘n’ terms is
Þ 1900 = n[5n + 5]
n
Sn = [2a + (n – 1)d]
Þ5n2 + 5n – 1900 = 0
2

38 Mathematics Class X

Click here to buy other Educart books on Amazon - https://amzn.to/40U8Txx



Þ n2 + n – 380 = 0 or x2 + 3x – 1720 = 0

Þ n + 20n – 19n – 380 = 0
2
⇒ x + 43x – 40x – 1720 = 0
2

(on splitting the middle term) ⇒ x(x + 43) – 40(x + 43) = 0



Þ n(n + 20) – 19(n + 20) = 0 ⇒ (x + 43) (x – 40) = 0

Þ (n – 19) (n + 20) = 0 ⇒ x – 40 = 0

Þ n = 19 or – 20 ( x + 43 ≠ 0, as this A.P. is
But ‘n‘ cannot be negative, hence n = 19. an increasing series, so x
cannot be negative.)
Hence, she continuous the savings for 19 days
and saves ` 950. ⇒ x = 40
Views on habit of saving: Thus, x = 40
(1) Child is developing a very good habit of 116. (A) Multiples of 2 and 5 will be multiples of
savings. LCM of 2 and 5.
(2) Consistent saving can creates a wonder. LCM of (2, 5) = 10
113. Let ‘a’ be the first term and ‘d’ be the common \Multiples of 2 as well as 5 between 1 and
difference of the A.P. 500 are 10, 20, 30, 40, ...490.
Also, let a – 3d, a – d, a + d, a + 3d be the four This series forms an A.P. with first term,
consecutive terms of the A.P. a = 10
As per the question,
Common difference, d = 20 – 10 = 10
(a – 3d) + (a – d) + (a + d) + (a + 3d) = 32
Last term, l = 490
⇒ 4a = 32, or a = 8 ...(i)
(a − 3d )(a + 3d ) We know that sum of n terms of an A.P. is
7
and = given by,
(a − d )(a + d ) 15 n
Sn = [a + l] ...(i)
a2 − 9d 2 7 2
⇒ =
a2 − d 2 15 Also, l = a + (n – 1) d
⇒ 15a2 – 135d2 = 7a2 – 7d2 ⇒ 490 = 10 + (n – 1)10
⇒ 8a2 = 128 d2 ⇒ 480 = (n – 1)10
Using (i), we have: ⇒ (n – 1) = 48
8 × 82 = 128 d2
⇒ n = 49
⇒ d2 = 4, or d = ± 2
Putting this value in equation (i), we get
Thus, the four numbers are 2, 6, 10, 14 or
14, 10, 6, 2. 49
S49 = [10 + 490]
2
114. For the given series, we have 49
4 – 1 = 3; 7 – 4 = 3; 10 – 7 = 3 and so on. = × 500
2
i.e. the difference between any two consecutive ⇒ S49 = 12250
terms is same. Hence, the required sum is 12250.
So, the given series is an A.P. with a = 1, d = 3
(B) Here, multiples of 2 as well as 5 from 1 to
and l = x.
500 are 10, 20, 30, ...500.
Let the A.P. contains ‘n’ terms.
Clearly, this series is an A.P.
Then, an = x
Here, first term, a = 10
⇒ a + (n – 1)d = x
⇒ 1 + (n – 1)3 = x Common difference, d = 20 – 10 = 10
⇒ 1 + 3n – 3 = x Last term, l = 500
⇒ 3n – 2 = x We know that,
x + 2 an = a + (n – 1)d
⇒ n=
3 Þ l = a + (n – 1)d
n  [where, n is total number of terms]
Now, Sn = [first term + last term]
2 Þ 500 = 10 + (n – 1)d
x + 2 Þ 490 = (n – 1)10
⇒ 287 = [1 + x ]
3×2 ⇒ (n – 1) = 49
⇒ (x + 1) (x + 2) = 1722 ⇒ n = 50

Arithmetic Progressions 39

Click here to buy other Educart books on Amazon - https://amzn.to/40U8Txx


Also, we know that, and, last term , l’’ = 500
n Let n3 be the number of terms in this A.P.
Sn =
(a + l)
2 Then, l’’ = a’’ + (n3 – 1) d’’
50 500 = 10 + (n3 – 1)(10)
⇒ S50 = (10 + 500) ⇒
2 ⇒ 490 = (n3 – 1)10
= 25 × 510
⇒ n3 – 1 = 49
= 12750
⇒ n3 = 50
Hence, the required sum is 12750.
(C) Multiples of 2 or 5 = Multiples of 2 + Sum of this series,
multiples of 5 – [Multiples of 2 and 5] ...(i) n3
Sn = (a” + l”)
Multiples of 2 are [2, 4, 6, ...500]
3
2
Multiples of 5 are 5, 10, 15, ...500. 50
= (10 + 500 )
Multiples 2 and of 5 are 10, 20, 30, ...500. 2
Now, for the series 2, 4, 6, ....500, we get first = 25× 510 =12750
term, a = 2 Now, sum of multiples of 2 or 5
common difference, d = 2 = Sn + Sn – Sn
1 2 3

and, last term, l = 500 = 62750 + 25250 – 12750


Let number of terms in this A.P. be n1. = 88000 – 12750 = 75250
Then, last term, l = a + (n1 – 1)d
118. Consider an A.P., whose first term be ‘a’ and
500 = 2 + (n1 – 1)(2) common difference be ‘d’.
498 = (n1 – 1)2 Given, Sp = q
⇒ (n1 – 1) = 249 Sq = p
⇒ n1 = 250 p
Now, Sp = [2a + (p – 1)d] = q
Sum of this series, 2
n 2q
Sn = 1 [a + l]
2 or 2a + (p – 1)d = ...(i)
1
p
250
= [2 + 500] q
2 and Sq = [2a + (q – 1)d] = p
2
Sn₂ = 125 × 502 = 62750
Similarly, for the series 5, 10, 15, ...500, we 2p
or 2a + (q – 1)d = ...(ii)
have q
First term, a’ = 5, On subtracting equation (ii) from equation (i),
Common difference, d’ = 5 we get
and, last term, l’ = 500 2q 2p
[(p – 1) – (q – 1)]d = −
Let n2 be the number of terms of second list. p q
We know that, an = a + (n – 1)d 2 2
2q − 2p 1
⇒ l’ = a’ + (n2 – 1) d’ ⇒ d= ×
pq p−q
⇒ 500 = 5 + (n2 – 1)5
⇒ 495 = (n2 – 1)5 −2 ( p + q)
⇒ d=  ...(iii)
pq
⇒ n2 – 1 = 99
⇒ n2 = 100 Substituting the value of d in equation (ii), we
get
Sum of this series
n  −2( p + q )  2p
Sn = 2 ( a ' + l ') 2a + (q – 1) ×   =
2  pq  q
2

 2p 2 (q − 1)( p + q )  1
100 a=  +
⇒ ×2
Sn = (5 + 500 ) q pq 

2 2
= 50 × 505 p2 + pq − p + q2 − q
a=
= 25250 pq
 ...(iv)
And, for the series 10, 20, .... 500, we have
Now, sum of first (p + q) terms, sp + q
first term, a’’ = 10
p+q
common difference, d’’ = 10 = 2a + ( p + q − 1) d 
2 

40 Mathematics Class X

Click here to buy other Educart books on Amazon - https://amzn.to/40U8Txx


p+q  p2 + q2 − p + pq − q  = 6[300 + 550]
= 2 ×  +
2   pq  = 6 × 850 = ` 5100

Then, Ramkali would be able to save ` 5,100 in
 −2 ( p + q )   12 months and she needs ` 5,000 to send her

( p + q − 1) ×   
 pq  daughter to school.
 [From (iii) & (iv)]
Hence, Ramkali would be able to send her
 p2 + q2 − p + pq − q − p2 − q2  daughter to school.
 − 2pq + p + q 
Values : Putting efforts to send her daughter
= p + q
 pq  to school shows her awareness regarding girls
education and educating a child.
(− pq)
= ( p + q) × = – (p + q)
pq 122. Let the first term of A.P. be ‘a’ and its common
difference be ‘d‘.
Þ Sp + q = –(p + q)
Given, a4 + a8 = 24
Hence, proved
⇒ a + 3d + a + 7d = 24  [an = a + (n – 1)d]
119. The first term of the A.P., a = – 7 ⇒ 2a + 10d = 24
Common difference,
or a + 5d = 12 ...(i)
d = (– 12) – (– 7) = (–12 + 7)
= – 5 and a6 + a10 = 44  (given)
Let the nth term of the given A.P. be – 82. ⇒ a + 5d + a + 9d = 44
Then, an = – 82 ⇒ 2a + 14d = 44
Þ a + (n – 1) d = – 82 or a + 7d = 22 ...(ii)
Þ –7 + (n – 1) (– 5) = – 82 On subtracting equation (i) from equation (ii),
Þ – 7 – 5n + 5 = – 82 we get,
Þ – 5n = – 80 a + 7d = 22
Þ n = 16 a + 5d = 12
Let, mth term of the given A.P. be – 100. – – –
\ am = – 100 2d = 10
Þ a + (m – 1) d = – 100 ⇒ d=5
Þ – 7 + (m – 1) (– 5) = – 100 If we put the value of ‘d’ in equation (i), we get
Þ (m – 1) 5 = 93 a = 12 – 25 = – 13
93 98 Then, first term of A.P., a = –13
Þ m= +1 = Common difference of A.P., d = 5
5 5
Now, sum of first ‘n’ terms is given by,
Since, the value of m is not a natural numbers,
n
Therefore, – 100 is not any term of the given Sn = 2a + (n − 1) d 
A.P. 2
Hence, – 82 is the 16th term of the A.P. and 10 
– 100 is not a term of the given A.P. ∴ S10 =
2 
( )
2 × − 13 + (10 − 1) × 5

121. The saving in first month is ` 150. = 5(–26 + 45)


The saving in second month is ` (150 + 50) = 5 × 19
= ` 200 = 95

Similarly, saving goes on increasing every Hence, the sum of the first 10 terms of the A.P.
month by ` 50. is 95.
Savings = ` 150, ` 200, ` 250, ` 300,..... 124. Let, ‘a’ be the first term of A.P. and ‘d’ be its

Savings forms an A.P. in which first term (a) = 150 common difference.

and common difference, (d) = 50 Let, Sm and Sn be the sum of the first ‘m’ and
first ‘n’ of terms of the A.P., respectively.

Then, total savings for 12 months
m
n Then, Sm = 2a + ( m − 1)d 
S12 = [2a + (n – 1)d] 2
2
12 n
= [2 × 150 + (12 – 1)50] and Sn = 2a + ( n − 1)d 
2 2

Arithmetic Progressions 41

Click here to buy other Educart books on Amazon - https://amzn.to/40U8Txx


Sm m2
The given equation is:
But, Sn = 2 (given) (–4) + (–1) + 2... + x = 437
n
⇒ Sn = 437
m ( x + 7) ( x − 4)
2a + ( m − 1)d  m2 ⇒ = 437
2
∴ = 2 6
n n
2a + ( n − 1)d 
2 ⇒ x2 – 4x + 7x – 28 = 437 × 6
⇒ x2 + 3x – 28 = 2622
2a + (m − 1)d m ⇒ x + 3x – 2650 = 0
2
⇒ =
2a + (n − 1)d n
By quadratic formula
⇒ n[2a + (m – 1)d] = m[2a + (n – 1)d]
−3 ± (3)2 − 4(1)( −2650 )
⇒ 2an + mnd – nd = 2am + mnd – md x=
2(1)
⇒ md – nd = 2a(m – n)
⇒ d = 2a ...(i) −3 ± 10609
=
Now, the ratio of the mth and nth terms is: 2
−3 ± 103
a =
m a + (m − 1)d 2
=
a a + (n − 1)d −3 + 103 −3 − 103
n
Þ x= ,
a + (m − 1)2a 2 2
= 100 −106
a + (n − 1)2a = , = 50, − 53
2 2
a [1 + 2m − 2]
But x can not be negative. So x = 50.
= 
a[1 + 2n − 2] 127. It is given that,
2m − 1 money saved in 1st month = ` 32
=
2n − 1 money saved in 2nd month = ` 36
Hence, the ratio of mth to nth term is money saved in 3rd month = ` 40
(2m − 1) : (2n − 1). Let Yasmeen save ` 2000 in n months.
125. The given equation is: –4 + (–1) + 2 + ... + x = 437 Clearly, monthly savings of Yasmeen are in A.P.
with a = ` 32, d = ` 4 and Sn = ` 2000.
The given equation is A.P. with first term, a = –4 We know that sum of first n terms of an A.P. is
Common difference, n
Sn = [2a + (n – 1)d]
d = (–1) – (–4) = –1 + 4 = 3 2
and last term, l = x n
⇒ 2000 = [2 × 32 + (n –1)(4)]
Let this A.P. contain n terms. 2
an = a + (n – 1)d
= n[32 + 2(n – 1)]
⇒ l = x = a + (n – 1)d ⇒ 2000 = n[32 + 2n – 2]
⇒ x = –4 + (n – 1)(3) ⇒ 2000 = n[30 + 2n]
⇒ x = –4 + 3n – 3
⇒ 2000 = 2n[15 + n]
x +7
⇒ n= ⇒ 1000 = 15n + n2
3
⇒ n + 15n – 1000 = 0
2
Now, we know that
Splitting the middle term, we get
n
Sn = [2a + (n – 1)d] n2 + 40n – 25n – 1000 = 0
2
n(n + 40) – 25(n + 40) = 0
x + 7  ( )  x + 7  ( ) (n + 40)(n – 25) = 0
⇒ Sn = 2 −4 +  − 1 3 
 
2×3  3 ⇒ n = –40, n = 25
x +7 ( x + 7) ( x − 4) But n ≠ –40, as number of months cannot be

= [−8 + x + 4] =
6 6 negative.
x +7 ( x + 7) ( x − 4) ∴ n = 25
[−8 + x + 4]==
6 6 Hence, Yasmeen will save ` 2000 in 25 months.

42 Mathematics Class X

Click here to buy other Educart books on Amazon - https://amzn.to/40U8Txx


 ANSWER SHEET

SELF PRACTICE
1. (c) BC.DE = AB.EF
Explanation:

DAFD ~ DDGE (AA)


It is given that AF FD

\ = (CPST )
ΔABC ~ ΔEDF DG GE

AB BC AC 16 − x x
\ = = = (CPST )
ED DF EF x 8− x

Also, ΔABC is not to similar to ΔDEF 16


128 = 24x or x = cm
AB BC 3
\ ≠
DE EF [CBSE Marking Scheme Term-1 SQP 2021]
i.e., AB.EF ≠ BC.DE Explanation: In DAFD and ∆ABE
ÐAFD = ÐABE = 90°
6. (c) ÐB = ÐD ÐA = ÐA [Common angle]
\ By AA similarity criterion,
Explanation:
DAFD ~ DABE
AF FD
\ =
AB BE
AB − BF BF

Þ =
AB BE
[ BF = FD = Sides of square FDGB]
16 − BF BF
Þ =
16 8
Þ 128 – 8BF = 16BF
In ΔABC and ΔDEF,
Þ 24BF = 128
AB BC
= [Given] 128 16
DE FD Þ BF = =
24 3
Angle formed by sides AB and BC is ÐB. \ Length of side of square FDGB
Angle formed by sides DE and FD is ÐD. 16
= cm
So, by SAS similarity criterion, DABC ~ DEDF if, 3
ÐB = ÐD.
24
16 10. (d) cm
8. (b) cm 7
3
Explanation: Since, PA, QB and RC are each

∆ABE is a right triangle & FDGB is a square of perpendicular to AC,
side x cm \ PA | | QB | | RC
Now, in DACR, QB | | RC.

Triangles 43

Click here to buy other Educart books on Amazon - https://amzn.to/40U8Txx


\ By Thales theorem, ay
15. (c) x =
AB QB a+b
=
AC RC [CBSE Marking Scheme SQP Std. 2022]
AB y
Explanation: Given, AE = a, EC = b DE = x, BC = y.
Þ = ...(i)
AC z As, DE || BC
Similarly, in DCAP, QB | | PA.
ÐADE = ÐABC
BC QB
\ = ÐAED = ÐACB
AC PA
So, by AAA property,
BC
Þ =y DADE ~ DABC
AC
AC − AB DE AE
Þ =y =
AC BC AC

AB x a
Þ 1− =y = [\ AC = AE = EC]
AC y a+b

y
Þ 1− =y [From (i)] ay
z x=
a+b
1 1
Þ 1 = y x + z 
  23. (b) Both assertion (A) and reason (R) are true
1 1 but reason (R) is not the correct explanation of
Þ 1 = y 8 + 6 
  assertion (A).
 7  Explanation: Since,
Þ 1 = y  24 
  DFEC @ DGDB ...(i)
24 Þ EC = BD
Þ y= cm
7 It is given that,
[CBSE Marking Scheme SQP 2022] Ð1 = Ð2

14. (b) 1 cm Þ AE = AD ...(ii)


Explanation: Let the length of AE be x [ Sides opposite to equal angles are equal]
Since, DE || BC Dividing (ii) by (i), we have
\ ÐADE = ÐABC
And, ÐAED = ÐACB AE AD
=
A EC BD
x Þ DE || BC
2 cm
[By the converse of basic
D E proportionality theorem]
3 cm 1.5 cm Þ Ð1 = Ð3 and Ð2 = Ð4
[Corresponding angles]
B C Thus, in tringles ADE and ABC, we have
Then, by AA similarity criterion,
ÐA = ÐA [Common]
DADE ~ DABC
AD AE Ð2 = Ð4 (Proved above)
=
AB AC Þ DADE ~ DABC (By AA similarity)
2 x Hence, both assertion and reason are true but
= reason is not the correct explanation of the
5 x +1 .5
assertion.
2x + 3 = 5x
5x – 2x = 3 24. (A) (a) Similar by AA criterion
3x = 3 Explanation:
x =1 Since, AB | | DC
Hence, the length of AE is 1 cm. \ ÐOAB = ÐOCD ...(i)

44 Mathematics Class X

Click here to buy other Educart books on Amazon - https://amzn.to/40U8Txx


and ÐOBA = ÐODC ...(ii) OC OB
\ =
[Alternate angles] OD OA
Now, in DAOB and DCOD, 3 2 1
= =
ÐOAB = ÐOCD [From (i)] OD 4 2
ÐOBA = ÐODC [From (ii)]
Þ OD = 6 cm
\ By AA similarity criterion,
DAOB ~ DCOD
25. (A) (c) 15 cm
Explanation:
CO
(B) (b) Width of model
DO Scale factor =
Actual width of boot
Explanation: Draw EF || AB, also parallel to
CD. 1 Width of model
Þ =
D C 4 60
60
Þ Width of model = = 15 cm
E F 4
O
(D) (d) 5 m
Explanation:
Since, the two triangles are similar
A B 5 12.5
\ =
In DACD, OE || CD 2 x
AE AO (where x is the length of shadow of tree)
= ...(i)
ED OC 12.5 × 2 25
Þ x= =
[by basic proportionality theorem] 5 5
In DABD, OE || BA = 5m
ED OD 26. (B) (b) SAS
=
AE OB Explanation: For smaller triangles at
the upper part of the kite, SAS similarity
AE OB
= ...(ii) criterion is applicable.
ED OD
(E) (b) Thales Theorem
[by basic proportionality theorem]
Explanation: Thales Theoem states that
AO OB if a line is drawn parallel to one side of
=
OC OD a triangle intersecting the other sides in
AO CO distinct points, then the other two sides are
= divided in the same ratio.
BO DO
(E) (a) 6 cm 27. (B) Using basic proportionality theorem, we have
Explanation:
B A AD AE
2 =
cm cm DB EC
4
x +3 x
O 3 Þ =
cm 3 x + 19 3x + 4
Þ (x + 3)(3x + 4) = x(3x + 19)
3x2 + 13x + 12 = 3x2 + 19x
D C Þ 13x + 12 = 19x
As AB || CD, the triangles COD and BOA Þ –6x = –12
are similar by AA similarity criterion as Þ x=2
ÐDCO = ÐABO
[Alternate interior angles] 28. (B) Now, as triangles ABC and DEF are similar,
ÐCOD = ÐBOA so their sides are proportional.
[Vertically opposite angles]
AB AC BC
Now, as DCOD ~ DBOA \ = =
DE DF EF

Triangles 45

Click here to buy other Educart books on Amazon - https://amzn.to/40U8Txx


AB BC 39. Let the two triangles be DABC and DPQR such
Þ =
DE EF that AB = 9 cm.

21 Now, DABC ~ DPQR [Given]


15
Þ =
10 EF A P

10 × 21
Þ EF = = 14 m
15
Hence, the measure of side EF is 14 m.

(C) As triangles ABC and DEF are similar, their


corresponding sides must be proportional.
B C Q R
DE DF
\ =
AB AC AB BC CA Perimeter of ∆ABC
\ = = =
10 DF PQ QR RP Perimeter of ∆PQR
Þ =
15 AC
AB Perimeter of ∆ABC
DF ⇒ =
2 PQ Perimeter of ∆PQR
Þ =
AC 3
9 30
Thus, the ratio of sides DF : AC is 2 : 3 ⇒ =
PQ 20
29. (B) Length of shadow of Ajay’s house = 20 m
⇒ PQ = 6
Height of Vijay's house
\ Hence, the length of corresponding side of
Length of its shadow
second triangle is 6 cm.
Height of Ajay's house
=
Length of its shadow 40. Given: DPQR is right-angled at P, and PM ^ QR.
To prove: PQ2 = QM × QR
20 Height of Ajay's house
Þ = Proof: In DPMQ and DRPQ,
10 20 ÐPMQ = ÐRPQ = 90º
Þ Height of Ajay’s house = 40 m
and ÐQ is common.
34. Since DAHK ~ DABC \ By AA similarity criterion,
Then their corresponding sides will be ∆PMQ ~ ∆RPQ
proportional.
PQ RQ
AH HK AK \ =

\ = = MQ PQ
AB BC AC
HK AK ⇒ PQ2 = QM × QR

Þ =
BC AC Hence, proved.
7 10

Þ =
AC
43. Yes.
3.5

Þ AC = 5 cm

35. In DABC and DADE,


ÐABC = ÐADE
 (as DE || BC, Corresponding angles)
ÐBAC = ÐDAE (Common angle)

\ DABC ~ DADE
AD DE Let the sides of the first triangle be a, b and x.
So, =
AB BC
Then, the corresponding two sides of the other
1.5 DE triangle will be 3a, 3b and let the third side be

Þ =
6 8 y respectively.
4 Perimeter of the first triangle,

Þ DE = × 1.5 = 2 cm P1 = a + b + x
3

46 Mathematics Class X

Click here to buy other Educart books on Amazon - https://amzn.to/40U8Txx


Perimeter of the second triangle, DQ 6 1
and = =
P2 = 3a + 3b + y QF 18 3
It is given that,
DP DQ
Þ P2 = 3P1 Since, ≠
Þ 3a + 3b + y = 3(a + b + x) PE QF
Þ y = 3x So, converse of Thales theorem does not apply
here.
AB AC BC 1
So,
= = = Hence, PQ is not parallel to EF.
DE DF EF 3
Hence, DABC ~ DDEF, by SSS similarity criterion. 54. In triangles LMK and PNK, we have
ÐM = ÐN = 50°
45. (A) In ΔABD and ΔCBE
and ÐK = ÐK [Common angle]
ÐADB = ÐCEB = 90º So, by AA similarity criterion,
C
DLMK ~ DPNK
D
Thus,
P
LM KM
=
PN KN
LM MN + NK
A B Þ =
E PN KN
ÐABD = ÐCBE (Common angle) a b + c
Þ =
Þ ΔABD ~ ΔCBE (AA criterion) x c
ac
(B) In ΔPDC and ΔBEC Þ x=
b + c
ÐPDC = ÐBEC = 90º
56. Given: In DDEF, R and S are points on sides DE
ÐPCD = ÐBCE (Common angle)
and EF, respectively.
Þ ΔPDC ~ ΔBEC (AA criterion)
[CBSE Marking Scheme SQP Basic 2022]

48. No, the given statement is not correct.


We know that by SAS similarity criterion, if one
angle of a triangle is equal to one angle of the
other triangle and the sides including these
angles are proportional, then the two triangles
are similar.
Here, an angle of one triangle is equal to an
angle of another triangle and two sides are
proportional, even then the triangles are not
We have,
similar because these proportional sides may
RE = 5 cm and RD = 2.5 cm
or may not include the equal angles.
RE 5 2
So, = =
51. We have, RD 2.5 1
Similarly, we have
ES = 1.5 cm and SF = 3.5 cm
ES 1.5 3
So, = =
SF 3.5 7
Here, we can see that
RE ES
DP DP ≠≠
= RD SF
PE DE − DP
So, converse of Thales theorem does not apply
5 5 1 here.
= = =
15 – 5 10 2 So, RS is not parallel to DF.

Triangles 47

Click here to buy other Educart books on Amazon - https://amzn.to/40U8Txx


57. Given: AB = 4 cm, DE = 6 cm, EF = 9 cm, AB AC
=
FD = 12 cm and ∆ABC ~ ∆DEF. AD AE


AD AE
or =
AB AC
AD AB

⇒ =
AE AC
Now, in DADE and DABC, we have
AD AB
=
AE AC

Since, ∆ABC ~ ∆DEF  [Given] and, ÐA is common.



\ DADE ~ DABC [SAS similarity]
AB BC AC

\ = =
\ ÐD = ÐB and ÐE = ÐC
DE EF DF
But ÐD = ÐE [Given]
4 BC AC
\ ÐB = ÐC
Þ = =
6 9 12
⇒ AC = AB 
Taking first two terms, we have [Sides opposite to equal angles
 are equal]
4 BC
=
\ BAC is an isosceles triangle.
6 9
Hence, proved.
4×9 63. Given: ABCD is a quadrilateral in which AC

Þ BC =
6 and PQ intersect each other at the point O and

Þ BC = 6 cm AB || DC
Taking last two terms, we have To prove: OA.CQ = OC.AP
Proof: In ∆AOP and ∆COQ,
BC AC
= ÐAOP = ÐCOQ
9 12
[Vertically opposite angles]
6 AC
Þ = ÐAPO = ÐCQO [AB || DC and PQ is
9 12
transversal, equal alternate angles]
6 × 12 \ ∆AOP ~ ∆COQ

Þ AC =
9 [By AA similarity criterion]

Þ AC = 8 cm OA AP
\ = [Corresponding sides of
Perimeter of ∆ABC = AB + BC + AC OC CQ

= 4 + 6 + 8 = 18 cm two similar triangles are proportional]


⇒ OA × CQ = OC × AP
Thus, the perimeter of the ∆ABC is 18 cm.
Hence, proved.
AD AE 69. For the Theorem :
62. Given: ÐD = ÐE and=
BD EC Given, To prove, Construction and figure
To prove: BAC is an isoceles triangle.
Proof
D C
Proof: We have,

AD AE
=
BD EC E
G
F

BD EC
or, =
AD AE

BD EC

⇒ +1 = +1  A B
AD AE
Let ABCD be a trapezium DC || AB and EF is a
 [Adding 1 to both sides]
line parallel to AB and hence to DC.
BD + AD EC + AE DE CF

⇒ = To prove : =
AD AE EA FB

48 Mathematics Class X

Click here to buy other Educart books on Amazon - https://amzn.to/40U8Txx


Construction : Join AC, meeting EF in G. Note that DBDE and DDEC are on the same
Proof : In DABC, we have base DE and between the same base DE and
GF || AB between the same parallels BC and DE.
CG CF So, ar(BDE) = ar(DEC) ...(iii)
= [By BPT] ...(i)
GA FB Therefore, form (i), (ii) and (iii), we have:
In DADC, we have AD AE
=
EG || DC (EF || AB & AB || DC) DB EC

DE CG
= [By BPT] ...(ii) Let ABCD be a trapezium with AB||DC
EA GA
AD and BC are non-parallel sides.
From (i) & (ii), we get, Let, EF be a line parallel to AB, DC.
DE CF Join AC such that DACD and DACB are triangles
=
EA FB and AC meets EF at G.
[CBSE Marking Scheme SQP Std. 2022] D C

Detailed Answer:
We are given a triangle ABC in which a line
parallel to side BC intersects other two sides E F
AB and AC at D and E respectively. G

N M
A B

D E Apply Thales theorem on DACD and DACB


Basic Proportionality Theorem: In a triangle, a
line drawn parallel to one side to intersect the
other sides distinct points divides two sides in
B same ratio.
C
In DACD
AD DB . Here, EG||DC. According to Basic Proportionality
We need to prove that =
AE EC Theorem
Let us join BE and CD and then draw DM ^ AC AE AG
= ...(i)
and EN ^ AB. ED GC
1 In DACB
Now, area of DADE = ( base × height)
2 Here, GF||AB.
According to Basic Proportionality Theorem
1
= AD × EN. FC GC
2 =
area of DADE is denoted as ar(ADE). BF AG
1 BF AG
So, ar(ADE) = AD × EN
Þ = ...(ii)
2 FC GC

Equating equation (i) and equation (ii)
1
Similarly, ar(BDE) = DB × EN, AE BF
2 =
ED FC
1 Hence proved that a line drawn parallel to the
ar(ADE) = AE × DM
2 parallel sides of a trapezium divides the non-
1 parallel sides proportionally.
and ar(DEC) = EC × DM
2 70. Given: In ΔABC, PQRS is a parallelogram and
1 AB || PS.
ar( ADE ) × AD × EN To prove: OC || SR
AD
Therefore, = 2 = ...(i) Proof: Since, PQRS is a parallelogram,
ar(BDE ) 1 DB
× DB × EN \ PQ || SR and PS || QR
2
Also given, AB || PS
1 \ AB || PS || QR
ar( ADE ) × AE × DM
AE Now, AB || PS
= 2 = ...(ii)
ar(DEC ) 1 EC \ ÐOSP = ÐOBA...(i)
× EC × DM
2  [Corresponding angles]

Triangles 49

Click here to buy other Educart books on Amazon - https://amzn.to/40U8Txx


and, QR || AB From equations (iii), (iv) and (v), we get
\ ÐCRQ = ÐCBA ...(ii) OS CR
 [Corresponding angles] =
OB CB
Now, in ΔOAB and ΔOPS,
ÐPOS = ÐAOB [Common angle] OB CB
or =
ÐOSP = ÐOBA [From (i)] OS CR
\ ∆OPS ~ ∆OAB Subtracting 1 from L.H.S and R.H.S, we get
 [By AA similarity criterion]
OB CB
PS OS −1 = −1
\ =  …(iii) OS CR
AB OB
Similarly, in ∆CQR and ∆CAB, OB − OS CB − CR
=
OS CR
ÐQCR = ÐACB  [Common angle]
ÐCRQ = ÐCBA  [From (ii)] BS BR
=
\ ∆CQR ~ ∆CAB OS CR

 [By AA similarity criteria]

\ By the converse of Thales theorem in DOBC,
QR CR
\ = we get
AB CB  ...(iv)

Þ SR || OC
Since PQRS is a parallelogram,

\ PS = QR ...(v) Hence, proved.

50 Mathematics Class X

Click here to buy other Educart books on Amazon - https://amzn.to/40U8Txx


 ANSWER SHEET

SELF PRACTICE

4. (d) IV quadrant Comparing the two, we get

Explanation: We know that, if P (x, y) divides a 
 , 4 = (–4, 4)
the line segment joining A (x1, y1) and B (x2, y2) 3
internally in the ratio m : n, then
a
Þ = –4
mx2 + nx1 my2 + ny1 3
x= and y =
m+n m+n
Þ a = –12

Given that x1 = 7, y1 = –6, x2 = 3, y2 = 4, Hence, the required value of a is –12.
m = 1, n = 2
14. (a) (0, 13)
1(3) + 2(7) 3 + 14 17
∴ x= = =
Explanation: Let the perpendicular bisector of
1+2 3 3 the line segment joining the points A(1, 5) and
1( 4 ) + 2( −6 ) 4 − 12 −8 B(4, 6) cuts the y-axis at P.
y = = =
1+ 2 3 3
As x-coordinate is positive and y-coordinate is
negative,
 17 −8 
∴ (x, y) =  ,  lies in the IV quadrant.
 3 3
7. (a) (0, 0)
Explanation: Let P(α, β) be a point which lies on Then, point P will be of the form (0, b) as any
the perpendicular bisector of the line segment point on the y-axis will have x-coordinate zero.
AB.

We know that, AP = PB
2
( )2 ( ) = (4 − 0)2 + (6 − b)2
0 − 1 + b − 5
P (, b )
2 2
Þ 1 + (b − 5) = 16 + (6 − b)
A (–2, –5) B (2, 5)
Also, P is the mid-point of AB. Þ 1 + (b – 5)2 = 16 + (6 – b)2
 −2 + 2 −5 + 5  [Squaring both sides]

\ P(a, b) =  , 
 2 2  Þ 1 + b2 + 25 – 10b = 16 + 36 + b2 – 12b
= (0, 0) Þ 12b – 10b = 52 – 26
So, (0, 0) lies on the perpendicular bisector. Þ 2b = 26
Þ b = 13
13. (b) –12
a 
Hence, point P is (0, 13).

Explanation: It is given that, P  , 4 is the
3  19. (c) − 3
mid-point of the line segment joining the
points Q(–6, 5) and R(–2, 3). ( )
Explanation: Since, A 3, 3 , B(0, 0) and C(3, k)
 −6 − 2 5 + 3   −8 8 
\ Mid-point of QR = P  , = ,  are vertices of an equilateral triangle,
 2 2   2 2
\ AB = BC = CA
 −6 − 2 5 + 3   −8 8  or, (AB)2 = (BC)2 = (CA)2
P , ==  , 
( )
 2 2   2 2 2

Þ (0 – 3)2 + 0 − 3 = (3 – 0)2 + (k – 0)2
= P(–4, 4)

( )
a  2
But mid-point is given as P  , 4 . = (3 – 3)2 + k − 3
3 

Coordinate Geometry 51

Click here to buy other Educart books on Amazon - https://amzn.to/40U8Txx


Clearly, 3.1 < 6.5
( )
2
Þ (0 – 3)2 + 0 − 3
= (3 – 0)2 + (k – 0)2
 7
Þ Point 2,  lies in the interior of the
Þ

9+3=9+k 2  3
Þ

k2 = 3 circle.
Þ k= ± 3  1

(c) Distance between (0, 0) and 5, − 
3  2
But for k =

( )
2
2 2
CA = (3 − 3) + 3− 3  
= (5 − 0)2 +  −21 − 0  = 25 +
1
4
= 0, which is not possible

\ k= − 3 101 10.04
= = = 5.02
4 2
 5
20. (d)  −6,  Clearly, 5.02 < 6.5.
 2

Explanation: It is given the that centre of the  −1
circle is origin i.e., O (0, 0) and it passes through Þ Point 5, lies in the interior of the circle.
 2 
 13 
 , 0   5
2 (d) Distance between (0, 0) and −6, 
 2
Þ Radius of circle = Distance between
2
 13   
(0, 0) and  , 0  .
2  = (−6 − 0)2 +  25 − 0  = 36 +
25
4
2
 13  144 + 25 169 13
=  − 0 + ( 0 − 0 )2 = = =
2  4 4 2
= 6.5 = radius
2
 13 
=    5
2 So, the point −6,  lies on the circle and
 2
13 not in the interior.
= = 6.5
2
We know that the point which does not lie 2
22. (a)
in the interior of circle will be at a distance 3
greater than the radius from the centre.

Since, P divides the line segment joining R(–1,
 3  3) and S(9, 8) in ratio k : 1
(a) Distance between (0, 0) and − , 1
 4 
 9k − 1 8k + 3 

\ Coordinates of P are  , 
2
 3  ( 2 9  k +1 k +1 
=  − − 0 + 1 − 0) = +1
4 16 Since, P lies on the line x – y + 2 = 0,
25 5 9k − 1 8k + 3
= = = 1.25
then, − +2 = 0
16 4 k +1 k +1
Clearly, 1.25 < 6.5
9k – 1 – 8k – 3 + 2k + 2 = 0
 −3  2
Þ Point  , 1 lies in the interior to the
which gives, k = .
 4  3
circle. [CBSE Marking Scheme Term-1 SQP 2021]
 7 Explanation: Using section formula,
(b) Distance between (0, 0) and  2, 
 3 Coordinates of P
2  k × 9 + 1x( −1) k × 8 + 1 × 3 
7  49
=  , 
= (2 − 0 )2 +  − 0 = 4 +  k +1 k +1 
3  9
 9k − 1 8k + 3 
36 + 49 85 9.22
=  , 
= = = = 3.1  k +1 k +1 
9 9 3

52 Mathematics Class X

Click here to buy other Educart books on Amazon - https://amzn.to/40U8Txx



But the two mid-points are the same.
x +6 y +7
1
So, = 3 and =3
2 2
k
R(–1, 3) P S(9, 8) ⇒ x + 6 = 6 and y + 7 = 6
⇒ x=0 and y = –1
x–y+2=0
Hence, the fourth vertex D (x, y) = (0, –1).

Since, point P lies on the line x – y + 2 = 0,
31. 5 units

so, it must satisfy the equation,
Explanation:
9k − 1 8k + 3

\ − +2 = 0 O(0, 0) B(4, 0)
k +1 k +1

9k − 1 − (8k + 3) + 2( k + 1)

Þ =0 C
k +1 A(0, –3)

3k − 2 In a rectangle, both the diagonals are of equal



Þ =0 lengths.
k +1
So, by the distance formula,

Þ 3k – 2 = 0
2
AB = (4 − 0)2 + (0 + 3)2

Þ k =
3 = 16 + 9

25. (d) IV = 25 = 5

Explanation: The point P divides the line \ AB = OC = 5 units
segment A(2, –5) and B(5, 2) in the ratio of 2 : 3.
34. B(5, 2)
Explanation: Let coordinates of B be (x, y).
3 + x − 2 + y 

\ Using section formula, \ (4, 0) =  ,
 2 2 
 2 × 5 + 3 × 2 2 × 2 + 3 × ( −5)  On comparing, we get
P(x, y) =  , 
 2+3 2+3 3+ x −2 + y
4= and 0 =
2 2
 16 −11
=  , 3 + x = 8 and –2 + y = 0
 3 3  Þ
Þ x=5 and y = 2
Since, x-coordinate is positive and y-coordinate \ Coordinates of B are (5, 2).
is negative.
So, the point P lies in the IV quadrant. 38. 3 units
28. (b) (0, –1) Explanation: Distance of a point from y-axis
 = |x-coordinate of that point|

Explanation: It is given that ABCD is a
\ Distance of (–3, 4) from y-axis = |–3| = 3 units
parallelogram with vertices A (–2, 3), B (6, 7)
and C (8, 3). 44. True.
Let fourth vertex be D (x, y). Explanation: We know that opposite sides of
a rectangle are equal. Also, its diagonals are
We know that diagonals of a parallelogram equal and bisect each other.
bisect each other.
A(–1, –2), B(4, 3), C(2, 5) and D(–3, 0)

\ Mid-point of AC = Mid-point of BD Distance between A(–1, –2) and B(4, 3)

Now,

AB = (4 − ( −1))2 + (3 − ( −2))2 = (4 + 1)2 + (3 + 2)2
Midpoint of diagonal AC
 −2 + 8 3 + 3   6 6  = 52 + 52 = 5 2
(x1, y1) =  ,  =  ,  = (3, 3)
Distance between B(4, 3) and C(2, 5)
2 2  2 2
Mid-point of diagonal BD

BC = (2 − 4)2 + (5 − 3)2 = ( −2)2 + (2)2
 x +6 y +7
(x2, y2) =  ,  =
 2 2  4+4 =2 2

Coordinate Geometry 53

Click here to buy other Educart books on Amazon - https://amzn.to/40U8Txx


Distance between C(2, 5) and D(–3, 0) 47. (a) Both assertion (A) and reason (R) are true
2 2 2
( −3 − 2) + (0 − 5) = ( −5) + ( −5) 2 and reason (R) is the correct explanation
CD =
of assertion (A).
= 52 + 52 = 5 2
Explanation:
y-axis
Distance between A(–1, –2) and D(–3, 0)
(0, 4)

AD = ( −3 − ( −1))2 + ( 0 + 2)2 = ( −2)2 + 22

= 4+4 =2 2 x-axis
(0, 0)
Distance between A(–1, –2) and C(2, 5)


AC = (2 − ( −1))2 + (5 − ( −2))2

(2 + 1)2 + (5 + 2)2 We can see that the point (0,4) lies on y-axis
=
and the x-coordinate on the point on y-axis is
= 32 + 72 = 9 + 49 = 58 zero.
Distance between B(4, 3) and D(–3, 0) Hence, both assertion and reason are true and
the reason is the correct explanation of the

BD = ( −3 − 4)2 + (0 − 3)2 = ( −7)2 + ( −3)2
assertion.
= 49 + 9 = 58 50. (A) (c)  2 , 1
Clearly, AB = CD, AD = BC and AC = BD  3 
i.e., opposite sides are equal and diagonals are
also equals. Centroid of DEHJ with E(2, 1), H(–2, 4) &
Hence, points A(–1, –2), B(4, 3), C(2, 5) and J(–2, –2) is:
D(–3, 0) form a rectangle.  2 + −2 + −2 1 + 4 + −2   2 
 ,  =  − 3 , 1
46. (a) Both assertion (A) and reason (R) are true 3 3
and reason (R) is the correct explanation of
assertion (A) [CBSE Marking Scheme Term-1 SQP 2021]
[CBSE Marking Scheme SQP Std. 2022] Explanation: From the graph,
Explanation:  M and N are the mid-point of Coordinates of E = (2, 1)
AB and AC. Coordinates of H = (–2, 4)
\ MN || BC (mid-point theorem) Coordinates of J = (–2, –2)
\ In DAMN and DABC \ Centroid of DEHJ
ÐA = ÐA x + x + x y +y +y 
=  1 2 3 , 1 2 3 
(common)  3 3 
ÐAMN = ÐABC
(corresponding angles)  2 + ( −2) + ( −2) 1 + 4 + ( −2) 
=  , 
ÐANM = ÐACB  3 3 
(corresponding angles)
\ DAMN ~ DABC (AAA)  2 3  2 
=  − ,  =  − , 1
 3 3  3 
AM MN AN 1
\ =  
AB BC NC 2  3
(B) (c)  2, 
MN 1  2
\ =
BC 2
If P needs to be at equal distance from
BC = 2MN A(3, 6) and G(1, –3), such that A, P
and G are collinear, then P will be the
\ MN = ( 3 3)2  ( 3 5)2
mid point of AG.
So, coordinates of P will be :
= 36  64  10
 3 + 1 6 + ( −3)   3 
\ BC = 2 × 10 = 20  2 , = 2,
2   2 
Hence, both assertion and reason are true and
reason is the correct explanation of assertion. [CBSE Marking Scheme Term-1 SQP 2021]

54 Mathematics Class X

Click here to buy other Educart books on Amazon - https://amzn.to/40U8Txx


(D) (b) (0, 1) Applying the section formula, we get
Let the coordinates of the position of 1 × 5 + 3 × 1 1 × 5 + 3 × 1 
a player Q such that his distance from (x, y) =  , 
 1+3 1+3 
K(–4, 1) is twice his distance from 8 8
E(2, 1) be Q(x, y) then KQ : QE = 2: 1 x =  ,  = (2, 2)
4 4
 2 × 2 + 1 × ( −4 ) 2 × 1 + 1 × 1
Q(x, y) =  ,  53. (C) From the graph,
 3 3
B = (0, 7), C = (5, –5), D = (–4, –2)
= (0, 1) \ Using distance formula,
[CBSE Marking Scheme Term-1 SQP 2021] BC = (5 − 0 )2 + ( −5 − 7)2
Explanation: Let the coordinates of Q be
(x, y). = (5)2 + ( −12)2 = 25 + 144
From the graph, K = (–4, 1) and E = (2, 1 = 169 = 13 units
According to question,
and CD = ( −4 − 5)2 + ( −2 + 5)2
QK = 2KE
QK 2
Þ KE = 1 = ( −9)2 + (3)2 = 81 + 9

units
= 90 = 3 10
54. (C) A(–2, 2), G(–4, 7)
∵ Points K, Q, E are collinear, Let the point on y-axis be Z(0, y)
\ Using section formula, AZ2 = GZ2
 2 × 2 + 1 × ( −4 ) 2 × 1 + 1 × 1  (0 + 2) + (y – 2)2 = (0 + 4)2 + (y – 7)2
2

Q(x, y) =  ,  (2)2 + y2 + 4 – 4y = (4)2 + y2 + 49 – 14y


 2+1 2+1 
8 – 4y = 65 – 14y
 4 − 4 2 + 1
=  , 10y = 57
 3 3 
So, y = 5.7
 3 i.e. the required point is (0, 5.7)
=  0,  = (0, 1)
 3 OR
13
51. (D) (a) (16, 0)
R Q
12
S P
Explanation: The coordinates of D are 11
10
(16, 0). T 9 O
F E
8
(E) (b) (-12, 6) U G 7 D N
Explanation: Given, to take the D as origin, 6
then the coordinates of P are (–12, 6). 5
V H 4 C M
3  A
3
B
52. (B) (d)  , 4 W 2 L
2  1
0
Explanation: The coordinates of points Q X K
0 –9–8–7 –6 –5 –4 –3 –2 –1 –11 2 3 4 5 6 7 8
and R are (1, 3) and (2, 5) respectively. I –2 J

By applying the mid-point formula the


–3
–4
Coordinates of the mid-point of QR
3  A(–2, 2), F(–2, 9), G(–4, 7), H(–4, 4)
1 + 2 3 + 5 
=  , or  , 4  Clearly GH = 7 – 4 = 3units
 2 2  2
AF = 9 – 2 = 7 units
(D) (c) (2, 2) So, height of the trapezium AFGH = 2 units
Explanation: The coordinates of the points 1
So, area of AFGH = (AF + GH) x height
P and S are (1, 1) and (5, 5) respectively. 2
Let the coordinates of required point be
(x, y). 1
= (7 + 3) × 2
1 3 2
= 10 sq. units
P(1, 1) (x, y) S (5, 5)
(CBSE Marking Scheme SQP 2022)

Coordinate Geometry 55

Click here to buy other Educart books on Amazon - https://amzn.to/40U8Txx


55. (C) Explanation: The coordinates of science Explanation: Let P(x, 0) be the required point
lab, grocery store and hotel are (1, 5), on the x-axis. And the given points are A(2, – 2)
(4, 5) and (3, 4) respectively. and B(–4, 2).
Using the distance formula, we can find \ PA = PB
the distances between the science lab and or PA2 = PB2
grocery store and also between grocery Þ (x – 2)2 + (0 + 2)2 = (x + 4)2 + (0 – 2)2
store and hotel. [by distance formula]
Þ x2 + 4 – 4x + 4 = x2 + 16 + 8x + 4
Distance between science lab and grocery Þ –4x + 4 = 8x + 16
store = ( 4  1)2  (5  5)2 = 3 units Þ –12x = 12
Distance between grocery store and hotel Þ x=–1
\ Coordinates of P are (–1, 0).
= (3  4 )2  ( 4  5)2 = 2 units
Total distance travelled by Sumit 68. Consider a parallelogram ABCD with A (3, 2)
= (3 + 2 ) units and B (–1, 0).

56. (C) L(5, 10), N(2, 6), P(8, 6)


LN = (2 − 5)2 + (6 − 10 )2 = (3)2 + ( 4 )2

9 + 16 = 25 = 5

NP = ( 8 − 2)2 + (6 − 6 )2 = ( 4 )2 + ( 0 )2 = 4

PL = ( 8 − 5)2 + (6 − 10 )2 = (3)2 + ( 4 )2

And the coordinates of the point where
Þ PL = 9 + 16 = 25 = 5 diagonals AC and BD intersect are M(2, –5).
as LN = PL ¹ NP, so DLNP is an isosceles
Let the coordinates of other two vertices i.e.,
triangle. C be (x1, y1) and D be (x2, y2).
OR

Since, diagonals of a parallelogram bisect each
Let A (0, b) be a point on the y – axis then
AL = AP other, so, M is the mid-point of AC and BD.

For line AC, where M is its mid-point.
Þ (5 − 0 )2 + (10 − b )2 = ( 8 − 0 )2 + (6 − b )2
 3 + x1 2 + y1 
Þ (5)2 + (10 − b )2 = ( 8 )2 + (6 − b )2
Then, M(2, –5) =  , 
 2 2 
Þ 25 + 100 – 20b + b2 = 64 + 36 – 12b + b2
Þ 8b = 25 3 + x1
i.e., 2=
25 2
Þ b =
8
2 + y1
 25  and –5=
So, the coordinate on y axis is 0, . 2
  8  Þ x1 = 1 or y1 = – 12
[CBSE Marking Scheme SQP 2022] \ Coordinates of C are (1, –12)
Similarly,
58. Given points are A(c, 0) and B(0, –c) M is the mid-point of BD.
2 2 Then,

\ Distance AB = ( c − 0 ) + ( 0 + c )
 −1 + x2 0 + y2 
M (2, −5) =  , 
= c2 + c2 = 2c 2 = c 2  2 2 

64. Let P (X, 0) be a point on X-axis −1 + x2


i.e., 2=
PA = PB 2

Þ x2 = 5
PA2 = PB2
(x – 2)2 + (0 + 2)2 = (x + 4)2 + (0 – 2)2 0 + y2
x2 + 4 – 4x + 4 = x2 + 16 + 8x + 4
and −5 =
2
–4x + 4 = 8x + 16
x = –1
Þ y2 = –10
P (–1, 0) Hence, the coordinates of the other two
[CBSE Marking Scheme SQP 2020] vertices are (1, –12) and (5, –10).

56 Mathematics Class X

Click here to buy other Educart books on Amazon - https://amzn.to/40U8Txx


70. Let, the ratio in which point P(–3, k) divides the On squaring both sides, we get
given line segment be m : 1 Þ (2a – 2)2 + (a + 5)2 = (2a + 3)2 + (a – 6)2
A m (–3, k) 1 B Þ 4a2 – 8a + 4 + a2 + 10a + 25
(–5, –4) (–2, 3) = 4a2 + 12a + 9 + a2 – 12a + 36
Þ –8a + 10a = 9 + 36 – 25 – 4

\ By the section formula,
Þ 2a = 16
 m x + m2 x1 m1y2 + m2y1  Þ a =8
P(x, y) =  1 2 , 
 m1 + m2 m1 + m2  Then, y-coordinate = 8
and x-coordinate = 16
 m( − 2) + ( − 5) m(3) + ( − 4 ) 
Þ (–3, k) =  , Hence, the coordinates of the required point P
 m+1 m + 1 
are (16, 8).
−2m − 5 81. Let, the coordinates of point P be (x, y).
\ –3 =
m +1 1
AP
Þ – 3 (m + 1) = –2m – 5 Now, =
AB 3
Þ –3m – 3 = –2m – 5
AB 3
Þ – m = –2 or =
Þ m =2 AP 1
\ The required ratio is 2 : 1. AB

Þ – 1 = 3 – 1
3m − 4 AP
Also, k=  [Subtracting 1 from both sides]
m +1
Put m =2 AB − AP

Þ =2
AP
6−4 2
Þ k= = BP 2
3 3 Þ =
AP 1
Hence, the required ratio is 2 : 1 and the value
AP 1
2 or =
of k is . BP 2
3
1 2
75. Let, the coordinates of point P be (x, y). A(2, 1) P(x, y) B (5, –8)

The ratio by which P divide AB is 1 : 2
\ Using section formula,
coordinates of P are given by
 1 × 5 + 2 × 2 1 × ( −8 ) + 2 × 1
 4 − 3a + 6a − 4 a − 1 + 4 + 2a  P(x, y) =  , 
P ,   1+2 1+2
 3 3
 9 −6 
 3a 3a + 3  =  ,
= P ,   3 3 
 3 3 
= (3, – 2)
= P(a, a + 1)
So, 2a – 3(a + 1) + 5 = 0 Since, P(3, –2) lies on the line 2x – y + k = 0,
so it must satisfy the equation of line.
–a + 2 = 0
a =2
\2(3) – (–2) + k = 0
76. Let the y-coordinate of point P be ‘a’.
Þ k=–8
Then, the x-coordinate is ‘2a’. Hence, the value of k is –8.
\ Coordinates of P are (2a, a).
82. Let P(r, s) be a point which is equidistant from
Since, point P is equidistant from Q(2, –5) and the points A(–5, 4) and B(–1, 6).
R(–3, 6), then by the distance formula ∴ PA = PB
PQ = PR Þ (PA)2 = (PB)2
Þ (–5 – r)2 + (4 – s)2 = (–1 – r)2 + (6 – s)2
Þ ( x − x1 )2 + ( y − y1 )2 = ( x − x2 )2 + ( y − y2 )2 Þ 25 + r2 + 10r + 16 + s2 – 8s
Here, x = 2a, y =a = 1 + r2 + 2r + 36
+ s2 – 12s
x1 = 2, y1 = –5
Þ 25 + 10r + 16 – 8s = 1 + 2r + 36 – 12s
x2 = –3, y2 = 6 Þ 8r + 4s + 4 = 0
Þ 2r + s + 1 = 0
\ (2a − 2)2 + ( a + 5)2 = (2a + 3)2 + ( a − 6 )2
Þ 2r + s = –1 …(i)

Coordinate Geometry 57

Click here to buy other Educart books on Amazon - https://amzn.to/40U8Txx



Now, 87. Here, line AB is trisected at points P and Q.
 −5 − 1 4 + 6  Let the co-ordinates of P be (x, y).

Mid-point of AB =  ,  = (–3, 5)
 2 2  Then, AP : PB = 1 : 2
By the section formula,

At point (–3, 5), from eqn (i), we get
Þ 2r + s = 2 (–3) + 5

0
k=
= –6 + 5 = –1

y+
Þ 2r + s + 1 = 0

2x –
Hence, mid-point of AB satisfies eqn (i). This B
A (2, 1) P (x, y) Q (5, – 8)
implies that there are infinite number of points
which satisfy eqn (i) and are equidistant from
points A and B.
84. Let the line x – 3y = 0 intersect the segment  mx2 + nx1 my2 + ny1 
P(x, y) =  , 
joining A(–2, –5) and B(6, 3) in the ratio k : 1 at  m+n m+n 
point P(x, y).
Here, m = 1, n=2

By using the section formula, coordinates of
x1 = 2, y1 = 1
P(x, y) are:
x2 = 5, y2 = –8
 mx + nx1 my2 + ny1 
P(x, y) =  2
,   1 × 5 + 2 × 2 1 × ( −8 ) + 2 × 1
 m+n m+n 
Then, P(x, y) =  , 
 1+2 1+2
Here x1 = –2, y1 = –5
x2 = 6, y2 = 3  9 −6 
=  , = (3, − 2)
 3 3 
m = k, n= 1
 6k − 2 3k − 5 
So, the co-ordinates of P are (3, –2).
\ P(x, y) =  , 
Since, point P lies on the line 2x – y + k = 0, so
 k +1 k +1 

its co-ordinates will satisfy the equation of the

But, P lies on the line x – 3y = 0, line.
i.e., x = 3y \ 2(3) – (–2) + k = 0
 3k − 5  Þ 6+2+k=0
6k − 2
\ = 3  Þ k = –8
k +1  k +1 
Hence, the value of k is –8.
Þ 6k – 2 = 9k – 15
88. It is given that A (2, –4) is equidistant from
Þ 3k = 13
P (3, 8) and Q (–10, y).
Þ k = 13 Þ Distance between P (3, 8) and A (2, –4)
3
= Distance between A (2, –4) and Q (–10, y)
  13  
13 ( −10 − 2)2 + ( y + 4)2
 6× − 2 3  3  − 5  Þ (2 − 3)2 + ( −4 − 8)2 =
3  
\ Coordinates of P =  , 
 13 13  2
 +1 +1  Þ ( −1)2 + ( −12)2 = ( −12)2 + ( y + 4)
 3 3 
Squaring both sides, we get
 78 − 6 39 − 15  (–1)2 + (–12)2 = (–12)2 + (y + 4)2
=  , 1 + 144 = 144 + y2 + 16 + 8y
 13 + 3 13 + 3  Þ
Þ y + 8y + 15 = 0
2

 72 24  Þ y2 + 5y + 3y + 15 = 0
=  ,
 16 16  Þ y (y + 5) + 3(y + 5) = 0

Þ (y + 5)(y + 3) = 0
9 3
=  ,  If y+5=0
 2 2  then y = –5
Hence, the coordinates of point of intersection, If y+3=0
9 3 then y = –3
P are  ,  .
2 2
\ y = –5, –3

58 Mathematics Class X

Click here to buy other Educart books on Amazon - https://amzn.to/40U8Txx



Distance between P (3, 8) and Q (–10, y) when 94. Let the co-ordinates of required point on x-axis
y = –3 be P(x, 0).
PQ = ( −10 − 3)3 + ( −3 − 8)2
Here, the given points are A(–2, 0) and B(6, 0).
According to the question,
( −13)2 + ( −11)2 == 290
= 169 + 121
PA = PB
= ( −13)2 + ( −11)2 = 169 + 121 = 290 or PA 2
= PB2

Distance between P (3, 8) and Q (–10, y), when By the distance formula, we have
y = –5 (x1 – x)2 + (y1 – y)2 = (x2 – x)2 + (y2 – y)2
Here, x1 = –2, y1 = 0
PQ = ( −10 − 3)2 + ( −5 − 8)2
x2 = 6, y2 = 0
(–2 – x) + (0 – 0) = (6 – x)2 + (0 – 0)2
2 2

−13 + −13
= ( )2 ( )2
Þ 4 + x2 + 4x = 36 + x2 – 12x
= 169 + 169 = 13 2 Þ 16x = 32
Hence, the values of y are –3 and –5 and the or x=2
corresponding values of PQ are 290 units Hence, the co-ordinates of P are (2, 0).
and 13 2 units, respectively.
95. Given, AB is a line segment with coordinates of
90. The given line segment is A(3, – 2) and B(–3, –4). A as (–4, 0) and coordinates of B as (0, 6).
Here, C(x, y) and C'(x’, y’) are the points of 3
and AR = AB
trisection of AB. 4
Then, AC : CB = 1 : 2 and AC’ : C’B = 2 : 1
Þ 4AR = 3(AR + RB)
1 C 1 C' 1
Þ AR = 3RB
A(3, –2) B (–3, –4) AR 3
Þ =
By section formula, RB 1
 mx2 + nx1 my2 + ny1  3 1
C(x, y) =  , 
 m+n m+n  A(–4, 0) R(x, y) B(0, 6)
Here, m = 1, n = 2 Let the coordinates of R be (x, y).
x1 = 3, y1 = –2 So, by the section formula,
x2 = – 3, y2 = –4
 mx2 + nx1 my2 + ny1 
 1 × ( −3) + 2 × 3 1 × ( −4 ) + 2 × ( −2)  R(x, y) =  , 
 m+n m+n 
\ C(x, y) =  , 
 1+2 1+2 Here, m = 3, n = 1
− 3 + 6 − 4 − 4 x1 = – 4, y1 = 0
=
 3 , 3  x2 = 0, y2 = 6
 3 × 0 + 1 × ( − 4) 3 × 6 + 1 × 0 
 8 ∴ R(x, y) =  , 
=  1, −  3+1 3+1
 3 
 9
Now, ∴ R(x, y) =  − 1, 
 2
 m′ x2 + n′ x1 m′ y2 + n′ y1 
C’ (x’, y’) =  ,  9
m' + n' m' + n'  
 Hence, the coordinates of R are  − 1,  .
 2
Here, m’ = 2, n’ = 1
 2 × ( −3) + 1 × 3 2 × ( −4 ) + 1 × ( −2)  101. Given, point C(–1, 2) which divides the line
\ C’(x’, y’) =   segment A(2, 5) and B(x, y) in the ratio of 3 : 4.
 1+2 1+2
A 3 C 4 B
 −6 + 3 −8 − 2   10 
=  , = −1, −  (2, 5) (– 1, 2) (x, y)
 3 3   3

\ By the section formula,
Hence, the coordinates of the points of trisection
 m x + m2 x1 m1y2 + m2y1 
 8 
are  1, −  and  −1, −  .
10  C(x, y) =  1 2 , 
 3  3   m1 + m2 m1 + m2 

Coordinate Geometry 59

Click here to buy other Educart books on Amazon - https://amzn.to/40U8Txx


Here, x1 = 2, y1 = 5 Taking the first and third parts, we get
x2 = x, y2 = y AB2 = AC2
x = –1, y=2 Þ (4 + 4) + (3 – 3)2 = (x + 4)2 + (y – 3)2
2

m1 = 3, m2 = 4 Þ 82 + 02 = x2 + 16 + 8x + y2 + 9 – 6y
3 × x + 4 × 2 3 × y + 4 × 5
\ C(–1, 2) =  ,  Þ x2 + y2 + 8x – 6y – 39 = 0 ...(ii)
 3+ 4 3 + 4 

Subtracting eqn. (i) from eqn. (ii), we get
 3 x + 8 3y + 20 

=  ,  Þ (x2 + y2 + 8x – 6y – 39) – (x2 + y2 – 8x – 6y – 39)
 7 7 
On equating x and y-coordinates, we get  =0
3x + 8 Þ 16x = 0
–1 =
7

Þ x =0
3y + 20
and 2 =
Putting the value of x in eq. (i), we get
7
Þ 0 + y2 – 8(0) – 6y – 39 = 0
Þ 3x = –7 – 8 = –15
Þ      y2 – 6y – 39 = 0  ...(iii)
and 3y = 14 – 20 = –6
Þ x = –5 Using quadratic formula,

and y = –2 − ( −6) ± ( −6)2 − 4(1) ( −39)


y =
\ x + y = (–5) + (–2)
2 2 2 2
2(1)

= 25 + 4 = 29 6 ± 36 + 156
Hence, the value of x2 + y2 is 29. =
2
102.
6 ± 192 6 ± 8 3
Y Þ y= = = 3±4 3
2 2
Þ y = 3 + 4 3 or y = 3 − 4 3

It is given that origin lies in the interior of
the triangle and the x-coordinate of the third
X' X vertex is zero.

So, the value of y should be negative.
∴ Third vertex, C (x, y) = (0, 3 − 4 3 )

104. Yes, we can place Jaspal in the drill.


By observing the given diagram we get to
Y' know positions of the students A, B, C and D
forming a quadrilateral.

Let the third vertex of the equilateral triangle
be C(x, y). So the vertices of this quadrilateral will be as
follows: A(3, 5), B(7, 9), C(11, 5) and D(7, 1).

The vertices of the triangle will be A(–4, 3),

To find the type of this quadrilateral, we have
B(4, 3) and C(x, y).
to calculate the length of its all sides and also
We know that in an equilateral triangle its diagonals.
AB = BC = AC
Þ AB2 = BC2 = AC2
Taking the first two parts, we get
AB2 = BC2
Þ (4 + 4) + (3 – 3)2 = (x – 4)2 + (y – 3)2
2

Þ 82 + 02 = x2 + 16 – 8x + y2 + 9 – 6y
Þ 64 = x2 – 8x + y2 – 6y + 25
Þ x2 + y2 – 8x – 6y – 39 = 0 ...(i)

60 Mathematics Class X

Click here to buy other Educart books on Amazon - https://amzn.to/40U8Txx


\ Using distance formula,
which represent a square with all its sides
equal and diagonals also equal.

AB = (7 − 3)2 + (9 − 5)2 = 42 + 42 = 4 2
We know that diagonals of a square bisect

BC = (11 − 7)2 + (5 − 9)2 = 42 + ( −4)2 = 4 2 each other, so P should be the position of
Jaspal at which he is equidistant from A, B, C

CD = (11 − 7)2 + (5 − 1)2 = 42 + 42 = 4 2 and D.


AD = (7 − 3)2 + (1 − 5)2 = 42 + ( −4)2 = 4 2 ∴ Coordinates of point P = Midpoint of AC

Now, we will find length of diagonals  3 + 11 5 + 5 


=  2 , 2 
 

AC = (11 − 3)2 + (5 − 5)2 = 82 = 8
 14 10 

BD = (7 − 7)2 + (1 − 9)2 = ( −8)2 = 8 =  , = (7, 5)
 2 2 
Þ AB = BC = CD = DA and AC = BD
Hence, the required position of Jaspal is (7, 5).


Coordinate Geometry 61

Click here to buy other Educart books on Amazon - https://amzn.to/40U8Txx


 ANSWER SHEET

SELF PRACTICE
8 17 17 12
1. (b) tan q = , cosec q = ,sec q =
15 8 15 13 = 12
=
5 5
Explanation: From the given figure,
13
P YZ 1 5
tan q = = Þ cot A = =
B YX tan A 12
8
= 12 5 169
15 So, tan A + cot A = + =
5 12 60
H XZ
cosec q = = 12. (b) 60º, 30º
P YZ
Explanation:
17
= sin(A + B) = 1 = sin 90, so A + B = 90 ...(i)
8
3
H XZ cos(A – B) = = cos 30, so A – B= 30 ...(ii)
sec q = = 2
B YX
From (i) and (ii) ÐA = 60°
17
= And ÐB = 30°
15
Detailed Answer:
7. (b) A = B = 45º We have, sin(A + B) =1
Explanation: Given, sin (A + B) = cos (A – B) = 1 (A + B) = sin–11
Þ sin (A + B) = 1 We know, sin–11 = 90°
Þ sin (A + B) = sin 90º A+B = 90° ...(i)
Þ A + B = 90º [ sin 90° = 1] ...(i) 3
Also, cos (A – B) = 1 cos(A – B) =
2
Þ cos (A – B) = cos 0º
Þ A – B = 0 or A = B [ cos 0° = 1] 3
A – B = cos −1
Putting in (i), we get 2
2A = 90º
3
Þ A = 45º we know, cos −1 = 30°
2
\ A = B = 45º
A – B = 30° ...(ii)
169 solve (i) and (ii)
8. (a)
60 A + B = 90°
5 A – B = 30°
Explanation: Given, cos A =
13 add 2A = 120°
ÐA = 60° and ÐB = 30°
25
Þ cos2 A = [CBSE Marking Scheme SQP Std. 2022]
169
25 16. (a) 0
⇒ 1 – sin2 A =
169 Explanation: sec q cot q – cosec q tan q
144 12 1 cos θ 1 sin θ
Þ sin2 A = or sin A = = · − ·
169 13 cos θ sin θ sin θ cos θ
sin A 1 1
Now, tan A = = −
cos A sin θ cos θ

62 Mathematics Class X

Click here to buy other Educart books on Amazon - https://amzn.to/40U8Txx


cos θ − sin θ Þ tan2 a – 2tan a + 1= 0

=
sin θ cos θ
Þ (tan a – 1)2 = 0

Þ tan a – 1 = 0
 1 
Þ tan a = 1
 At θ = 45º, sin θ = cos θ = 
 2 1 1
and, = =1
cot a =
= 0 tan α 1
So, tan20 a + cot20 a = (tan a)20 + (cot a)20
3 = (1)20 + (1)20
20. (b)
2 2 = 1 + 1
= 2
Explanation:
Given, 3 cos q = 1 33. (a) –1
1 Explanation: We have,
Þ cos q =
3 sec q + tan q + 1 = 0
Here, B = 1, H = 3 Þ sec q + tan q = – 1 ...(i)
Now, we know, sec2 q – tan2 q = 1
\ P= H2 − B2 = 9 − 1 = 8
Þ (sec q – tan q) (sec q + tanq) = 1
Þ (sec q – tan q)(–1) = 1 [Using (i)]
3 Þ sec q – tan q = –1
1
8 34. (c)
θ 2
1 Explanation:
1
3 3
Here, tan q =
\ cosec q = i.e., 3
8 2 2
23. (c) 3 1
Then, cot q = = 3
Explanation: We know that, tan θ
cosec2 q – cot2 q = 1 We know,
1 + tan2 q = sec2 q
Þ (cosec q – cot q) (cosec q + cot q) = 1
2
 1 
1
Þ sec2 q = 1 + 
Þ cosec q + cot q =  3

cosec θ − cot θ

1 1 4
= =3 = 1 + =
1 3 3
3 and 1 + cot2 q = cosec2 q

( 3)
2
25. (b) 30°
Þ cosec2 q = 1 +
Explanation: 2 sin 2q = 3 = 1 + 3 = 4
3 4
Þ sin 2q = 4−
2 cosec2 θ − sec2 θ 3
Now, =
Þ sin 2q = sin 60° cosec2 θ + sec2 θ 4
4+
 3 3
∵sin 60° =  8
 2 
 1
= 3 =
Þ 2q = 60° 16 2
Þ q = 30° 3
40. tan2A
29. (b) 2
Explanation:
tan a + cot a = 2 gives a = 45°.
So tan a = cot a = 1. 1 + tan2 A sec2 A
=
tan20 a + cot20 a = (1)20 + (1)20 = 1 + 1 = 2 1 + cot2 A cosec2A
[CBSE Marking Scheme Term-1 SQP 2021]
1
Explanation: We have, 2 sin2 A
Þ tan a + cot a = 2 = cos A = = tan2 A
1 1 cos2 A
Þ tan a + =2 2
tan α sin A

Introduction to Trigonometry 63

Click here to buy other Educart books on Amazon - https://amzn.to/40U8Txx


3 Hypotenuse
44. 58. (B) cosec P =
4 Side opposite to angle P
Explanation: Given, 3 sec q – 5 = 0
PR 17
Þ 3 sec q = 5 = =
QR 8
5
Þ sec q =
3
72
59. (B) (c)
25 65
Þ sec2 q =
9 Explanation:
Side adjacent to angle P
25 cot P =
Þ 1 + tan2 q = Side opposite to angle P
9 
PQ 72
[ sec2 q = 1 + tan2 q] = =
QR 65
16
Þ tan2 q = 333
9 (E) (c)
20
4 PQ
Þ tan q = 72
3 Explanation: sin R = = ;
PR 97
QR 65
3
Þ cot q = sin P = PR = ;
4 97
QR 65
tan P = = .
53. (a) Both assertion (A) and reason (R) are true and PQ 72
reason (R) is the correct explanation of
6 13
assertion (A). Now, + – 12 tan P
sin R sin P
Explanation: x = 2 sin2 q and y = 2 cos2 q + 1
Now, x + y = 2 sin2 q + 2 cos2 q + 1 6 13 65
= + – 12 ×
= 2(sin q + cos q) + 1 2 2 72 65 72
= 2 + 1 [Q sin2 q + cos2 q = 1] 97 97
= 3 97 97 65
= 6 × + 13 × –
Hence, both assertion and reason are true and 72 65 6
reason is the correct explanation of assertion.
97 97 65
= + –
54. (d) Assertion (A) is false but reason (R) is true. 12 5 6
Explanation: For the purpose of verification, 97 × 5 + 97 × 12 − 65 × 10
=
Let A = 30° and B = 60° 60
L.H.S. = sin (A + B) 999 333
= sin (30° + 60°) = =
60 20
= sin 90°
PC
= 1 60. (C) tan 60º =
AC
R.H.S. = sin A + sin B
= sin 30° + sin 60° 18
Þ 3=
AC
1 3
+ =
2 2 Þ AC = 6 3 m
(1 + 3 )
=
2 tan 30º = 6 3

Since, sin (A + B) ≠ sin A + sin B. 1 18


Þ = Þ CB = 18 3 m
We know that, for any value of q, 3 CB
2 2
1 + tan q = sec q.
Width AB = AC + CB 6 3 + 18 3 = 24 3 m
Hence, assertion is false but reason is true.

64 Mathematics Class X

Click here to buy other Educart books on Amazon - https://amzn.to/40U8Txx


OR 76. By Pythagoras Theorem
RB = PC = 18 m and PR = CB = 18 3 m QR = (13)2 − (12)2 cm

QR
tan 30º = QR = 5 cm
PR
5
tan P =
1 QR 12
Þ =
3 18 3 5
cot R =
Þ QR = 18 m 12
QB = QR + RB = 18 + 18 = 36 m.
5 5
Hence height BQ is 36 m. tan P – cot R = −
[CBSE Marking Scheme SQP Basic 2022] 12 12
= 0
opposite side CD 5
61. (A) tan q = = =
[CBSE Marking Scheme SQP 2020]
adjacent side AD 12

5 3
12 tan q = 12 × =5 80. Given, sin (A + 2B) = = sin 60°
12 2
 3
63. Given, sin x + cos y = 1 and x = 30º. Q sin 60° = 
 2 
\ sin 30º + cos y = 1 \ A + 2B = 60° ...(i)
1
Þ + cos y = 1 1
2 Also, cos (A + B) = = cos 60°
2
1 1  1
Þ cos y = 1 − = Q cos 60° = 
2 2  2

Þ cos y = cos 60º
\ A + B = 60° ...(ii)
Subtracting equations (ii) from (i), we get
 1
Qcos60° =  B = 0°
 2
\ y = 60º Putting the value of B in equation (ii), we get
Hence, the value of y is 60º.
A = 60° – 0° = 60°
3 So, A = 60° and B = 0°.
66. (1 + cos A) (1 – cos A) =
4
3 82. Given, x = a cos3 q , y = b sin3 q
Þ 1 – cos2 A =
4  x
2/3
 y
2/3
Now, L.H.S. =   +  
3 1  a  b

Þ cos2 A = 1 – =
4 4
2/3 2/3
sec2 A = 4  3   3 

Þ =  a cos θ  +  b sin θ 

Þ sec A = ±2  a   b 

72. Here, cosec q = 2x = (cos q)3 × 2/3 + (sin q)3 × 2/3


2 = cos2 q + sin2 q
and cot q = = 1 = R.H.S.
x
2
 2 [Q sin2 q + cos2 q = 1]
Now, cosec2 q – cot2 q = (2x)2 –   Hence, proved.
 x
4 85. (1 + tan2 q) (1 – sin q) (1 + sin q)

Þ 1 = 4x2 –
2
x = (1 + tan2 θ) (1 – sin2 θ)
[cosec2 q – cot2 q = 1] [Q(a – b) (a + b) = a2 – b2]
2 2
= sec θ cos θ
 1

Þ 4  x2 −  = 1 [Q 1 + tan2 θ = sec2 θ and cos2 θ = 1 – sin2 θ]
 x2 
1  1 
 1 1 = .cos2 θ = 1 Qsec θ = 

Þ 2  x2 −  = 2
cos θ   cos θ 
 2
x  2

Introduction to Trigonometry 65

Click here to buy other Educart books on Amazon - https://amzn.to/40U8Txx


3 sin θ cos θ
88. Given, tan q = tan θ + cot θ = +
4 cos θ sin θ
Squaring both sides, we get
9 sin2 θ + cos2 θ
tan2 q = =
sin θ cos θ
16
9 1
⇒ sec2 q – 1 =
[tan2 q = sec2 q – 1] =
16 sin θ cos θ
25

⇒ sec2 q =
16 1
= [Using (i)]
5 1

⇒ sec q = = 1
4
 Hence, proved.
4 1 

⇒ cos q =  Qcos θ =  102. L.H.S. = (sin q + cos q + 1) (sin q + cos q – 1) .
5  sec θ 
16 sec θ cosec θ
1 − cos θ
2 1−
So, = 25 = 9 = [(sin θ + cos θ)2 – (1)2] . sec θ cosec θ
2
1 + cos θ 1 + 16 41 [Q (a + b) (a – b) = a2 – b2]
25 = [sin2 θ + cos2 θ + 2 sin θ cos θ – 1] sec θ cosec θ
= [1 + 2 sin θ cos θ – 1] sec θ cosec θ
89. Given, tan q = 3
⇒ tan q = tan 60° [Q sin2 θ + cos2 θ = 1]
1 1
[ tan 60° = 3] = (2 sin q cos q) ×
⇒ q = 60° cos θ sin θ
2 sec θ 2 sec θ 2  1 
So, 2
= 2
= = 2 cos θ Q sec θ = 
1 + tan θ sec θ sec θ cos θ 

1 
= 2cos 60° = 2 × =1 1 
2  cosec θ = 
 sin θ 

95. L.H.S. = cot θ + cosec θ − 1
cot θ − cosec θ + 1 = 2 = R.H.S.
Hence, proved.
(cot θ + cosec θ ) − ( cosec2θ − cot2 θ ) cos A 1 + sin A
=
cot θ − cosec θ + 1 106. L.H.S. = +
1 + sin A cos A
[Q cosec­2 q – cot2 q = 1] cos2 A + (1 + sin A )2
=
(cot θ + cosec θ) − (cosec θ – cot θ) cos A (1 + sin A )
=
(cosec θ + cot θ)
cot θ − cosec θ + 1 cos2 A + 1 + sin2 A + 2 sin A
=
cos A (1 + sin A )
(cot θ + cosec θ)(1 − cosec θ + cot θ)
=
cot θ − cosec θ + 1 (cos2 A + sin2 A ) + 1 + 2 sin A
=
= cot θ + cosec θ cos A (1 + sin A)
cos θ 1 1+ cos θ 1 + 1 + 2 sin A
= + = = R.H.S. =
sin θ sin θ sinθ [ cos2 A + sin2 A = 1]
cos A (1 + sin A )
Hence, proved.
2 + 2 sin A
=
97. Given, sin θ + cos θ = 3 cos A (1 + sin A )
Þ (sin θ + cos θ)2 = 3 2 (1 + sin A )
Þ sin2 q + cos2 q + 2 sin q cos q = 3 =
cos A (1 + sin A )
Þ 1 + 2 sin θ cos θ = 3
[sin2 q + cos­2 q = 1] 2
=
Þ 2 sin q cos q = 3 – 1 = 2 cos A
Þ sin q cos q = 1 ...(i) = 2 sec A = R.H.S.
Now, Hence, proved.

66 Mathematics Class X

Click here to buy other Educart books on Amazon - https://amzn.to/40U8Txx


 ANSWER SHEET

SELF PRACTICE
1. (b) JL From DACB, we have
Explanation: The subject is present at the AC
position L. cot45° =
AB
So, from the figure, we can see that the line of
sight is JL. AC
Þ
5. (c) 12 m 15
Explanation: Let AD be the pole of 16 m and AC
CE be the pole of 10 m and ÐACB = 30° 1=
A 15
AC = 15 m
l
Þ
30°
B C From DEBD, we have
DE
tan30° =
16 m 10 m BE

1 DE
= [ BE = AC = 15 m]
3 15
D E
Then, BD = CE = 10 m 15
AD = AB + BD DE =
3
16 = AB + 10
AB = 6 m
Þ 5 3 m

AB
Now, sin 30° =
\ Height of the tower = EC + DE
AC
1 6 = 15 + 5 3
=
2 AC
= 5(3 + 3 ) m
AC = 12 m
Þ l = 12 m
15. (a) 45°
9. (a) 5( 3 + 3 ) m Explanation: Let AB be the height of a man
and BC be his shadow.
Explanation: Let AB be the building and CD be
the tower. A
Draw BE ^ CD
Then, CE = AB = 15 m,
ÐEBD = 30°
and ÐACB = ÐCBE = 45°
D
θ
C B
Then, AB = BC [Given]
In right–angled DABC,
AB
30° tan q =
B E BC
45°
Þ tan q = 1
Þ tan q = tan 45°
15 m 15 m [ tan 45° = 1]
Þ q = 45°.
45° So, the angle of elevation of the sun is 45°.
A C

Some Applications of Trigonometry 67

Click here to buy other Educart books on Amazon - https://amzn.to/40U8Txx


21. 45° m
In DPQR,
P
Explanation: Let BC be the height of the tower
which is standing on the ground. Let A be a
point on the ground which is 60 m away from
the foot of the tower. h'
C
2
Q x' R

11h
PR h' 10 h
tan q2 = = = =
QR x ' 11x x

45° 10
A 60 m B h
Þ tan q2 =  ...(ii)
Then, AB = 60 m and ÐBAC = 45° x
In right–angled DABC,
From equations (i) and (ii), we get
BC tan q1 = tan q2
tan45° =
AB
Þ q1 = q2
 Perpendicular 
∵tanθ = 
Hence, the required angle of elevation of its top
 Base  remains unchanged.
BC
1= 29. (b) Both assertion (A) and reason (R) are true
60 but reason (R) is not the correct explanation of
BC = 60 m assertion (A).
Hence, the height of the tower is 60 m. Explanation: The height of vertical pole, = AB
Length of pole’s shadow = BC
25. True.
A
Explanation: Let AC be the tower of height h
and the distance of the point of observation
from its foot be x.


In DABC, B C
A
In ∆ABC
AB
tan q =
BC
h
AB
= (AB = BC)
θ1 AB
B x C =1
tan q = tan 45º
AC h q = 45º
tan q1 = = ...(i)
BC x We know that, Pythagoras theorem states that
Now, if the height of the tower is increased by “In a right-angled triangle, the square of the
10%, new height h’ = h + 10% of h. hypotenuse side is equal to the sum of squares
10 h 11h of the other two sides“.
= h + h = h+ =
100 10 10 Hence, both assertion and reason are true
but reason is not the correct explanation of

Distance of point of observation from its foot is
assertion.
also increased by 10%.
10 x 30. (d) Assertion (A) is false but reason (R) is true.
New distance x’ = x +
100 Explanation: Let AC be the ladder of length
20m at the height AB, i.e., h m from the ground.
x 11x
= x+ = The ladder makes an angle of 60º with the
10 10
wall.

68 Mathematics Class X

Click here to buy other Educart books on Amazon - https://amzn.to/40U8Txx


A P
X
30° θ
60°
20 m
h m 240 m

30° θ
C B B Q
B'
\ ∠CAB = 60º
Now in ∆ABC, We have, BB’ = 240 ( 3 − 1)  [Given]
AB base
cos 60º = = and BQ = 240 3  [from part (i)]
AC hypotenuse So, B’Q = BQ – BB’
1 h = 240 3 – 240 ( 3 − 1)
=
2 20 = 240 m

h = 10 m Let q be the new angle of depression of
Hence, assertion is false but reason is true. boat from the top of tower PQ.

32. (B) (d) 30 m


\ ÐXPB’ = ÐPB’Q = q
So, in DPB’Q
Explanation: The distance of the ship from
the base of the light house after 10 seconds PQ
tan q =
from the initial position when the angle of B′Q
depression changes to 45° is BC which can 240
=
be found by finding tan 45° in right–angled 240
triangle ABC, as ship has moved from D to C = 1
in 10 s. = tan 45°
AB 30
Þ q = 45°
tan 45° = 1 = =
BC BC Hence, the new angle of depression of boat
Þ BC = 30 m from the top of the tower is 45°.

34. (C) (a) 32 3 m


(E) (d) 5( 3 + 1)s
Explanation: The distance of the top B of
Distance the building AB from the point P = BP.
Explanation: Time taken =
speed So, in DABP
BC 30 AP
= = cos 30° =
3( 3 − 1) 3( 3 − 1) BP
3 48
Þ =
10( 3 + 1) 10( 3 + 1) 2 BP
= =
( 3 − 1)( 3 + 1) 2
96 96 3
Þ BP = = ×
3 3 3
= 5( 3 + 1)s
96 3
33. (C) Distance of boat from tower = = 32 3 m
3
= 240 3 – 240 ( )
3 −1
(E) (a) 96 m
= 240 m
Explanation: The distance of the top C of

Let the angle of depression = q
the telecom tower BC from the point P = PC.
240 \ In DAPC,
tan q =
=1
240 AP
Þ q = 45° cos 60° =
PC
[CBSE Marking Scheme Term-2 SQP 2022]
1 48
Explanation: Let B’ be the position of boat Þ =
2 PC
after 10 minutes.
Þ PC = 96 m

Some Applications of Trigonometry 69

Click here to buy other Educart books on Amazon - https://amzn.to/40U8Txx


35. (A) PC
37. (A) sin 60º =
PA

3 18
⇒ =
2 PA

⇒ PA = 12 3 m
(B)
Here A is the initial position of the
aeroplane and AB is its vertical height. 38. (C) (b) 120( 3 – 1) m
Similarly, A’ is the position of the Explanation: Height of the building CD = AE
aeroplane after 15 seconds and A’B’ is (since DEAC is a rectangle) = AB – BE
its vertical height.
So, in DBDE,
\ ∠APB = 45°, ∠A’PB’ = 30° and
BE
AB = A’B’ = 2500 m tan 45° =
DE
Now, in DAPB,
AB BE
tan 45° = ⇒ 1= [ DE = AC]
PB 120
⇒ BE = 120
2500
⇒ 1=
PB So, CD = 120 3 – 120 = 120( 3 – 1) m
⇒ PB = 2500 m
(E) (a) 240 m
Also, in DA’PB’

A'B' A'B' Explanation: The distance of the straight


tan 30° = = line joining the top of the tower AB and the
PB' PB + BB'
bottom of the building CD = BC .
1 2500
⇒ = In DABC,
3 2500 + BB'
AC
⇒ 2500 + BB’ = 2500 3 cos 60° =
BC
⇒ BB’ = 2500 ( 3 − 1) m
1 120
So, distance covered by the plane during Þ =
2 BC
the period of observation
Þ BC = 240 m
= A’A = B’B
= 2500( 3 − 1)m 39. Let the height of the tower BA be ‘h’ metres.
In right ∆ABC, we have
6000 AB
36. (C) speed = = tan 60º
30 BC

= 200 m/s h
Þ = 3
30
3600
= 200 ×
1000 Þ h = 30 3 m
= 720 km/hr Hence, the height of the tower is 30 3 metres.

3000( 3 − 1) 40. A
Alternatively: speed =
15( 3 − 1)
= 200 m/s
3600
= 200 ×
1000 60°
C B
= 720 km/hr 2 3
[CBSE Marking Scheme SQP Std. 2022]
Let AB be the pole and BC be its shadow.

70 Mathematics Class X

Click here to buy other Educart books on Amazon - https://amzn.to/40U8Txx



\ In ABC, 57. Let, height of the tower be AB and its shadow
AB be BC.
tan 60° =
BC
\ AB: BC = 3 :1
AB

Þ 3 = AB 3
2 3
⇒ = ...(i)
BC 1
Þ AB = 6 m
Hence, the height of the pole is 6 m. Let angle of elevation of the Sun be q.
47. In right–angled DABC, ÐB = 90° Then, in ∆ABC
Let ÐDAC = q A
Then ÐDAC = ÐACB = q
[alternate angles]
AB 5 3
Now, tanq = =
BC 5
tanq = 3 θ
B C
tanq = tan60°
AB
q = 60° = tan q
Hence, the angle of depression from A is 60º. BC

51. Here, AB is a pole of height 6 m, DC is a ladder which ⇒


3
= tan q [Using (i)]
reaches the pole at point D and AD = 2.54 m. 1
Then, BD = AB – AD = 6 – 2.54 \ tan q = 3
= 3.46 m ⇒ tan q = tan 60°
The angle of elevation from point C to the ⇒ q = 60°.
point D is 60º.
Hence, the angle of elevation of the sun is 60°.
Now, in DBDC,
BD 62. Let W be the position of window in the house
sin 60º = AB and PQ be the other house.
CD
P
3 3.46
Þ =
2 CD
3.46 × 2 6.92 A
Þ CD = =
3 1.73  W
M
= 4 m 
Hence, the length of the ladder is 4 m.
h
52. Let, AC be a ladder placed along a wall AB.
Also, let ‘h’ be the height of the wall. 
Q B
In DABC
A
∴ ∠PWM = a , ∠MWQ = ∠WQB = b
and WB = hm
Now, in DWQB
r
de

h
d
la

WB
tan b =
60° BQ
C 2m B

AB h
⇒ tan b =
tan 60º = BQ
BC
h h
Þ 3 = ⇒ BQ = = h cot β ...(i)
2 tan β
Þ h= 2 3m
 1 
 ∵ cot θ = 
Hence, the height of the wall is 2 3 m.  tan θ

Some Applications of Trigonometry 71

Click here to buy other Educart books on Amazon - https://amzn.to/40U8Txx


Also, in DPWM
PM
tan a = h
WM
PM
⇒ tan a = [∵WM = BQ]
BQ
PM
=  [From (i)]
h cot β
⇒ PM = h tan a cot b Let the height of AE be ‘h’ m.
Now, height of other house, PQ Then the height of the chimney AB
= PM + MQ = AE + BE = (h + 1.5) m
= PM + WB [∵MQ = WB] Now, in the right–angled DAED,
= h tan a cot b + h AE
tan 30º =
= h(tan a cot b + 1) DE
Hence, proved 1 h
⇒ =
67. Let AB be the tree, PB be the river and P’ be the 3 20 3
final position of the person.
⇒ h = 20
A
So, AB = h + 1.5 = 20 + 1.5
= 21.5 m
Hence, the height of the chimney is 21.5 m.
74. Let AB be a building of height 50 m and CD be
30° 60° a tower at a horizontal distance of BC from the
P B
building.
40 m P

\ AB = CE = 50m, ÐDAE = 45° and ÐDBC = 60°
∴ PP’ = 40 m, ∠APB = 60°, ∠AP’B = 30° D

°
So, in DAPB,
45
60°
AB h
tan 60° =
PB A 45°
E
AB
⇒ 3 = 50 m 50 m
PB
⇒ AB = PB 3 ...(i) 60°
B C
Also, in DAP’B
Let DE = ‘h’ m
AB AB
tan 30° = = Then, the height of tower = (h + 50) m
P'B P'P + PB
Now, in the right–angled DDEA,
1 PB 3
⇒ =  [From (i)] DE
3 40 + PB tan 45º =
AE
⇒ 3PB = 40 + PB
h
⇒ 2PB = 40 Þ 1=
AE
⇒ PB = 20 m
Þ AE = h...(i)
Putting PB = 20 in (i), we get
Now, in the right–angled DDBC,
AB = 20 3 m
DC
Hence, width of the river is 20 m and height tan 60º =
BC
of the tree is 20 3 m.
DE + EC
72. Let AB be the chimney and DC be the observer Þ 3 = [ BC = AE]
AE
at a distance of 20 3 m from the chimney.
Þ 3 h = h + 50 [Using (i)]
\ CD = BE = 1.5 m
BC = DE = 20 3 m, ÐADE = 30°. Þ h ( 3 – 1) = 50

72 Mathematics Class X

Click here to buy other Educart books on Amazon - https://amzn.to/40U8Txx


50 ( 3 + 1) 78. B C
Þ h= ×
( 3 – 1) ( 3 + 1)

50
Þ h= ( 3 + 1) 1500 3
2
= 25 × (1.73 + 1) 30°
60°
= 25 × 2.73 A
L M
= 68.25
BL
Hence, the height of the tower, DC Let AL = x \ x = tan 60°
= DE + EC
1500 3
= 68.25 + 50 Þ = 3
x
= 118.25 m
Þ x = 1500 m
and the horizontal distance between the tower
and the building is 68.25 m. CM 1
= tan 30° =
AL + LM3
76. Let BC be the statue, standing on a pedestal
Þ 1500 + LM = 1500(3) = 4500
AB and O be the point of observation on the
ground. Þ LM = 3000 m
3000

\ BC = 1.6 m, ÐAOC = 60° and ÐAOB = 45°. \ Speed =
15
C = 200 m/s
= 720 Km/h
1.6 m
[CBSE Marking Scheme SQP 2015]
B
60° Explanation: Let B and C be the two positions

of the aeroplane.
45°
O A
\ ÐBAL = 60°, ÐCAM = 30°
In right ∆OAB,
and BL = CM = 1500 3 m
AB
= tan 45º = 1 B C
OA
Þ OA = AB ...(i)
In right ∆OAC,
AC
= tan 60º = 3 °
OA 60
30°
AC AB + 1.6
Þ OA = = ...(ii) A L M
3 3
From (i) and (ii), we have Now, in DBAL,

AB + 1.6 BL
tan 60° =
AB = AL
3
⇒ 3 AB = AB + 1.6
1500 3

⇒ 3 =
⇒ AB ( 3 − 1) = 1.6 AL
1.6 3 +1 ⇒ AL = 1500 m  ...(i)
or AB = ×
3 −1 3 +1 Similarly, in DCAM,
CM
1.6 ( 3 + 1) tan 30° =
= AM
2
= 0.8 (1.73 + 1) 1 1500 3

⇒ =
= 0.8 × 2.73 = 2.184 3 AM
Thus, the height of the pedestal is 2.184 m. AM = 4500 m

Some Applications of Trigonometry 73

Click here to buy other Educart books on Amazon - https://amzn.to/40U8Txx


Now, distance covered by aeroplane during the 84. Consider AB as the lighthouse of height 100 m,
flight of 15 seconds = LM C and D be two distance positions of the boat
= AM – AL at an instant, when the angle changes from
= 4500 – 1500
60º to 30º .
= 3000 m
Let the speed of the boat be ‘y’ m/min.
Distance covered Time taken by the boat to reach from D to C = 2 min
So, speed of plane =
Time taken \ Distance, CD = Time × Speed
3000 = 2y
= = 200 m/hr
15 Let: BC = x m
Now, in right DABD,
18
= 200 × km/h
5 AB
tan 30º =
= 720 km/h BD
Hence, the speed of the plane is 720 km/h 1 100
Þ =
80. Let C and E be the two positions of the 3 x + 2y
aeroplane and let A be the point of observation.
Þ x + 2y = 100 3 ...(i)
Let AX be the horizontal ground.
Draw BC ^ AX and ED ^ AX.
Then, ÐCAB = 60º, ÐEAD = 30º
and BC = ED = 3600 3 m
Let AB = ‘x’ m and BD = ‘y’ m
From right DACB, we have:
BC In right DABC,
tan 60º =
AB AB
tan 60º =
3600 3 BC
Þ 3 =
x 100
Þ 3 =
Þ x = 3600 x

Þ x = 100 = 100 3
3 3

Put the value of x in equation (i), we get


100 3
2y = 100 3 –
3
 2
From right DAED, we have Þ 2y = 100 3 ×  
3
ED
tan 30º = 100 3 100 × 1.732
AD Þ y= =
3 3
1 3600 3
Þ = = 57.73
3 x+y
Hence, the speed of the boat is 57.73 m/min.
Þ x + y = 3600 × 3 = 10800
Þ y = 10800 – x 88. Let DF be the tall boy of height 1.6 m and AE
= 10800 – 3600 be the building of height 40 m.
= 7200
In 30 seconds, the distance covered by the
aeroplane is 7200 m.
Distance
Then, speed of aeroplane =
Time
7200
=
30

= 240 m/s
Hence, the speed of the aeroplane is 240 m/s.

74 Mathematics Class X

Click here to buy other Educart books on Amazon - https://amzn.to/40U8Txx


The angle of elevation changes from 30º to 60º Þ AD = 120 m
when the boy moves from point D to point C.
Let the distance of CD be x m. AB
In DBAC, = tan 60°
Here, AE = 40 m, CA
and DF = BE = 1.6 m
120
\ AB = AE – BE Þ = 3
= 40 – 1.6 = 38.4 m CA
Now, inDABD,
120
Þ CA =
AB 3
tan 30º =
BD
1 120 3
38.4 = × = 40 3
= 3 3
Þ 3 BD

Þ BD = 38.4 3 m...(i) Now, CD = CA + AD


In D ABC, = 40 3 + 120
AB
tan 60º =
BC = 40 3(1 + 3 ) m

38.4 Hence, the distance between the two cars is


Þ 3 =
BC 40 3(1 + 3 ) m.
38.4 38.4 3
Þ BC = = × 92. Let AB be a tree on which the bird is sitting at
3 3 3 point A, D be its position after 2 seconds, and C
38.4 3 be the point of observation on the ground.
= = 12.8 3 ...(ii)
3 AB ^ CE

\ Distance between two positions,
CD = BD – BC
= 38.4 3 – 12.8 3

 [using (i) and (ii)]

= 25.6 3 m

Hence, the distance the boy walked towards \ AB = DE = 80 m, ÐACB = 45º and ÐDCE = 30º
the building is 25.6 3 m. Let AD = x m
Then, BE = AD = x m
89. Let AB be the tower of height 120 m. Let C and Now, in DABC,
D be the location of two cars on opposite side
AB
of the tower. tan 45º =
BC
X Y
80
Þ 1=
BC
Þ BC = 80 ...(i)
And in DDCE,
DE
tan 30º =
CE
\ AB = 120 m 1 80
ÐXBC = ÐBCA = 60° Þ =
3 CB + BE
and ÐYBD = ÐBDA = 45°
1 80
AB Þ = 80 + x
In DBAD, = tan 45° 3
AD
Þ 80 + x = 80 3
120
Þ =1 Þ x = 80( 3 − 1)
AD

Some Applications of Trigonometry 75

Click here to buy other Educart books on Amazon - https://amzn.to/40U8Txx


= 80 × (1.732 – 1) QR 1
= 80 × 0.732 = 58.56 tan 30º = = ...(i)
BR 3
Distance covered
Then, speed of bird= From right ∆BRQ’,
Time taken
RQ ′
58.56 tan 60º = = 3 ...(ii)
= = 29.28 m/s BR
2
Eliminating BR from (i) and (ii), we have
Hence, the speed of the bird is 29.28 m/s.
RQ′
93. Let AS be the surface of lake, B be a point of 3 QR =
observation, Q be the cloud and Q’ be its 3
reflection in the lake.
RQ ′
Draw BR ^ QQ′ ⇒ =3
QR
\ AB = RS = 10 m, ÐQBR = 30° and ÐRBQ′ = 60°.
Here, we need to determine the height SQ.
10 + SQ ′
⇒ =3
SQ − 10

[Q SQ’ = RQ’ – 10; SQ = 10 + RQ]

10 + SQ
⇒ = 3 [Q SQ’ = SQ]
SQ − 10

⇒ 3SQ – 30 = 10 + SQ

⇒ 2SQ = 40

⇒ SQ = 20
Here, we have SQ = SQ’. Thus, the height of the cloud from the surface of
From right ∆BRQ, the lake is 20 metres.

76 Mathematics Class X

Click here to buy other Educart books on Amazon - https://amzn.to/40U8Txx


 ANSWER SHEET

SELF PRACTICE
1. (d) 65º 10. (b) 4 cm
Explanation: We know, tangent is perpendicular
Explanation: CA ⊥ AP and CB ⊥ AP

to radius at the point of contact.
[As tangent to a circle is ⊥ to radius]
So, ∠OPT = ∠OQT = 90°
A
In quadrilateral OPTQ,
∠POQ + ∠OPT + ∠OQT + ∠PTQ = 360°
C P
⇒ ∠PTQ = 360° – (90° + 90° + 115°) = 65°

5. (a) 25 cm B
Explanation: PT = 24 cm, OT =7 cm
Also, ∠APB = 90°
Since PT is a tangent to the circle at T.
Now, in quad. APBC
∠PTO = 90°
[tangent is perpendicular to the radius of a ∠APB + ∠CAP + ∠CBP + ∠ACB = 360°
circle]
⇒ 90° + 90° + 90° + ∠ACB = 360°
In ∆PTO,
⇒ ∠ACB = 90°
By Pythagoras theorem, Now, each angle of quadrilateral APBC is 90°
OP2 = PT2 + OT2 and AP = PB.
⇒ OP2 = 242 + 72

\ APCB is a square.
⇒ OP2 = 576 + 49
⇒ OP2 = 625
\ AP = BP = BC = CA = 4 cm
⇒ OP = 25 cm Thus, the length of each tangent is 4 cm.

8. (c) 50° 21. 12 cm


Explanation:
Explanation: Given,
Here, ∠AOB = 100° OP = 5 cm  [radius]
∠OAT = 90° [As tangent at a OQ = 13 cm
P
point to a circle is
5 cm

perpendicular to the radius]


In DOAB, Q
O 13 cm
OA = OB [Radii of the circle]

⇒ ∠OBA = ∠OAB
Now, in DOPQ, ∠P = 90º
[ angle opposite to equal sides are equal]
[Radius is perpendicular to tangent at the
Since, sum of angles in a triangle is 180°.  point of contact]
180° − ∠AOB ∴By Pythagoras theorem,

\ ∠OBA = ∠OAB = (OQ)2 = (OP)2 + (PQ)2
2
180° − 100° ⇒ PQ = ( OQ )2 − ( OP )2
=
2
Now, ∠BAT = ∠OAT – ∠OAB ⇒ PQ = (13)2 − (5)2

= 90° – 40° = 169 − 25 = 144


= 50° = 12 cm

Circles 77

Click here to buy other Educart books on Amazon - https://amzn.to/40U8Txx


24. 8 cm line segment PQ. We know that perpendicular
Explanation: The distance between two
bisectors of a chord of a circle always passes
parallel tangents of a circle is equal to the through the centre of circle.
diameter of the circle.
Thus, perpendicular bisector of PQ passes
through C1 and C2.
4 i.e., C1P = C1Q
C2P = C2Q
4
Similarly, all the circles passing through PQ will
have their centre on perpendicular bisector of
PQ.
Thus, 4 + 4 = 8 cm.
25. 25º 32. True.
Explanation: Join OB.
Explanation:

A C

P 50º O A 30º
O B D

B
Since, APBO is a cyclic quadrilateral,
Join OC.
∠AOB = 180º – ∠APB In DAOC,
= 180º – 50º = 130º OA = OC
In DAOB, OB = OA [Radii] [Radii of the same circle]
Hence, ∠OAB = ∠OBA ∠OCA = ∠OAC = 30°

∠OAB + ∠OBA + ∠AOB = 180º [Angles opposite to equal
sides are equal]
1 Also, ∠ACB = 90° [Angle in a

⇒ ∠OAB = [180º − 130º]
2 semicircle is a right angle]
50° So, in DABC,
= ∠ABC + ∠ACB + ∠CAB = 180°
2
[Angle sum property]
= 25º
∠ABC + 90° + 30° = 180°
29. True. ∠ABC = 180° – 120°
= 60°
Explanation: The angle between two tangents
 ∠ACB = 90°
to a circle may be 0° only when both tangent
[Proved above]
lines coincide or are parallel to each other.
⇒ ∠OCA + ∠OCB = 90°
31. True. ⇒ 30° + ∠OCB = 90°
⇒ ∠OCB = 60° ...(i)
Explanation:

In DBOC,
OC = OB
 [Radii of same circle]
C2
∠OBC = ∠OCB = 60°
C1 [Angles opposite to equal sides are equal]
P Q
Now, ∠OBC + ∠CBD = 180°
[Linear pair]
⇒ 60° + ∠CBD = 180°

Centre of any circle passing through the end So, ∠CBD = 120° ....(ii)
points P and Q of a line segment are equidistant We have, OC ⊥ CD
from P and Q. [Tangent at a point on the circle is

We draw two circles with centres C1 and C2 perpendicular to the radius through
passing through the end points P and Q of a point of contact]

78 Mathematics Class X

Click here to buy other Educart books on Amazon - https://amzn.to/40U8Txx


∴ ∠OCD = 90° 12 17

⇒ ∠OCB + ∠BCD = 90°
(D) (c) cm
17

⇒ 60° + ∠BCD = 90° Explanation: To find the radius of the in

⇒ ∠BCD = 30° ...(iii) circle, join OA, OB and OC.
In ∆BCD, A

∠CBD + ∠BCD + ∠BDC = 180°


 [Angle sum property of triangle]
120° + 30° + ∠BDC = 180° [From (ii) and (iii)] R Q

∠BDC = 30° ...(iv) O


From eqn. (iii) and eqn. (iv)
∠BCD = ∠BDC = 30° B C
P
BD = BC [Sides opposite to
Then, OP ⊥ BC, OQ ⊥ AC and OR ⊥ AB, as a
equal angles are equal]
tangent to a circle is perpendicular to the
Hence, the given statement is true. radius through the point of contact.
Area of triangle,
38. (B) We have, OB = Radius = 5 cm,
OP = 13 cm. 1 1
⇒ 12 17 = (OP × BC) + (OR × AB)
B 2 2
5 cm 1
+ (OQ × AC)
O 13 cm
P 2
1 1 1
=(14 × r) + (9 × r) + (11 × r)
2 2 2
Also, OB ⊥ BP
1
∴ In DOBP, using Pythagoras theorem, ⇒ 12 17 = (34 × r) = 17r
2
OP = OB + BP
2 2 2

12 17
⇒ (13)2 = (5)2 + BP2 ⇒ r= cm
17
⇒ BP2 = 169 – 25 = 144
⇒ BP = 12 cm 40. (D) (c) 3 21 m
Explanation: Triangle ODC is also a
39. (B) (a) 34 cm
right- angled triangle as OD ⊥ CD. [Q
Explanation: Perimeter of triangle CD is a tangent and OD is a radius and
ABC = AB + BC + AC we know that tangent is perpendicular
to radius at the point of contact.]
= AR + RB + BP + PC + AQ + QC
Therefore, applying Pythagoras
Which is equal to AQ + BP + BP + PC + PC + AQ theorem, we get,
= 3 + 6 + 6 + 8 + 8 + 3 = 34 cm OC2 = OD2 + CD2
(C) (b) 12 17 cm2 ⇒ CD2 = OC2 – OD2
So, CD2 = 172 – 102
Explanation: We will use heron’s formula
to find the area of triangle ABC, = 289 – 100 = 189

\ A= s( s − a )( s − b )( s − c ) ⇒ CD = 189 = 3 21 m
(E) (b) ∠POD + ∠PCD = ∠OPC + ∠ODC
a+b+c
where, s = Explanation: In quadrilateral PCDO,

2
∠POD + ∠PCD + ∠OPC + ∠ODC = 360°
Here, a = BC = 14 cm, b = AC = 11 cm and (by angle sum property of quadrilateral).
c = AB = 9 cm.
But, ∠OPC = ∠ODC = 90°, as OP ^ PC and
Therefore, OD ^ CD (tangent at any point of a circle
s = 17 cm is perpendicular to the radius through the
point of contact).
⇒ A = 17 × 3 × 6 × 8
Therefore, ∠POD + ∠PCD = 360° – 180° = 180°
= 12 17 cm2 = ∠OPC + ∠ODC

Circles 79

Click here to buy other Educart books on Amazon - https://amzn.to/40U8Txx


41. (B) (d) 9 cm 42. (A) Extend OA to a point F such that AF = 4 cm
Explanation: as shown below:
CD = CR + RD F
= CQ + DS A
[Q CR = CQ; DR = DS] D
 CQ = BC – BQ
= BC – BP O E P

= 7 – 3 = 4 cm B
C
and DS = 5 cm
CD = 5 + 4 = 9 cm OA ⊥ AE and PB ⊥ AB. Therefore, AF || BP
and AF = BP = 4cm.
(C) (a) 6 cm
So, AFPB is a rectangle and therefore AB =
Explanation: Perimeter of quadrilateral
FP.
ABCD = AB + BC + CD + DA
OFP is a right triangle, right angled at F.
Now, AB = AP + BP = AS + 3; Therefore, by Pythagoras theorem,
BC = 7 cm; OP2 = OF2 + PF2
CD = CR + DR ⇒ 162 = (OA + AF)2 + AB2
= CQ + DS ⇒ 256 = 122 + AB2
= 4 + 5 = 9 cm; ⇒ AB2 = 112
DA = DS + AS = 5 + AS ⇒ AB = 4 7 cm
\ 36 = AS + 3 + 7 + 9 + 5 + AS
45. Given : ∠BOC = 130º
⇒ 2AS = 36 – 24 = 12
Since, AOB is the diameter of the circle.
⇒ AS = 6 cm
Then, ∠AOB = 180º
(E) (c) 11 cm
⇒ ∠BOC + ∠AOC = 180°
Explanation: Given that DS = 5 cm;
⇒ 130º + ∠AOC = 180º
Since DS and DR are tangents from the
same external to the circle, ⇒ ∠AOC = 50º
DS = DR = 5 cm Now, ∠OAC = 90º [Since a tangent at
Since AD = 23 cm, any point on a circle is
AR = AD – DR perpendicular to the radius]
= 23 – 5 = 18 cm.
In ∆OAC,
Similarly, AR and AQ are the tangents from
the same external point to the circle and
∠AOC + ∠OAC + ∠ACO = 180º
hence, ∠ACO = 180º – (90º + 50º)
AR = AQ = 18 cm
= 180º – 140º
Since AB = 29 cm,
BQ = AB – AQ = 40º
= 29 – 18 = 11 cm.
49. DOAP is a right triangle, right-angled at A and
Since CB and AB are the tangents to the
∠APO = 45°
circle, angle OQB is equal to 90°.
OA
Given that angle B is 90° and hence ÐPOQ So, = sin 45°
OP
is equal to 90° and hence, OQBP is a
r 1
square. ⇒ =
OP 2
Since BQ is 11 cm, the side of the square
OQBP is 11 cm.  Perpendicular 
From the figure it is clear that the side of Q sinθ = 
 Hypotenuse 
the square is the radius of the circle and
hence, radius of the circle is 11 cm. ⇒ OP = 2r

80 Mathematics Class X

Click here to buy other Educart books on Amazon - https://amzn.to/40U8Txx


50. Let O be the centre of the concentric circles (Since, angles opposite to equal sides are equal)
and AB be the chord for bigger circle and and ∠OKP = 90º
tangent to the smaller circle. [Since angle between the radius and tangent is
Let P be the point where AB meets smaller a right angle]
circle.
\ OA = a and OP = b ∠PKL = ∠OKP – ∠OKL
Now, ∠OPA = 90° = 90º – 30º = 60º
[As, tangent at any point is perpendicular Hence, the measure of ∠PKL is 60º.
 to the radius]
Now, in DOPA, by Pythagoras theorem 59. Given : An isosceles DABC in which a circle is
OA2 = OP2 + AP2 inscribed.


⇒ AP = a2 − b2
Here, AB = AC and sides BC, AC and AB touch
the circle at P, Q and R, respectively.
Now, AB = 2AP [as perpendicular
 from centre to the chord bisects the chord] To prove : P bisects the side BC.

⇒ AB = 2 a − b 2 2
Proof : Since the tangents drawn from an
53. Given : ∠RPQ = 50° external point to a circle are equal.
Now, ∠OPR = 90°
\ AR = AQ ...(i)
[As tangent makes an angle of 90° with radius] A
⇒ ∠OPQ + ∠QPR = 90°.
∠OPQ = 90° – 50° = 40°
In DOPQ, R Q
OP = OQ [Radii of a circle]
O
∠OQP = ∠OPQ = 40°
B C
So, ∠POQ = 180° – (40° + 40°) P
⇒ ∠POQ = 100° BR = BP ...(ii)
CQ = CP ...(iii)
54. Since tangents drawn from an external point And AB = AC (given)
to a circle are equal.
\ AR + BR = AQ + CQ
\ BP = BD, CD = CQ and AP = AQ = 12 cm
⇒ BR = CQ [Q AR = AQ; from (i)]
Now, the perimeter of ∆ABC = AB + BC + AC ⇒ BP = CP [from (ii) and (iii)]
= AB + (BD + DC) + AC \ P bisects BC.
= (AB + BP) + (CQ + AC)
Hence, proved.
[Q BP = BD & CD = CQ]
= (AB + BP) + (CQ + AC) 61. Given: A circle inscribed in a DABC, such that it
= AP + AQ touches the sides AB, BC and CA at points D, E
= 2AP and F respectively.
= 2 × 12 Also, AB = 12 cm, BC = 8 cm and CA = 10 cm.
C
= 24 cm.

58. Given, a circle with centre O and LN is a diameter.


PQ is a tangent to a circle and ∠KLN = 30º. 10 cm 8 cm
L F E

P
30º
O A B
D
K 12 cm

N Since, the lengths of tangents drawn from an


Q external point to a circle are equal, then:

AD = AF = x (say)
Then, in ∆OKL,
BD = BE = y (say)
OK = OL = radii of the circle CE = CF = z (say)
∴ ∠OLK = ∠OKL = 30º Then, AD + BD = AB

Circles 81

Click here to buy other Educart books on Amazon - https://amzn.to/40U8Txx



⇒ x + y = 12 ...(i) And, PR = RQ ...(i)
Also, BE + EC = BC [Q lengths of tangents from an

⇒ y + z = 8 ...(ii)  external point are equal]
and CF + AF = AC Now, in DOPR

⇒ z + x = 10 ...(iii) PR
cos 60° =
Adding equations (i), (ii) and (iii), we get OR
2(x + y + z) = 30 1 PR

⇒ =

⇒ x + y + z = 15 ...(iv) 2 OR
Subtracting eq. (i) from eq. (iv), we get ⇒ OR = 2PR
z=3 = PR + PR
= PR + RQ [Using (i)]
Subtracting eq. (ii) from eq. (iv), we get
⇒ OR = PR + RQ
x=7 Hence proved.
And, subtracting eq (iii) from eg (iv), we get
y=5 68. Given: Two concentric circles with centre O,

Hence, the lengths of AD, BE and CF are 7 cm, PRT and PQS are tangents to the inner circle.
5 cm and 3 cm, respectively. And, PR = 5 cm
Join OQ
67. Now, PR = PQ  ....(i)
S
P
Q

O R P O

T
∠POR = 90 – 60 = 30°
[Since, tangents from an external point to a
PR 1
⇒ = sin 30° = circle are equal]
OR 2
and ∠PQO = 90º [Since, the tangent
⇒ OR = 2 PR  at any point of a circle is perpendicular
to the radius through the point of contact]
= PR + QR
Now PQS is a chord in outer circle and OQ is
[CBSE Marking Scheme SQP 2015]
perpendicular on this chord.
Detailed Solution :
\ OQ bisects the chord PQS.
Given: ∠PRQ = 120°
[Since, perpendicular from the centre
To prove: OR = PR + RQ
Construction: Join OP and OQ to the chord bisects it]
Then, PQ = QS ....(ii)
P
From (i) and (ii), we get
60° PR = PQ = QS = 5 cm
O R
[Q PR = 5 cm]

Q \ PS = PQ + QS = 5 + 5 = 10 cm
Proof: We have,
Hence, the length of PS is 10 cm.
∠PRQ = 120°
69. Given, a circle, which touches the side BC of
120°

\ ∠ORP = ∠ORQ = = 60° DABC at F and touches AB and AC at D and E
2
respectively.
 [Since, tangents are equally inclined to
the line joining the centre of the circle to their Also, AD = 8 cm
common external point] Since, the lengths of tangents drawn from an
Also, OP ^ PR and OQ ^ QR external point to a circle are equal.
[Q Tangent is perpendicular to radius at Then, BF = BD = x  (say)
 the point of contact] Also, CF = CE = y (say)

82 Mathematics Class X

Click here to buy other Educart books on Amazon - https://amzn.to/40U8Txx


and AD = AE = 8 cm
Join OD.
Now, AB = AD – BD
We know that tangent AC and radius OD at
= 8 – x point D are perpendicular.
Also, AC = AE – CE = 8 – y
\ OD ⊥ AC
and BC = BF + CF = x + y
⇒ AD = DC = 4 cm [Perpendicular
\ Perimeter of DABC
drawn from centre of circle to any chord,
= AB + AC + BC
bisects the chord]
= 8 – x + 8 – y + (x + y)

In right-angled DAOD,
= 8 – x + 8 – y + x + y
OA2 = AD2 + OD2
= 16 cm
Hence, the perimeter of DABC is 16 cm. [By Pythagoras theorem]
⇒ OD = OA2 – AD2
2
72. Given: A circle with centre O and a quadrilateral
ABCD circumscribing it. Also, AD ^ AB and = 52 – 42
OS = 10 cm. = 25 – 16 = 9
C ⇒ OD = 3 cm
27 cm
Hence, radius of the inner circle is OD = 3 cm.
R
77. Join OA and OB.
D
Given : AB is a chord of a circle with centre O.
cm

AB = 16 cm and the radius of the circles


38

S O OA = OB = 10 cm
10 cm
Q Now, OP ⊥ AB
A A
P B
x cm cm
10

8 cm
Construction: Join OP
P O
Now, ∠A = ∠OPA = ∠OSA L

cm
= 90° [since, tangent to a

10
 circle is perpendicular to the B
 radius at point of contact.]
Since, the perpendicular from the centre bisects
Also, AS = AP = 10 cm ...(i) the chord.
[QSides of square are equal]

\ AL = BL = 8 cm
Now, CR and CQ are the tangents from an
external point C. In DOAL, by the Pythagoras theorem,
\ CR = CQ = 27 cm OA2 = AL2 + OL2
\ BQ = CB – CQ
= 38 – 27 = 11 cm OL = OA2 − AL2 = 102 − 82
Now, BP = BQ = 11 cm ...(ii)
 (tangents from an external point B) = 100 − 64 = 36
Then, AB = x = AP + PB
= 6 cm
= 10 + 11
[from (i), & (iii)] Now, ∠OAP = ∠PAL + ∠OAL = 90º
= 21 cm (Since a tangent is perpendicular to the radius
Hence, the value of x is 21 cm.
through the point of contact)
74. Given: Chord AC of circle C2 is a tangent of and ∠PAL + ∠APL = 90º (Using angle sum
circle C1 at point D.  property in DAPL, since, ∠ALP = 90º)
C2
\ ∠PAL + ∠OAL = ∠PAL + ∠APL
C1 ⇒ ∠OAL = ∠APL...(i)
Now, in DAPL and DOAL,
O
m ∠PLA = ∠OLA = 90º
5c
A C
∠APL = ∠OAL [from (i)]
D
\ DAPL ~ DOAL (By AA-similarity)

Circles 83

Click here to buy other Educart books on Amazon - https://amzn.to/40U8Txx


PA AL ⇒ ∠QPR = 180º – 140º = 40º
⇒ =
OA OL Now, ∠PRO = ∠PQO = 90º
(Since, the corresponding sides of similar [Since, a tangent at any point is perpendicular
triangles are proportional) to the radius through the point of contact]
PA 8 ∠OQR = ∠PQO – ∠PQR
⇒ =
10 6 = 90º – 70º = 20º
40 \ ∠QPR = 40° and ∠OQR = 20°.
⇒ PA = cm
3 87. Given: PA and PB are tangents to a given circle.

40 Also, PA = 4 cm and ∠BAC = 135°


Hence, the length of the tangent PA is cm.
3 Now, ∠PAC = 180°
[Q PAC is a straight line]
82. Given: PR and PQ are two tangents drawn
from an external point P to a circle with centre ⇒ ∠PAB + ∠BAC = 180°
O. ⇒ ∠PAB + 135° = 180°
R ∴ ∠PAB = 180° – 135°
= 45°
O P
But, PA = PB = 4 cm [Since tangents
drawn from an external point are equal]
So, in ∆PAB,
Q
∠PBA = ∠PAB = 45°
To prove: QORP is a cyclic quadrilateral
∠PAB + ∠PBA + ∠APB = 180°
Proof: We know that tangent at a point to a [Angle sum property of a triangle]
circle is perpendicular to the radius through ⇒ 45° + 45° + ∠APB = 180°
the point of contact.
⇒ ∠APB = 90°
⇒ OR ⊥ PR and OQ ⊥ PQ ∴ ∆APB is a right triangle.
⇒ ∠ORP = 90° and ∠OQP = 90° By using the Pythagoras theorem in DAPB,
⇒ ∠ORP + ∠OQP = 180° ....(i) AB2 = AP2 + BP2
= 42 + 42

Also, in quadrilateral QORP,
= 16 + 16
∠ORP + ∠OQP + ∠ROQ + ∠QPR = 360°
⇒ AB = 32 = 4 2
⇒ 180° + ∠ROQ + ∠QPR = 360°
[Using (i)] Hence, the length of the chord AB is 4 2 cm.

⇒ ∠ROQ + ∠QPR = 180° 88. Given: AB is a diameter of circle with centre O.


...(ii)
Also MN is a tangent to the circle at point A.

From eqn (i) and (ii), we see that the sum of M
opposite angles in a quadrilateral is 180°.
C
⇒ QORP is a cyclic quadrilateral.
Hence, proved.
A B
Concept Applied E O
 If sum of opposite angles in quadrilateral is 180°, then
quadrilateral is cyclic. D

85. Given, PQ and PR are the tangents to a circle N


with centre O.
To prove: AB bisects all chords which are
Also, ∠PRQ = 70º parallel to MN.
In DPQR, PQ = PR
Construction: Draw a chord CD parallel to
 [Since, lengths of tangents from an tangent MN.
external point are equal]

Proof: MN is a tangent of the circle and OA is
\ ∠PRQ = ∠PQR = 70º its radius.
Also, ∠PQR + ∠PRQ + ∠QPR = 180º ⇒ ∠MAO = 90° [Tangent at any point is
 (By the angle sum property of triangle) perpendicular to radius
⇒ 70º + 70º + ∠QPR = 180º through point of contact]

84 Mathematics Class X

Click here to buy other Educart books on Amazon - https://amzn.to/40U8Txx


Since, CD || MN Length of AP = 8 cm
\ ∠CEO = ∠MAO Since, the tangent to a circle at a point on a
[Corresponding angles] circle is perpendicular to its radius.

⇒ ∠CEO = 90°
\ ∠PAO = 90º


Thus, OE bisects CD. [Perpendicular or DAPO is a right-angled triangle.
from centre to chord bisects the chord] So, using Pythagoras theorem in DPAO,
Similarly, diameter AB bisects all chords which (OP)2 = (AP)2 + (OA)2
are parallel to the tangent at point A. ⇒ OP2 = 82 + 62
Hence proved.
⇒ OP = 64 + 36 = 100 = 10 cm
90. Given: AOB is the diameter of a circle and the Now, BP is a tangent to smaller circle. Then,
tangent at point T meets AB produced at P.
∠PBO = 90º

Also, ∠BAT = 30º
So, in DPBO,
Join OT.
OP2 = BP2 + OB2
T
⇒ (10)2 = BP2 + (4)2

30º ⇒ BP = 100 − 16 = 84
A B P
O
= 2 21 cm

We have, ∠ATB = 90º Hence, the length of BP is 2 21 cm.


[Angle in a semi-circle] 94.
In DTAB,

∠TAB + ∠ABT + ∠ATB = 180º
O
⇒ 90º + 30º + ∠ABT = 180º
1 2
⇒ ∠ABT = 60º A C
F
Now, ABP is a straight line.
D B T

\ ∠ABT + ∠PBT = 180º

B is mid-point of arc (ABC)
⇒ ∠PBT = 180º – 60º = 120º...(i)
\ ∠1 = ∠2
Now in DOBT,
\ ΔOAF ≅ ΔOCF [By SAS]
OB = OT [radii]
\ ∠AFO = ∠CFO = 90°
\ ∠OTB = ∠OBT = ∠ABT = 60º
...(ii) ∠AFO = ∠DBO = 90°

Now, ∠OTP = 90º But these are corresponding angles
[Since, the tangent at any \ AC || DT
point is perpendicular to the radius [CBSE Marking Scheme 2015]
through the point of contact] Detailed solution:
\ ∠OTB + ∠BTP = 90º

Consider a circle with centre O, AC is a chord

⇒ 60º + ∠BTP = 90º [from (i)]
and PQ is tangent on it at the point T, such that

⇒ ∠BTP = 30º ...(iii) point T bisects arc ATC.
Now, in DBTP
∠BTP + ∠TBP + ∠TPB = 180º
30º + 120º + ∠TPB = 180º O

⇒ ∠TPB = 180º – 150º = 30º A C


M
or ∠TPA = 30º P Q
T
Hence, ∠TPA = 30º
92. Given: Two concentric circles of radii 6 cm and AC || PQ
4 cm with centre O. AP is a tangent to the
Join OA, OC and OT.
larger circle and BP is a tangent to the smaller
circle. We have, point T bisects arc ATC

Circles 85

Click here to buy other Educart books on Amazon - https://amzn.to/40U8Txx



\ arc AT = arc TC  [lengths of tangents from

⇒ ∠TOA = ∠TOC an external point are equal]
[Q Equal arcs subtend equal angles at the So, ∆TPQ is an isosceles triangle.
centre.] ...(i) Also, ∠PTO = ∠QTO or OT is a bisector of ∠PTQ
Now, in DAOM and DMOC [since two tangents are equally inclined to the
 line segment joining the centre to that point]
OA = OC [radii]
So, OT ⊥ PQ
∠MOA = ∠MOC [from (i)]  [Angle bisector and
OM = OM [common side]  altitude of isosceles triangles are same]

\ By SAS congruency rule, Then, PR = RQ
DAOM ≅ DCOM  [Since, the perpendicular
 from the centre of a chord bisects it]

\ ∠OMA = ∠OMC[c.p.c.t.]...(ii)
1
So, PR = QR = PQ= 4 cm
Since, AMC is a straight line. 2
In right ∆ORP,

\ ∠OMA + ∠OMC = 180°
OP2 = PR2 + OR2

⇒ ∠OMA = 180° [using (ii)]  [Using Pythagoras theorem]

⇒ ∠OMA = 90° ...(iii) ⇒ 52 = 42 + OR2
Also, ∠OTP = 90° ⇒ OR2 = 25 – 16 = 9
⇒ OR = 3 cm
[Since, tangent is perpendicular to the radius at
Let PT = x
the point of contact] ...(iv)
Then, in ∆PRT
From (iii) and (iv), we get
TP2 = PR2 + RT2
∠OMA = ∠OTP ⇒ x2 = 42 + RT2
As these angles form a pair of corresponding ⇒ x2 = 16 + RT2...(i)
angles between the lines AC and PQ, Since, TP is a tangent and we know that

\ AC || PQ tangent is perpendicular to radius at the point
of contact.
Hence, proved. \ OP ⊥ PT
A In ∆POT,
95.
OT2 = OP2 + PT2
⇒ OT2 = 52 + x2
P O
⇒ (OR + RT)2 = 25 + x2
⇒ (3 + RT)2 = 25 + x2
B
⇒ 9 + RT2 + 2(3)RT = 25 + x2

Consider a circle with centre O in which
tangents PA and PB are drawn to it from an If we put the value of x2 from equation (i), we
external point P. get
9 + RT2 + 6RT = 25 + 16 + RT2
To prove: ∠APB + ∠AOB = 180°
⇒ 6RT = 32
Proof: We know tangent is perpendicular to the
radius at the point of contact. 16
⇒ RT =
3

\ OA ^ AP and OB ^ PB

⇒ ∠OAP = 90° and ∠OBP = 90° 16
Putting, RT = in equation (i), we get
3
Now, in quadrilateral AOBP, we have
2
∠OAP + ∠APB + ∠OBP + ∠AOB = 360°  16 
x2 = 16 +  
[Angle sum property of quadrilateral]  3


⇒ 90° + ∠APB + 90° + ∠AOB = 360° 16 × 9 + 256 400
⇒ x2 = =

⇒ ∠APB + ∠AOB = 180° 9 9
20
100. Given, a circle with centre O and radius 5 cm. ⇒ x=
3
PQ is a chord of length 8 cm.
20
In ∆TPQ, Hence, TP = x = cm.
3
TP = TQ

86 Mathematics Class X

Click here to buy other Educart books on Amazon - https://amzn.to/40U8Txx


101. Given: BC = a, CA = b and AB = c are sides of ∴ DADB ≅ ∆ADC
∆ABC. [By SAS congruence criterion]
To prove: BD = s – b ⇒ ∠ADB = ∠ADC [cpct]
Proof: But, ∠ADB + ∠ADC = 180° [Linear pair]
A ⇒ ∠ADB = ∠ADC = 90°
We know that perpendicular from centre of
circle bisects the chord.
So, OA is perpendicular bisector of BC.
F E Let AD = x
Q OA = radius = 9 cm
∴ OD = OA – AD = (9 – x) cm
In right-angled triangle ODC,
B D C OC2 = OD2 + DC2

We know that tangents drawn from an external [By Pythagoras theorem]
point are equal in length. ⇒ DC2 = OC2 – OD2
\ AF = AE, BD = BF and CD = CE ...(i) = 92 – (9 – x)2...(i)
We have, In right-angled ∆ADC,
a+b+c AC2 = AD2 + DC2
s =
2 [Pythagoras theorem]
⇒ DC2 = 62 – x2 ...(ii)
[As s is semiperimeter, s =
AB + BC + CA
] From eqn (i) and (ii), we get
2
⇒ a + b + c = 2s...(ii) 62 – x2 = 92 – (9 – x)2
Now, BC + CA + AB = a + b + c ⇒ 36 – x2 = 81 – (81 + x2 – 18x)
⇒ 36 – x2 = 81 – 81 – x2 + 18x
(BD + DC) + (CE + EA) + (AF + FB) = a + b + c
⇒ 18x = 36
⇒ 2(BD + CE + AE) = 2s [using eqn (i) and (ii)]
⇒ x=2
⇒ BD + CE + AE = s
∴ AD = 2 cm
⇒ BD = s – (CE + AE)
From eqn (i)
Also, AE + EC = b
DC2 = 92 – (9 – 2)2
⇒ BD = (s – b)
= 81 – 49 = 32
Hence, proved.
⇒ DC = 4 2 cm
105. Given: Isosceles triangle ABC with
AB = AC = 6 cm ∴ BC = 2 DC = 8 2 cm
Radius of circle = 9 cm 1
Area of DABC = × Base × Height
Join OB, OC and OA. 2
Let OA intersect BC at point D. 1
A 6c = × BC × AD
6c
m m 2
B 1 2 C
11
9 D = ×
× 8 2 ×× 22
cm 22
O
= 8 2 cm2

2
Hence, the required area of DABC is 8 2 cm .


In ∆ABO and ∆ACO
106. Given: Two circles with centres O and O’.
AB = AC [Given] Common tangents, AB and CD to these two
OB = OC [Radii of same circle] circles intersect at E.
AO = AO [Common] To prove: O, E, O’ are collinear.
⇒ ∆ABO ≅ ∆ACO
Construction: Join AO, OC, O’D and O’B
[By SSS congruence criterion]
A
⇒ ∠1 = ∠2 [cpct] D
Now in ∆ABD and ∆ACD,
O O'
AB = AC [Given] E
∠1 = ∠2 [Proved above] B
AD = AD [Common] C

Circles 87

Click here to buy other Educart books on Amazon - https://amzn.to/40U8Txx


Proof: In ∆OAE and OCE, Let ∠PTQ = q
OA = OC[Radii of same circle] TPQ is an isosceles triangle.
OE = OE [Common] ∠TPQ = ∠TQP
EA = EC [Tangents from 1
external point are equal in length] = (180° – q)
2
∴ ∆EOA ≅ ∆EOC [SSS congruence θ
criterion] = 90° –
2
⇒ ∠OEA = ∠OEC ∠OPT = 90°

i.e., OE is angle bisector of ∠AEC. ∠OPQ = ∠OPT – ∠TPQ

Similarly, O’E is angle bisector of ∠DEB.  θ θ
= 90° –  90° −  =
⇒ ∠AEC = 2∠AEO...(i)  2 2

and ∠DEB = 2∠O’ED...(ii) 1
∠OPQ = ∠PTQ
2

Now, CD is a straight line.
2∠OPQ = ∠PTQ
∠CEA + ∠AED = 180°
∴ [Linear pair]
[CBSE Marking Scheme Term-2 SQP 2022]

⇒ 2∠AEO = 180° – ∠AED
[Using eqn (i)] Given: Tangents TP and TQ are drawn to a
circle with centre O from an external point T.
1

∠AEO = 90° – ∠AED...(iii) P
2

Similarly, AB is a straight line.
∠AED + ∠DEB = 180°
∴ [Linear pair]
T O
2∠O’ED = 180° – ∠DEA
[Using eqn (ii)]
1 Q
∠O’ED = 90° – ∠AED ...(iv)
2
To prove:
∠PTQ = 2∠OPQ

Now, Proof: We have,


∠AEO + ∠AED + ∠O’ED ∠OPT = 90°
1 1 [∵ Tangent ⊥ Radius]

= 90° – ∠AED + ∠AED + 90° – ∠AED
2 2
⇒ ∠OPQ + ∠TPQ = 90°

[Using eqn (iii) and (iv)]


⇒ ∠OPQ = 90° – ∠TPQ...(i)
Now, we know, tangents drawn from an

= 90° + 90° = 180°
external point to a circle are equal in length.
⇒ ∠AEO + ∠AED + ∠O’ED = 180°
So, in DTPQ,

So, O, E and O’ lie on the same line
TP = TQ
i.e., O, E and O’ are collinear.

⇒ ∠TQP = ∠TPQ
Hence, proved.  [Equal angles opposite to equal sides]
108.

∴ ∠PTQ = 180° – ∠TQP – ∠TPQ
= 180° – 2∠TPQ
= 2(90° – ∠TPQ)

⇒ ∠PTQ = 2(∠OPQ) [From (i)]
Hence, proved.

88 Mathematics Class X

Click here to buy other Educart books on Amazon - https://amzn.to/40U8Txx


 ANSWER SHEET

SELF PRACTICE
5. (a) 10 m
Explanation: Let the radius of the new park be R
35 m
R m. 7m

According to the given condition : r

Area of new park = Area of park I + Area of



park II
2 2
Area of path = p(R2 – r2)
 16   12 

Þ pr2 = π   + π   22
 2  2 = [(42)2 – (35)2]
7
= p(8)2 + p(6)2 22
= 64p + 36p = (42 – 35) (42 + 35)
7

Þ pR2 = 100p [Q a2 – b2 = (a – b) (a + b)]

Þ R = 100
2
22
= × 7 × 77

Þ R = 10 m 7
= 1694 m2
12. (d) 9p cm2
Explanation: 20. (c) 28 cm
A B Explanation: According to the given condition,
Circumference of required circle
= Sum of circumferences of two circles

Let R be the radius of the required circle.

Then, 2πR = 2πr1 + 2πr2
D C  36   20 
= 2π   + 2π  

It is given that side of square = 6 cm  2  2
⇒ Diameter of the circle inscribed in a square, R = 36π + 20π

 = Side of square = 6 cm ⇒ 2πR = 56π
6 ⇒ R = 28 cm
∴ Radius of circle (r) = = 3 cm
2
26. (a) p
\ Area of circle = πr2 = π(3)2 = 9π cm2 Explanation : Raman draws first circle of
diameter 6 units.
13. (c) 33p cm2
Explanation : The outer and inner radii of the
 34   32 
ring are   cm and   cm i.e., 17 cm and
2  2
16 cm. P
O
Q

Now, Area of the circular ring


= p [172 – 162] cm2

= 33p cm2
where, AB = 6 units
16. (c) 1694 m2 radius, AO = OB = 3 units
Raman draws another circle by increasing the
Explanation: Radius of outer circle
radius of the first circle by 4 units.
= (35 + 7) m = 42 m Now, diameter of new circle = 3 + 4 = 7 units

Areas Related to Circles 89

Click here to buy other Educart books on Amazon - https://amzn.to/40U8Txx


Now, number of rounds made by wheel to
cover 2 km
Total distance covered by wheel
=
C D Circumference of wheel

2km (2 × 1000 )m
= =
2m 2m

CD = 7 units = 1000
Circumference of new circle = 2πr
Diameter = 2r 42. (b) Both assertion (A) and reason (R) are true
2πr but reason (R) is not the correct explanation
= =π of assertion (A).
2r
Explanation: circumference – Diameter = 5 cm
30. (b) R12 + R22 = R2 We know, circumference = 2pr
Explanation : According to the given condition, and, diameter = 2r
Therefore, 2pr – 2r = 5

Area of circle with radius R = Area of circle with
2r(p – 1) = 5
radius R1 + Area of circle with radius R2
 22 
⇒ πR2 = πR12 + πR22 2r  − 1 = 5
 7 
⇒ R2 = R12 + R22
5 ×7
34. 4 units r=
2 ×15
Explanation: Given, 2pr = pr2
= 1.166 cm
⇒ 2r = r2 And, we know that, area of circle = pr2
⇒ r = 2 units Hence, both assertion and reason are true
So, Diameter = 2r = 2 × 2 = 4 units. but reason is not the correct explanation of
assertion.
36. 88 cm
43. (B) Pizza (II) is cut into eight equal slices, so
Explanation: Let r be the radius of the circle, angle subtended by each slice at the centre
Now, Area = 616 cm2 [Given] 360°
q2 = = 45°
⇒ pr2 = 616 8
22 2 12
⇒ × r = 616 Also, radius of each slice (r) =
7 2
616 × 7 = 6 inches
⇒ r2 = = 28 × 7 Perimeter of a slice of pizza II
22
= l2 + 2r
⇒ r2 = 196 θ
⇒ r = 14 cm = 2 2πr + 2r
360
Circumference = 2pr
45
= ×2× π ×6 +2×6
22 360
= 2× × 14 = 88 cm
7

38. 1000. = (1.5p + 12) inches
7 (C) The angle subtended by a slice in pizza I is
Explanation : Given: Diameter of wheel = m
11 60°, and the angle subtended by a slice in
7
\ Radius of wheel (r) = m pizza II is 45°.
22
Distance covered by wheel in one round = A1 : A2
= Circumference of wheel θ1 θ2
= πr 2 : πr 2
= 2pr 360 360
= q1 : q2
22 7
= 2 × ×
7 22 Ratio of area as of a slice in the two cases is
= 2 m = 60 : 45 = 4 : 3

90 Mathematics Class X

Click here to buy other Educart books on Amazon - https://amzn.to/40U8Txx


44. (A) (b) 56 cm AM
To calculate AM, we will use cos 45° =
Explanation: It is given that the diameter OA
of the inner most circle C1 is 14 cm. 1 AM
\ Therefore, its radius (r1) = 7 cm. ⇒ =
2 4
It is given that radius of the next circle is
double the radius of the preceding inner 4
⇒ AM = =2 2m
circle. 2
So, radius of circle C2(r2) = 2 × 7 = 14 cm
Similarly, radii of the next circles C3 and C4 Therefore, AB = 2 × 2 2 = 4 2 m.
are 28 and 56 cm respectively. Hence, perimeter of cross-section
(D) (c) 147p cm2
Explanation: The area of the dotted
region between the circles C1 and C2
= 6π + 4 2 m ( )
= πr 2 − πr 2 51. Yes.
2 1
We know that,
(
= p r22 − r12 ) Distance covered in one revolution
= Circumference of the wheel
= p[(14)2 – (7)2] = 2πr
= p (14 – 7)(14 + 7) Where, r is radius of the wheel.
= 147p cm2.
45. (A) Area of minor segment = Area of sector
AOB – Area of DOAB

θ 1
= × πr 2 − × OA × OB
360 2
Let’s assume total revolutions made by the
wheel to cover a distance s is y.
90 1
= ×π×4×4− ×4×4
So, y × 2πr = s
360 2
= 4p – 8 s
\ y=
= 4(p – 2) m2 2πr


(C) Perimeter of cross-section = Length of 54. Given: Radius of circle, r = 28 cm
major arc ACB + Length of chord AB

and central angle, θ = 45º
Length of major arc ACB
θ
θ 270 ∴ Area of sector = × pr2
= × 2πr = × 2π × 4 = 6π m 360
360 360
45 22
θ 270 = × × 28 × 28
× 2πr== × 2π × 4 = 6π m 360 7
360 360
= 11 × 28
θ 270
× 2πr = × 2π × 4== 6π m = 308 cm2
360 360

Hence, the required area of sector of the circle
Length of chord AB = 2 × AM
is 308 cm2.
 [Perpendicular drawn from the centre
to the chord, bisects the chord] 55. Radius of the circle (r) = 21 cm
In ∆AOB, it is given that ÐAOB = 90°. Central angle (q) = 60º
Also, OA = OB = radius, so DAOB is a
right-angled, isosceles triangle. Therefore,
ÐOAB = 45°. O
OM
\ sin 45° =
OA
1 OM
⇒ =
2 4
Area of sector OAPB
4 θ
⇒ OM = =2 2 = × πr 2
2 360

Areas Related to Circles 91

Click here to buy other Educart books on Amazon - https://amzn.to/40U8Txx


60 22 90º
= × × 21 × 21 = × π(2)2
360 7 360º
= 11 × 21
=p
= 231 cm2
22
62. Let the radius of the circle be ‘r’ cm.   = cm2
7

72. Given: Two concentric circles of radii 7 cm and


14 cm.
i.e., OB = OD = r = 7 cm
and OA = OC = R = 14 cm
Also, ÐAOC = 60º
We know, Now, Area of the shaded region
θ  = Area of sector OAC – Area of sector OBD
Length of arc = × 2πr
360° θ θ
= πR 2 − πr 2
60 22 360 360

\ AB = ×2× ×r
360 7 θ
= π (R2 – r2)
1 22 360
⇒ 22 = × × r
3 7 60 22
= × [(14)2 – (7)2]
⇒ r = 21 cm 360 7
Hence, the radius of the circle is 21 cm. 1 22
= × × (14 – 7) × (14 + 7)
66. No, this statement is true. 6 7

Let C1 and C2 be the two cirles with radii r and [Q a2 – b2 = (a – b) (a + b)]
2r respectively. 1 11
= × × 7 × 21
According to the questions. 3 7
Arc length of C1 = Arc length of C2 = 11 × 7

i.e., AB = CD...(i)
= 77 cm2
Hence, the area of shaded region is 77 cm2.
74. Given: Radius of wheel, r = 35 cm

Speed of wheel, s = 66 km/h
 As 
66 ×1000  1 km = 1000 m 
d= m / min  
60

Let θ1 be the angle subtended by arc AB and and1 hr = 60 min
θ2 be the angle subtended by arc CD at the

= 1100 m/min = 1,10,000 cm/min
centre.
We have, Circumference of wheel = 2πr
θ θ 22
1 × 2πr = 2 × 2π(2r ) [Using (i)] = 2 × × 35 = 220 cm
360 360  7
⇒ θ1 = 2θ2 Distance

We know that, Speed =

Hence, angle of corresponding sector of C1 is Time
double the angle of the corresponding sector
2πrn
of C2. s=
t
68. We know that a minute hand of a clock sweeps st
an angle of 6° in one minute. ⇒ n=
2πr
So, it will sweep an angle of 90° in 15 minutes. where, n is the total number of revolutions
So, area swept by the minute-hand in 15 made by the wheel.
minutes ∴ Number of revolutions in 1 min,

92 Mathematics Class X

Click here to buy other Educart books on Amazon - https://amzn.to/40U8Txx


1,10, 000
According to the question,
n= = 500
220 ⇒ p(x + 7)2 – px2 = 1078

Hence, the required number of revolutions per ⇒ p[x + 49 + 14x – x2] = 1078
2

minute is 500. 22
⇒ × [14x + 49] = 1078
7
76.
1078
⇒ (2x + 7) =
22
⇒ 2x = 49 – 7
⇒ 2x = 42
⇒ x = 21

Hence, the radius of the smaller circle is 21 cm.

81. r = 14 cm, q = 60°



The area of the shaded region is equal to the Area of minor segment
sum of areas of three sectors of same radii but
of different angle, say q1, q2 and q3. θ 1 2
= πr2 × – r sin q
360 2
But, q1 + q2 + q3 = 180°

[Angle sum property of Δ] 22 60 1 3
= × 14 × 14 × – × 14 × 14 ×

Given: Radii of each arc, r = 14 cm. 7 360 2 2

\ Area of sector with central angle q1  308 
=  − 49 3  cm2 or 17.89 cm2
θ1 2 2  3 
= × π(14 ) cm  or 17.9 cm2 (Approx).
360

Area of sector with central angle q2  308 
Area of major segment = πr2 –  − 49 3 
θ2  3 
= × π(14 )2 cm2  1540 
360 =  − 49 3  cm or 598.10 cm
2 2
 3 

Area of sector with central angle q3 or 598 cm2 (Approx).
∠R θ [CBSE Marking Scheme SQP 2015]
== 3 × π(14 )2 cm2
360 360 83. (A) Perimeter of 1st figure = p(7) + AB
⇒ Sum of areas of 3 sectors
22
θ1 θ2 = × 7 + 14
= × p(14) +2
× p(14) 2
7
360 360
= 22 + 14 = 36 cm
θ3
+ × p(14) 2
Perimeter of 2nd figure = 2 × Circumference
 360 of a semi-circle of diameter 7 cm + AB
π
= × (14 )2 θ + θ + θ  = 2 ×
1
pd + AB
360  1 2 3
2
π
= × 196 × 180 [Using (i)] = pd + AB
360 
22
22 = × 7 + 14
= 98π = 98 × = 308 cm2 7
7
= 22 + 14

Hence, the area of shaded region is 308 cm2.
= 36 cm
77. Let, the radii of smaller circle be x cm. Perimeter of 3rd figure = 3 × Circumference

Then, the radii of larger circle is (x + 7) cm. 14

Area of smaller circle = px2 of a semi-circle of diameter cm + AB
3
[∵ Area of circle = pr2] 1
= 3 × pd + AB

Area of larger circle = p(x + 7)2 2

Areas Related to Circles 93

Click here to buy other Educart books on Amazon - https://amzn.to/40U8Txx


3 22 14 85. Given: Diameter of pond, d = 17.5 m
= × × + 14
2 7 3 d 17.5
∴ Radius of pond, r1 = = = 8.75 m
= 22 + 14 2 2

= 36 cm
Perimeter of 4th figure = 4 × Circumference
14 7
of a semi-circle of diameter cm i.e., cm
4 2

+ AB

1
= 4 × pd + AB
2
Width of path around the pond = 2 m

Radius of the path including the pond,
= 2pd + AB
r2 = 8.75 + 2 = 10.75 m
22 7
= 2 × × + 14 Now, Area of circular path
7 2
= Area of the pond including the path
= 22 + 14 – Area of the pond
= 36 cm = πr22 – πr12
14 = π[(10.75)2 – (8.75)2]
Here, radius = = 7 cm
2 = π[(10.75 – 8.75) (10.75 + 8.75)]
π 2 [∵ (a2 – b2) = (a – b)(a + b)]
(B) \ Area of 1st figure = (7) sq. cm
2
= π × 2 × 19.5
= 77 sq. cm
= 3.14 × 2 × 19.5
7 = 122.46
Here, radius of each semi-circle = = 3.5 cm
2

Since, cost of constructing the path per square
π  metre = ` 25
\ Area of 2 nd
figure = 2  (3.5)2 sq. cm
2 
Total cost of constructing the path of 122.46 m2
77 area = ` (25 × 122.46) = ` 3061.50
= sq. cm
2
Hence, the required cost is ` 3061.50.

14 88. Given, OP = PQ = 10 cm
Here, radius of each semi-circle = cm Arc PAQ is a part of a circle with centre O and
6
radius (r) = 10 cm
7
= cm PBQ is a semi-circle drawn on the side PQ, with
3
radius (r’) = 5 cm
 2 Now, in DOPQ,
π  7
\ Area of 3rd figure = 3     OP = PQ (given)
 2  3  
and OP = OQ = radii of same circle.
77 \ DOPQ is an equilateral triangle.
= sq. cm
2 Then, ÐPOQ = 60º
14
Here, radius of each semi-circle = cm Now,
8
Area of the shaded region = Area of the semi-
7  circle PBQ – (Area of sector OPAQ
= cm
4 – Area of D OPQ)
 2 2  
 
\ Area of 4th figure = 4  π  7   = π( r ′ ) −  θ × πr 2 − 3 × (OP)2 
 2  4   2  360 4 

77 π  60 3 
= × (5)2 −  × π × (10 )2 − × (10 )2 
= sq. cm
4 2  360 4 

94 Mathematics Class X

Click here to buy other Educart books on Amazon - https://amzn.to/40U8Txx


25π 100π 3 And, Area of major segment = Area of circle
= − + × 100 – Area of minor segment
2 6 4
= pr2 – 3.57
25π 50π 2
= − + 25 3 22  5 
2 3 = × − 3.57
7  2 
 75π − 100π  = 39.28 – 3.57
= 25 3 +   = 35.71 cm2
 6
So, difference of the areas of the two segments =
25π Area of major segment – Area of minor segment
= 25 3 − = 35.71 – 3.57
6
= 32.14
 π Hence, the required difference of the areas of
= 25  3 −  cm2
 6 two segments of the circles is 32.14 cm2.

Hence , the area of shaded region is


92. Given: Let two circles of radii 7 cm and 21 cm
have centres O1 and O2 respectively. Let AB
 π be the arc of first circle and CD be the arc of
25  3 −  cm2.
 6 second circle with central angles θ1 = 120º and
Hence, proved. θ2 = 40º respectively.

90. Consider a circle with centre O and radius r in


which AB is a chord of length 5 cm.

21
cm
r1 = 7 cm r2 = 21 cm
q1 = 120º q2 = 40º
Area of sector with central angle 120°
Then, OA = OB = r cm and AB = 5 cm. θ
= 1 × πr 2

Now, angle subtended by AB at the centre of 360 1

the circle, ÐAOB = 90°
120 22

\ Triangle AOB is a right-angled triangle. = × ×(7)2
360 7

So, in ∆AOB, we have
1
= × 22 × 7
(AB)2 = (OA)2 + (OB)2 3
[Pythagoras theorem] 154
= cm2

⇒ (5) = (OA)2 + (OB)2
2
3


⇒ 25 = r2 + r2 Area of sector with central angle 40°


⇒ 25 = 2r2 θ 2
2
= × πr
5 360 2
⇒ r= cm
cm
2
40 22
Now, Area of minor segment = × ×(21)2
360 7
= Area of sector OAB – Area of DOAB
1
=
× 22 × 3 × 21
θ 1 9
= πr 2 − × OA × OB
360 2 = 22 × 7 = 154 cm2
2 Corresponding arc length of sector with central
90 22  5  1 5 5
= × ×  − × × angle 120°
360 7  2  2 2 2
θ
1 11 25 25 = 1 × 2πr
= × × − 360 1
2 7 2 4
= 9.82 – 6.25 120 22 44
= × 2 × ×7 = cm
= 3.57 cm2 360 7 3

Areas Related to Circles 95

Click here to buy other Educart books on Amazon - https://amzn.to/40U8Txx


Corresponding arc length of sector with central
angle 40°

θ
= 2 × 2π r
360 2

40 22 44
= × 2 × × 21 = cm θ
360 7 3 Area of minor sector = × πr 2
360
Hence, for a circle with radius 7 cm and central
angle 120º, 120 22
= × × 21 × 21
360 7
154 2 2
Area of sector is cm
cm and arc length is
3 = 22 × 21 = 462 cm2
44 Area of major sector
cm
cm.
3 = Area of circle – Area of
And for a circle with radius 21 cm and central minor sector
angle 40º = pr2 – 462
Area of sector is 154 cm2 and arc length is 22
44 = × 21 × 21 – 462
cm
cm. 7
3
= 22 × 3 × 21 – 462
Hence, we observe that the arc lengths of the = 1386 – 462 = 924 cm2
two sectors are equal but their areas are not
Difference of the areas of major sector and
equal.
minor sector = (924 – 462) = 462 cm2
93. Given: Radius of circle, r = 21 cm Hence, the difference in areas of the two
Central angle, θ = 120º sectors of the given circle is 462 cm2.

96 Mathematics Class X

Click here to buy other Educart books on Amazon - https://amzn.to/40U8Txx


 ANSWER SHEET

SELF PRACTICE
1. (a) a sphere and a cylinder = 142296
Explanation: The top part of a surahi is Hence, the cube can accomodate 142296
cylindrical in shape and the bottom part number of marbles.
is spherical in shape. Therefore, surahi is a
combination of sphere and a cylinder. Concept Applied
 If we divide the total volume filled by marbles in a
cube by the volume of a marble, we get required
= + number of marbles.

= Sphere + Cylinder 5. (a) 0.36 cm3


Explanation:
4. (a) 142296
Capacity of capsule = Volume of 2 hemispherical
Explanation: Given, internal edge of cube part + volume of cylindrical part
= 22 cm Length of capsule = 2 cm
\ Volume of cube = (Side)3 Diameter of capsule = 0.5 cm
= (22)3 = 10648 cm3 \ Radius of cylinder, r = Radius of hemisphere
Let the radius of spherical marble be r. 0.5
Radius of capsule = = 0.25 cm
\ Diameter of the marble = 0.5 cm 2
\ Radius of marble,
0.5
r= = 0.25 cm
2

4 3
\ Volume of 1 marble = pr
3

4 22
= × × ( 0.25)3 Length of entire capsule = Radius of 2
3 7 hemispherical parts + Height of cylindrical part
1.375 3 ⇒ 2 = 2r + h
= cm
21 ⇒ h = 2 – 2r
= 2 – (2 × 0.25 )
1 7
As part of the cube remains unfilled, only = 1.5 cm
8 8
part of cube remains filled. Now, Volume of capsule
= Volume of cylindrical part + Volume of two
7
\ Volume of filled cube = × Volume of cube  hemispheres
8
2
= pr2h + 2 × × pr3
7 3
= × 10648
8
22 2 22 3
= ×(0.25)2 ×1.5 + 2× × × (0.25)
= 7 × 1331 7 3 7
= 9317 cm3
22 ( 2 4 
\ Required number of marbles = × 0.25) 1.5 + × 0.25
7  3 
Total space filled by marbles
= 22 × 0.0625
Volume of 1 marble = 1.5 + 0.33
7
9317 9317 × 21
= = = 0.36 cm3
1.375/21 1.375

Surface Areas and Volumes 97

Click here to buy other Educart books on Amazon - https://amzn.to/40U8Txx


9. (a) 2 : 1 bases, we get a sphere of the same radius.
Explanation: Since, the radius of cone and Curved surface area of hemisphere
cylinder are equal i.e., r(say). = 2pr2
Also, prl = 2prh [Given] Curved surface area of sphere = 2pr2 + 2pr2
⇒ l = 2 ⋅= 2 : 1
2
= 4pr
h 1
22. False.
15. (b) 9 : 4 Explanation: As the ball is exactly fitted into
Explanation: Given that: the cubical box of side a,

h1 = h2 ⇒ Diameter of ball = Edge length of cube


⇒ 2r = a
r 3
and 1 = a
r 2 r =
2
⇒ 2
3 4 3
Þ r1 = r2 Volume of sphere = πr
2 3
The ratio of their volume 4 a3
3
4  a
1 2 = π   = 
v πr h 3  2 3 8
= 3
1 1 1
Þ 3
v 1 2 πa
2 πr h =
3 2 2 6

1 3 
2 27. (a) Both assertion (A) and reason (R) are true
π r  h and reason (R) is the correct explanation of
 2
= 3 2 assertion (A).
1 2
π( r ) h Explanation: We have,
3 2
4 3
 3 9
2 πR
=   = 3 64
=
 2 4 4 3 125
πr
3
18. 4 : 9 R3 64
=
Explanation: Let r and R be the radii of the two r 3 125
spheres
R 4
Then, Ratio of their volumes =
r 5
8
= [Given]
27  4 πR2
Ratio of surface area =
4 πr 2
4
π 3 3 2
3 r 2 R2 4
Þ = 2 =  
4  R  =  3  r 5
π
3
16
r 2 =
Þ = 25
R 3
Hence, both assertion and reason are true and
Now ratio of their surface areas reason is the correct explanation of assertion.
2 2
4πr 2 r 2 29. (B) (c) pr (2h + l) sq. units
= =   =  
4πR2 R  3 Explanation: TSA of tent = CSA of cylindrical

portion + CSA of conical portion
4 = 2prh + prl sq. units
= i.e., 4 : 9
9
= pr (2h + l)
21. False. where l = slant height of the cone and is
Explanation: When two hemispheres of equal
base radius are joined together along their
given by l = r 2 + h2

98 Mathematics Class X

Click here to buy other Educart books on Amazon - https://amzn.to/40U8Txx


(D) (a) 4 : 1 \ Area (space) occupied by each student
Explanation: Required ratio
1 154
= pr2 h : pr2 h’ = = 38.5 m2
3 4
32. (C)
Useful canvas = 1650 – 30
h'
= h : =4: 3 = 1650 m2
3 3 Cost of 1m2 canvas = ₹ 200
= 4 : 1 Therefore, cost of canvas bought for the tent
= (1620 × 200)
30. (C) It is given that the number of coins in the
stack = 40. = ₹ 3,24,000
33. (B) Volume of cylindrical part = pr2h
As height of one coin = 0.5 cm, therefore,
height of 40 coins stacked one above the 2
Volume of hemispherical part = πr 3
other = 0.5 × 40 = 20 cm. 3
\ h = 20 cm and r = 1 cm.
πr 2h 3h
22 ∴ Required ratio = =
\ TSA = 2pr(r + h) = 2 × × 1 × (1 + 20 ) 2 3 2r
7 πr
3
22
=2× × 21 = 132 cm2. 3 × 140 6
7 = =
2 × 35 1
31. (D) (c) 24m
Hence, the ratio of volumes of cylindrical
Explanation: We have, and hemispherical parts is 6 : 1.
Area of canvas= 551 m2
Area of canvas wasted in stiching etc. 34. (B) (c) 343 cm3
= 1 m2 Explanation: Volume of the solid figure
\ Area of canvas used in making the tent = Volume of the cube
= 551 – 1 = 550 m2 = (Edge)3 = (7)3
Now, = 343 cm3
Area of canvas used in making the tent (E) (a) 858 cm2
= CSA of tent Explanation: TSA of cone with
= prl heimispherical ice cream top
22 = CSA of cone + CSA of
⇒ 550 = × 7 × l
7 hemisphere
= prl + 2pr2
[ r = 7 m (Given)]
22
⇒ l = 25 m = × 7 × 25 + 308
7
Since, l2 = h2 + r2
 [Using parts (C) and (D)]
⇒ (25)2 = h2 + (7)2 = 550 + 308
⇒ 625 = h2 + 49 = 858 cm2
⇒ h2 = 625 – 49 = 576
35. Let, the radius of the hemisphere be ‘r’ cm.
2 3
⇒ h = 576 = 24 m So, Volume of the hemisphere = πr ...(i)
3
Hence, height of the conical tent is 24 m.
and, Surface area of the hemisphere
(E) (b) 38.5 m2
= 3πr2 ...(ii)
Explanation: Area of the ground = Area of
According to the question,
circular base of a tent
= pr2 2 3
πr = 3πr2 [From (i) and (ii)]
22 3
= × 7 × 7 = 154 m2
7 9
⇒ r=
Since, in each tent, four students are 2
accomodated, Hence, the diameter of the hemisphere is 9 cm

Surface Areas and Volumes 99

Click here to buy other Educart books on Amazon - https://amzn.to/40U8Txx


41. Radius of cylinder = Radius of cone = r = 6cm. 3
 2
2
r
Height of cylinder = Height of cone = h = 14 cm Þ  =  
 R   3
Now,
r 2
1 2 ⇒ =
Volume of remaining solid = πr2h – πr h R 3
3
2 2 3
= πr h = r

3
2
2 22
= × × 6 × 6 × 14 3 
3 7 Now, (R – r) : r =  r − r  : r
2
= 2 × 22 × 2 × 6 × 2
r 1
= 1056 cm2 = :r = : 1 = 1 : 2.
2 2
Hence, the volume of the remaining solid is Hence, the required ratio is 1 : 2
1056 cm2
58. When two cones with the same base radius
44. Let the length, breadth and height of the and height are joined, the shape so formed is
cuboid is l, b, and h respectively. shown in the figure.
Then, X= l × b
Y= b × h
Z= l × h
So, XYZ = l2 × b2 × h2
⇒ lbh = XYZ
Volume of cuboid = l × b × h
= XYZ .

49. Let r be the radius and h be the height of the Given: radius of cone, r = 8 cm
cylinder. height of cone, h = 15 cm.
Then, it is given that
∴ Slant height of cone,
r + h = 37 ...(i)
and 2pr(r + h) = 1628 ...(ii) l= r 2 + h2
⇒ 2pr × 37 = 1628 [Using (i)]
= ( 8 )2 + (15)2
1628
⇒ 2pr = = 44
37
22 = 64 + 225 = 289
⇒ 2× × r = 44
7 = 17 cm
⇒ r= 7
Putting the value of r in eqn (i), we have When two identical cones are joined base
h = 37 – 7 = 30 cm to base, the total surface area of new solid
becomes equal to the sum of curved surface
Now,
areas of both the cones.
Volume of cylinder = pr2h
∴ Surface area of the shape formed
22 = Curved surface area of 1st cone + Curved
= × 7 × 7 × 30
7 surface area of 2nd cone
= 4620 = 2 × curved surface area of cone
So, the volume of the cylinder is 4620 cm3.
[As both cones are identical]
51. Ratio of the volumes of the two spheres 22
= 2prl = 2 × × 8 ×17
Volume of 1st sphere 7
=
Volume of 2nd sphere
5984
= = 854.857
4 3 7
πr
8 r 2
= 3 = = = Hence, surface area of the shaped, so formed,
4 3 27 R 3
πR is 854.85 cm2.
3

100 Mathematics Class X

Click here to buy other Educart books on Amazon - https://amzn.to/40U8Txx


61. Ice cream cone can be considered as a \ Radius of conical and cylindrical part
hemisphere surmounted on a cone. (r) = 28 m
Radius of cone = Radius of hemisphere = 5 cm Height of cylindrical part
Total height = 10 cm (h) = 6 m
∴ Height of cone = Total height – Radius of Total height of tent = 27 m
hemisphere
\ Height of conical part (h’) = 27 – 6 = 21 m
= 10 – 5
Slant height of conical part,
= 5 cm
l= h '2 + r 2
Volume of hemisphere
2 2
2 = 21 + 28
= πr 3
3
= 441 + 784

2 22 3 2 × 22 ×125
= × × (5) = = 1225 = 35 m
3 7 21
Area of canvas used to make the tent
5500
= = 261.90 cm3 = C.S.A. of tent
21
= C.S.A. of cylinder + C.S.A. of cone
1 2 = 2prh + prl
Volume of cone = πr h
3 = pr (2h + l)
1 22 22
= × × (5)2 × 5 = × 28 (2 × 6 + 35)
3 7 7

2750 = 88 (12 + 35)


= =130.95 cm3 = 88 × 47
21
= 4,136 m2
Total volume of ice cream cone Hence, the area of canvas used in making the
= Volume of hemisphere + Volume of cone tent is 4, 136 m2.
= 261.90 + 130.95 = 392.85 cm3 Hence, required number of marbles is 150.

1 th
Since, part is left unfilled with ice cream,
6 66. Capacity of first glass
th
 1 5 th 2 3
this implies 1 −  i.e. part is filled with = pr2H − pr
 6 6 3
ice cream.
= p × 9(10 − 2)
\ Required volume of ice cream = 72p cm3
5 Capacity of second glass
= × 392.85
6 1 2
= pr2H − pr ℎ
= 327.4 cm3 3
= p × 3 × 3(10 − 0.5)
Concept Applied
= 85.5p cm3
 Ice-cream cone is a combination of a hemisphere and
cone. \ Suresh got more quantity of juice.
[CBSE Marking Scheme SQP 2019]
62.
Explanation :
For the first type of glass:
Radius of cylindrical glass = 3 cm
Height of cylindrical glass = 10 cm
∴ Capacity of glass = Volume of cylindrical
glass – Volume of hemispherical portion

Diameter of conical and cylindrical parts 2 3  2 
= πr2h – πr = πr2  h − 3 r 
(d) = 56 cm 3

Surface Areas and Volumes 101

Click here to buy other Educart books on Amazon - https://amzn.to/40U8Txx


22  2  Now, TSA of Solid
=
7
×3×3  10 − 3 × 3 = CSA of cube + CSA of hemisphere – Area of
top of hemisphere
22
= ×9×8 = 6a2 + 2πr2– πr2
7
= 6a2 + πr2
= 226·28 cm3 ........(i)
= 6 × (10)2 + 3.14 × (5)2
For the second type of glass:
= 678.5 cm2
Radius of cylindrical glass, r = 3 cm
Since, cost of painting 100 cm2 of solid = ` 5
Height of glass, h = 10 cm ∴ Cost of painting 1 cm2 of solid
Height of conical portion, H = 1·5 cm
5
∴ Capacity of glass = Volume of cylindrical = ` × 678.5
100
 glass – Volume of conical portion
= ` 33.925
2 1 2
= πr h – πr H 69. Let h cm be the rise in level of water in the
3
cylindrical vessel
 H
= πr2  h −  Then, volume of water raised in the vessel
 3 V = π(10)2h cu. cm. ...(i)
Volume of 9000 small spherical balls of radius
22  1.5  0.5 cm,
=
7
×3×3  10 − 3 
3
 
V1 = 9000 × 4 π  5  cu. cm
22 3  10 
= × 9 × 9·5
7
= 1500 π cu. cm ...(ii)
= 268.71 cm3  ...... (ii)
Equating the equations, we get
From (i) and (ii) π(10)2h = 1500 π
It is clear that capacity of second type of glass 1500
⇒ h = = 15 cm
is more. Thus, Suresh got more quantity of juice. 100
And, Required difference = 268.71 – 226.28 Hence, the rise in the level of water in the
= 42·43 cm3 vessels is 15 cm.

68. Largest possible diameter of hemisphere 74. For the small cylinder:
= 10 cm Radius, r1 = 8 cm
\ Radius = 5 cm Height, h1 = 60 cm

Total surface area = 6(10)2 + 3.14(5)2


678.5 × 5 ` 3392.50
Cost of painting = =
100 100
= ` 33.9250 = ` 33.93
[CBSE Marking Scheme SQP 2015]
Explanation: Given, Each side of cubical block,
a = 10cm
So, the largest diameter of the hemisphere is
equal to the side of the cube i.e. 10 cm.
10 For the big cylinder:
∴ Radius of the hemisphere, r = = 5 cm
2 24
Radius, r2 = = 12 cm
2
Height, h2 = 220 cm
Volume of the pole = Volume of two cylinders
= pr12h1 + pr22h2
= p[(8)2 60 + (12)2 220]
= 3.14[64 × 60 + 144 × 220]
= 3.14 × [3840 + 31,680]
= 111532.8 cm3

102 Mathematics Class X

Click here to buy other Educart books on Amazon - https://amzn.to/40U8Txx


77. Given : Rocket is in the form of right circular Now,
cylinder, surmourted by a cone at the top. Volume of rocket = volum of cylinder +
volume of cone
= 108 p + 12 p =120 p
= 120 × 3.14
= 376.8 cm3
Hence, volume of the rocket, 376.8 cm3 and the
total surface area of the rocket is 301.44 cm2.
78.

For conical part:


\ Radius of base, r = 3 cm
Slant height, l = 5 cm
Let h be the height of the cone.
We know, l2 = h2 + r2 When we place a solid right circular cone in
a right circular cylinder full of water, then the
Þh= l2 − r 2 = 52 − 32 = 25 − 9 volume of the water that flows out from the
cylinder is equal to the volume of right circular
= 16 = 4 cm cone.
\ Height of cone, h = 4 cm \ Volume of water left in the cylinder
= Volume of cylinder – Volume of cone ...(i)
1 2
\ Volume of cone = πr h For cylinder,
3
Base radius, R = 60 cm
1
= π(3)2 × 4 Height, H = 180 cm
3
\Volume of cylinder = pR2H
= 12p cm3
= p × (60)2 × 180
Also, curved surface area of cone
= 648000p cm3
= prl For cone,
= p(3) (5) = 15p cm2 Base radius, r = 60 cm
For cylindrical part: Height, h = 120 cm
Diameter of cylinder = 6 cm 1 2
Volume of cone = πr h
6 3
\ Radius (R) = = 3 cm
2 1
= × p × (60)2 × 120
Height of cylinder (H) = 12 cm 3
\ Volume of cylinder = pR2H = 144000p cm3
= p × (3)2 × 12 From (i),
= 108p cm3 Volume of water left in the cylinder
= (648000p – 144000p) cm3
Also, curved surface area of cylinder = 2prh
= 504000p cm3
= 2 × p × 3 × 12 = 72p cm2
22
Total curved surface area of rocket = 504000 × = 1584000 cm3
7
= Curved surface area of cylinder + Area of
base of cylinder + Curved surface area of cone 1584000 33
= m = 1.584 m3
m
= 72p + p × (3) + 15p 2 (10)6
= 72p + 9p + 15p Hence, the volume of the water left in the
= 96p = 301.44 cm2 cylinder is 1.584 m3.

Surface Areas and Volumes 103

Click here to buy other Educart books on Amazon - https://amzn.to/40U8Txx


81. 22 2
= × (3) × 21
7
= 594 cm3.
Volume of remaining portion of solid
= Volume of cylinder – (Volume of cone A
+ Volume of cone B)
Given, Diameter of cylinder = 6 cm = 594 – (132 + 66)
\ Radius of cylinder, r = 3 cm = 594 – 198
From the figure, both cones have equal radius. = 396 cm3
\ Radius of cone A = Radius of cone B = r = 3 cm Hence,
Let h1 be the height of cone A and h2 be the For cone A, height = 14 cm
height of cone B. and volume = 132 cm3
It is given that: For cone B, height = 7 cm
Volume of cone A 2 and volume = 66 cm3
=
Volume of cone B 1
Volume of remaining solid = 396 cm3.
1 2
\ Volume of a cone = πr h 83. Given : Edge of cube, a = 7 cm
3
Since, diameter of the hemisphere is equal to
1 2 the edge of the cube.
πr h
3 1 2
\ Diameter of hemisphere = 7 cm
\ =
1 2 1
πr h Þ Radius of hemisphere, r = 7 cm

3 2
2
Þ h1 2
=
h2 1
Þ h1 = 2h2
Now, total height of cylinder = 21 cm
Þ h1 + h2 = 21
Þ 2h2 + h2 = 21
Þ 3h2 = 21
Þ h2 = 7 cm
\ h1 = 2h2 = 2 × 7 = 14 cm
Now, Surface area of the remaining solid
1 = Surface area of the cubical block
Now, Volume of cone A = πr 2h
3 1
– Area of the top of the hemispherical part
1 22
= × × (3)2 ×14 = 132 cm3 + Inner curved surface area of the hemisphere
3 7
= 6a2 – pr2 + 2pr2
1 2 = 6a2 + pr2
Volume of cone B = πr h
3 2 2
= 6 × (7)2 + 22 ×  7 
1 22 2 3 7  2 
= × × (3) × 7 = 66 cm
3 7 = 294 + 38.5
= 332.5
We know that,
Hence, the surface area of the remaining
Volume of cylinder = pr2h
solid is 332.5 cm2.

104 Mathematics Class X

Click here to buy other Educart books on Amazon - https://amzn.to/40U8Txx


 ANSWER SHEET

SELF PRACTICE
5. (c) 20 Cumulative frequency just greater than 28.5 is
Explanation: 38, which belongs to class 11.5–17.5.
Thus, median class is 11.5–17.5 whose upper
Cumulative
Class Frequency limit is 17.5.
Frequency
65-85 4 4
20. (a) Both assertion (A) and reason (R) are true
85-105 5 9
and reason (R) is the correct explanation of
105-125 13 22
assertion (A).
125-145 20 42
Explanation:
145-165 14 56
165-185 7 63 Class Frequency Class- f i xi
185-205 4 67 (f i) marks
(f i)
n 67 3–5 5 4 20
Here, = = 33.5, and the cumulative
2 2
frequency just greater than 33.5 is 42 which 5–7 10 6 60
belongs to class 125 – 145. 7–9 10 8 80
\ Median class = 125 – 145.
9 – 11 7 10 70
Hence, the upper limit of median class is 145.
Here, 20 is the highest frequency which lies in 11 – 13 8 12 96
the interval 125 -145. Σfi = 40 Σfi xi = 326
\ Modal class = 125 – 145. We know that,
Hence, the lower limit of modal class is 125.
Thus, the required difference  fi xi
Mean =
= Upper limit of median class
 fi
– Lower limit of modal class
326
= 145 – 125 = 20. =
40
8. (d) remains the same as that of the original = 8.15
set. Hence, both assertion and reason are true and
Explanation: Since, n = 9 i.e. odd reason is the correct explanation of assertion.
9+1
Then, median = = 5th term 21. (D) (c) 160.6 cm
2
Explanation: Let the assumed mean,
Now, last four observations are increased by 2.
a = 162.5 cm and complete the table as
But the median is 5th observation, which
given below:
remains unchanged.
So, there will be no change in the median. Height of Number xi di = f idi
children of xi–a
14. 17.5 (in cm) children

Explanation: The classes in exclusive form are: fi
0-5.5; 5.5-11.5; 11.5-17.5; 17.5-23.5; 23.5-29.5 145–150 8 147.5 –15 -120
with cumulative frequencies of 13, 23, 38, 46
150–155 4 152.5 –10 -40
and 57 respectively.
Here, N = 57. 155–160 11 157.5 –5 -55

N 160–165 12 162.5 0 0
So, = 28.5 =a
2

Statistics 105

Click here to buy other Educart books on Amazon - https://amzn.to/40U8Txx


24. (A) Cumulative
165–170 8 167.5 5 40 Class
Frequency
Interval Frequency
170–175 5 172.5 10 50
0 – 100 2 2
175–180 2 177.5 15 30
100 – 200 5 7
Total Sf i = 50 Sf idi
=-95 200 – 300 x 7+x
300 – 400 12 19 + x
Here, a = 162.5
400 – 500 17 36 + x
Σf i d i
Now, Mean = ( x ) = a + 500 – 600 20 56 + x
Σf i
( −95) 600 – 700 y 56 + x + y
= 162.5 +
50 700 – 800 9 65 + x + y
= 162.5–1.9 = 160.6 800 – 900 7 72 + x + y
Thus, mean height of the children is 160.6 900 – 1000 4 76 + x + y
cm. N = 100
(E) (d) 18
Since, sum of frequencies = 100
Explanation: ⇒ 76 + x + y = 100
We know, ⇒ x + y = 24 ...(i)
∵ Median = 525 [Given]
sum of observations
Mean = ∴ Median class = 500 – 600
Total number of observations
(B) We know,
sum of observations
⇒ 15 = N 
5  − cf 
Median = l +  2 ×h
\ Sum of observations = 5 × 15 = 75.  f 
It is given that first, second, third, fourth  
and fifth observations are increased by N 100
Here, l = 500, = = 50, cf = 36 + x,
1, 2, 3, 4 and 5 respectively. 2 2
Therefore, the new sum of observations [from (A)]
= 75 + 1 + 2 + 3 + 4 + 5 = 90.
 f = 20, h = 100.
90
\ New mean = = 18  50 − (36 + x ) 
5 525 = 500 + 
\  × 100
22. (A) We know,  20

Lower limit + Upper limit ⇒ 25 = (14 – x) 5


Class mark = ⇒ 5x = 70 – 25 = 45
2
\ For class 20 – 40, ⇒ x= 9
Putting the value of x in eq. (i), we get
20 + 40
Class mark = = 30 9 + y = 24
2
and, for class 60 – 80 ⇒ y = 15
\ x = 9, y = 15
60 + 80
Class mark = = 70 25. (A) (d) 66
2
23. (B) Median class is the class whose cumulative Explanation: Let us draw the frequency
frequency is just greater than half of sum distribution table,

N Durability of Number of Cumulative


of all the frequencies. Here, = 100. As
2 LED bulb (in bulbs frequency
the cumulative frequency of the class Hours) (Frequency)
5000–6000 is 115, which is just greater 1000–1500 100–92 = 8 8
than 100, therefore the median class is
1500–2000 92–81 = 11 19
5000–6000 and thus the lower limit of the
meian class is 5000. 2000–2500 81–59 = 22 41

106 Mathematics Class X

Click here to buy other Educart books on Amazon - https://amzn.to/40U8Txx


Durability of Number of Cumulative 27. (D) (a) 45.60
LED bulb (in bulbs frequency Explanation:
Hours) (Frequency)
Speed Number Class di = f idi
2500–3000 59–34 = 25 66 (in km/h) of four mark xi–a
3000–3500 34–15 = 19 85 wheelers xi
(f i )
3500–4000 15–7 = 8 93
0–10 1 5 –30 –30
4000–4500 7–0 = 7 100
10–20 3 15 –20 –60
The number of LED bulbs having
durability less than 3000 hours will 20–30 7 25 –10 –70
include LED bulbs having durability 30–40 16 35 = a 0 0
1000–1500, 1500–2000, 2000–2500
and 2500–3000. 40–50 35 45 10 350

So, the number of LED bulbs having 50–60 29 55 20 580


durabilityless than 3000 hours 60–70 7 65 30 210
= 8 + 11 + 22 + 25 = 66.
70–80 2 75 40 80
(C) (b) 3000 Let a = assumed mean = 35
Explanation: The median class is the class Here, Sf idi = 1060 and Sf i = 100.
whose cumulative frequency is just greater Σf d
than half of sum all the frequencies. Here, Now, Mean ( x ) = a + i i .
Σf i
N 1060
N = 100. So, = 50 . We observe from the = 35 +
2
table that cumulative frequency of the 100
class 2500–3000 is 66 which is just greater = 35 + 10.6 = 45.6
than 50. Therefore, median class is 2500–
3000 and its upper limit is 3000. Therefore, mean speed of cars is 45.6 km/
hr.
26. (C)
(E) (d) 11.95
Number of Number Class
plants of houses Mark f ixi Explanation: The empirical relationship
Contributed fi xi between the three measures of central
tendency is:
0.5 – 3.5 10 2 20
3 Median = Mode + 2 Mean
3.5 – 6.5 8 5 40
3 Median − Mode
\ Mean =
6.5 – 9.5 x 8 8x 2
9.5 – 12.5 7 11 77 3 × 11.5 − 10.6
=
2
12.5 – 15.5 12 14 168
34.5 − 10.6
15.5 – 18.5 4 17 68 =
2
Σf i = 41+ x Σf ixi = 23.9
373 + 8x = = 11.95
2
We know, 28. (A) (c) 43
∑ fi xi Explanation:
Mean =
∑ fi

Time in Number of Class f ixi
373 + 8 x (sec) students mark
⇒ 8.9 =
41 + x (f i ) (xi)
⇒ 8.9(41 + x) = 373 + 8x
⇒ 0.9x = 373 – 364.9 0–20 8 10 80
⇒ 0.9x = 8.1
20–40 10 30 300
⇒ x =9
Hence, 9 houses contributed 7 to 9 plants. 40–60 13 50 650

Statistics 107

Click here to buy other Educart books on Amazon - https://amzn.to/40U8Txx


60–80 6 70 420 = 35 + 1.818
= 36. 818
80–100 3 90 270
= 36.82
åf i = 40 åf ixi Hence, the average age for which maximum
= 1720 cases occurred is 36.82 years.
∑ fi xi (D) (a) 41.4
1720
Mean = = Explanation: Here, the modal class is 35 -
∑ fi 40
45, as it has maximum frequency 42.
= 43 \ l = 35, f1 = 42, f0 = 10, f2 = 24 and
(D) (c) 80 h = 10.
Explanation: We know,
 f1 − f0 
Class Frequency Cumulative Mode = l +   × h
Frequency  2f1 − f0 − f2 
0–20 8 8
 42 − 10 
= 35 +  × 10
20–40 10 18  84 − 10 − 24 
40–60 13 31
32
60–80 6 37 = 35 + × 10
50
80–100 3 40 = 35 + 6.4
N = 40 = 41.4
N 40 (E) (b) 40.2
\ = = 20
2 2 Explanation:
Class corresponding to cumulative Age No. of cases Cumulative
frequency just greater than 20 i.e., 31 is (in years) (f ) Frequency
40–60.
5–15 8 8
So, Median class is 40–60.
Lower limit of median class = 40 15–25 16 24

Lower limit of modal class = 40 25–35 10 34


Required sum 40 + 40 = 80 35–45 42 76
45–55 24 100
29. (A) (c) 36.82
Explanation: To find the average age for 55–65 12 112
which maximum cases occurred, we have Here, N = Sf = 112
to calculate the mode of the data.
N
In the given distribution, 35 - 45 is the \ = 56
2
modal class at it has maximum frequency Since, cumulative frequency just greater
23. than 56 is 76, which belongs to class
So, l = 35, f1 = 23, f0 = 21, f2 = 14 and 35 - 45.
h = 10. \ Median class = 35–45
We know, \ l = 35, f = 42, cf = 34 and h = 10
 f1 − f0  We know,
Mode = l +   × h N 
 2f1 − f0 − f2 
 2 − cf 
Median = l +   × h
 23 − 21 
= 35 +   f 
× 10  
 46 − 21 − 14 
 56 − 34 
2 = 35 +   × 10
= 35 + × 10  42 
11

108 Mathematics Class X

Click here to buy other Educart books on Amazon - https://amzn.to/40U8Txx


22 45. We know,
= 35 + × 10
42 Mode = 3 Median – 2 Mean
= 35 + 5.24
= 40.24 ⇒ Z = 3M – 2X ...(i)

34. Class mark of 20–50 = 20 + 50 , i.e., 35 Given X:M=9:8


2 X 9
35 + 60 ⇒ =
Class mark of 35–60 = , i.e, 47.5 M 8
2
42. Here, the modal class is 60-80, as it has 9M
⇒ X=
maximum frequency 16. 8
For this class, On putting the value of X in eq (i), we get
l = 60, h = 20, f1 = 16, f0 = 12 and f2 = 4.
9M
Z = 3M–2 ×
 f1 − f0  8
We know, Mode = l +   × h
 2f1 − f0 − f2  9M 3M
= 3M − =
 16 − 12  4 4
= 60 + 
 × 20
 32 − 12 − 4  M 4
=
80 Z 3
= 60 +
16 M:Z= 4:3
= 60 + 5 = 65
49. We first find the class marks (xi) of each class
43. The modal class is 30-35, as this class has the
maximum frequency of 10. Class Class Frequency f ixi
mark (xi) (f i)
For this class,
l = 30, h = 5, f1 = 10, f0 = 9 and f2 = 3. 1–3 2 9 18
3–5 4 22 88
 f −f  5–7 6 27 162
 1 0 
We know, Mode = l +  2f − f − f  × h 7–10 8.5 17 144.5
 1 0 2 
 10 − 9 
Sf i = 75 Sf ixi = 412.5
= 30 +   ×5
 20 − 9 − 3  Σfi xi 412.5
Thus, Mean, x = = = 5.5
5 Σf i 75
= 30 + = 30 + 0.625
8
Hence, the mean of the given distribution is 5.5.
= 30.625

51. Classes 2-4 4-6 6-8 8-10 10-12 12-14


Class–mark (xi) 3 5 7 9 11 13
Frequency (f i) 6 8 15 p 8 4
f ixi 18 40 105 9p 88 52
Here, Σf i = 41 + p 303 + 9p
and Σf ixi = 303 + 9p \ 7.5 =
41 + p
Σf i x i
⇒ 303 + 9p = 307.5 + 7.5 p
We know, Mean = Σf
i ⇒ 1.5 p = 4.5
But, Mean = 7.5 (Given) ⇒ p=3
Hence, the value of p is 3.

56. Classes 500–600 600–700 700–800 800–900 900–1000


Frequency 36 32 32 20 30
Cumulative 36 68 100 120 150
Frequency

Statistics 109

Click here to buy other Educart books on Amazon - https://amzn.to/40U8Txx


N 150 N
Here, = = 75 − cf
2 2
So, Median = l + 2 × h
Cumulative frequency just greater than 75 is f
100, which belongs to class 700 - 800. 75 − 68
= 700 + × 100
So, the median class is 700–800 32
For this class, 700
N = 700 +
l = 700, h = 100, = 75, cf = 68, f = 32. 32
2 = 700 + 21.875 = 721.875
60. Given frequency table is in inclusive form, so we need to convert the data into exclusive form or continuous
form by subtracting 0.5 from the lower limit and adding 0.5 in the upper limit of each class.
Also, we find class mark (xi) of each class and then proceed as follows:

Class Class mark (xi) Frequency (f i) f ixi


3.5–7.5 5.5 5 27.5
7.5–11.5 9.5 4 38
11.5–15.5 13.5 9 121.5
15.5–19.5 17.5 10 175

Sf i = 28 Sf ixi = 362
Σfi xi 362
We know, Mean x = = == 12.93
12.93.
Σf i 28
62. We need to convert the data into continuous classes by subtracting 0.5 from the lower limit and adding
0.5 to the upper limit of each class.

Class Class mark (xi) Number of days (f i) f ixi


15.5–18.5 17 1 17
18.5–21.5 20 3 60
21.5–24.5 23 4 92
24.5–27.5 26 9 234
27.5–30.5 29 13 377
Sf i = 30 Sf ixi = 780
i i
Σf x 780
∴ Mean, x = = = 26
Σf i 30
Hence, she wrote an average of 26 pages per day.
64. The given data is not continuous, so we need to convert the data into continuous form, by subtracting 0.5
from the lower limit and adding 0.5 to the upper limit of each class:

Income Class mark (xi) di = (xi–a) fi f idi


0.5–200.5 100.5 –200 14 –2800
200.5–400.5 300.5 = a 0 15 0
400.5–600.5 500.5 200 14 2800

600.5–800.5 700.5 400 7 2800

Sf i = 50 Sf idi = 2800

Here, a = assumed mean = 300.5 = 300.5 + 56


We know,
Σf i d i x = 356.5
Mean ( x ) = a +
Σf i Thus, the mean daily income of the employees
2800 is ` 356.50.
= 300.5 +
50

110 Mathematics Class X

Click here to buy other Educart books on Amazon - https://amzn.to/40U8Txx


66. Class Interval Class mark Deviation Frequency
f idi
(No. of seats) (xi) di = (xi–a) (f i)
100–104 102 –8 15 –120
104–108 106 –4 20 –80
108–112 a = 110 0 32 0
112–116 114 4 18 72
116–120 118 8 15 120

Sf i = 100 Sf idi = –8
Here, assumed mean, a = 110 (−8)
= 110 + = 110 − 0.08
Σfidi 100
\ ( x ) = a + Σf = 109.92
i
Thus, the mean number of seats occupied
during the flights is 109.92 or 110 (approx)

67. Class Interval Class mark Deviation Frequency no. of f idi


(Weight in kg) (xi) di = (xi–a) wrestlers (f i)
100–110 105 –20 4 –80
110–120 115 –10 14 –140
120–130 a = 125 0 21 0
130–140 135 10 8 80
140–150 145 20 3 60

Sf i = 50 Sf idi = –80
Here, Assumed mean, a = 125 = 125–1.6
Σf d
∴ Mean ( x ) = a +
i i = 123.4
Σf
i Hence, the mean weight of the wrestlers is
( − 80 ) 123.4 kg.
= 125 +
50

72. Class Class mark (xi) Frequency (f i) f i xi


10–30 20 5 100
30–50 40 8 320
50–70 60 12 720
70–90 80 20 1600
90–110 100 3 300
110–130 120 2 240

Sf i = 50 Sf ixi = 3280
We know, 3280
= = 65.6
Σ fi x i 50
Mean x = Σ f
i Hence, the mean of the given distribution is 65.6.

Statistics 111

Click here to buy other Educart books on Amazon - https://amzn.to/40U8Txx


74. The frequency distribution for calculating the mean, for the given data is:
Deviation
Frequency Class mark x −a
Classes ui = i f i ui
( fi ) ( xi ) h
{a = 50, h = 20}
0–20 5 10 –2 –10
20–40 f1 30 –1 –f1
40–60 10 50 = a 0 0
60–80 f2 70 1 f2
80–100 7 90 2 14
100–120 8 110 3 24

Sf i = 30 + f1 + f2 Sf iui = 28–f1 + f2

Since, sum of all the frequencies is 50.  28 − f1 + f2 


\ 30 + f1 + f2 = 50 ⇒ 62.8 = 50 +   × 20
 50 
⇒ f1 + f2 = 20
2
⇒ f1 = 20–f2...(i) ⇒ 12.8 = (28–f1 + f2)
5
Now, mean = 62.8 ⇒ 32 = 28–f1 + f2
We know, ⇒ 32 = 8 + 2f2 [Using (i)]
⇒ 2f2 = 24
 Σf u 
Mean = a +  i i  × h ⇒ f2 = 12
 Σf i  So, using (i), f1 = 20–12 = 8
\ f1 = 8 and f2 = 12
76. Calculation of Mean:
Number of Class-mark Frequency ui = xi – a
f iui
wickets (class) (xi) (f i) where a = 120
20-60 40 7 –80 –560
60-100 80 5 –40 –200
100-140 120 = a 16 0 0
140-180 160 12 40 480
180-220 200 2 80 160
220-260 240 3 120 360

Sf i = 45 Sf iui = 240
Σfi ui N
We know, Mean = a + Here, N = 45, ⇒ = 22.5
Σf i 2
Cumulative frequency just greater than 22.5 is
240 28, which belongs to class 100-140.
= 120 +
45 So, median class is 100 – 140
= 120 + 5.33 = 125.33 For this class,
Calculation of Median: N
l = 100, h = 40, cf = 12, = 22.5, f = 16
Cumulative 2
Classes Frequency N 
Frequency
 2 − cf 
20 – 60 7 7 So, Median = l +   × h
 f 
60 – 100 5 12  
100 – 140 16 28
 22.5 − 12 
140 – 180 12 40 = 100 +  × 40
 16 
180 – 220 2 42 = 100 + 26.25
220 – 260 3 45 = 126.25

112 Mathematics Class X

Click here to buy other Educart books on Amazon - https://amzn.to/40U8Txx


Hence, the mean number of wickets taken is It is given that Σf = 100.
125.33 and the median number of wickets So, 76 + x + y = 100
taken is 126.25. ⇒ x + y = 24 ...(i)
Also, it is given that median is 525.
77. Class Frequency
Cumulative
So, the median class is 500–600.
Frequency
For this class,
0–100 2 2 N
l = 500, h = 100, = 50, cf = 36 + x and f = 20.
100–200 5 7 2
200–300 x 7+x N 
300–400 12 19 + x  2 − cf 
So, Median = l +   × h
400–500 17 36 + x  f 
 
Cumulative
Class Frequency
Frequency  50 − (36 + x ) 
⇒ 525 = 500 +   × 100
500–600 20 56 + x  20 
600–700 y 56 + x + y ⇒ 5(14–x) = 25
⇒ 14–x = 5
700–800 9 65 + x + y ⇒ x=9
800–900 7 72 + x + y From (i), y = 24–x
900–1000 4 76 + x + y = 24– 9 = 15
x = 9, y = 15

79. Class mark Number of students Frequency


Marks f i xi
(xi) (Cumulative frequency) (f i)
0–10 5 80 80 – 77 = 3 15
10–20 15 77 77 – 72 = 5 75
20–30 25 72 72 – 65 = 7 175
30–40 35 65 65 – 55 = 10 350
40–50 45 55 55 – 43 = 12 540
50–60 55 43 43 – 28 = 15 825
60–70 65 28 28 – 16 = 12 780
70–80 75 16 16 – 10 = 6 450
80–90 85 10 10 – 8 = 2 170
90–100 95 8 8–0=8 760
100–110 105 0 0 0

Sf i = 80 Sf ixi = 4140
Σf i x i 4140
∴ Mean = = = 51.75
Σf i 80

81. No. of Cumulative It is given that, N = 100.


Marks
Students frequency ⇒ 75 + x + y = 100
fi (cf) ⇒ x + y = 25 ...(i)
0–10 10 10 Also, it is given that median is 32.
10–20 x 10 + x So, the median class is 30 - 40
For this class,
20–30 25 35 + x
N
30–40 30 65 + x l = 30, f = 30, cf = 35 + x, = 50 and h = 10.
2
40–50 y 65 + x + y
50–60 10 75 + x + y N 
 2 − cf 
Total 100 Now, Median = l +   × h
 f 
 

Statistics 113

Click here to buy other Educart books on Amazon - https://amzn.to/40U8Txx


 50 − (35 + x )  Putting the value of ‘x’ in equation (i), we get
⇒ 32 = 30 + 
30  × 10 y = 25–9 = 16
 
⇒ 2 × 3 = 15–x Hence, the values of ‘x’ and ‘y’ are 9 and 16
⇒ x = 15–6 = 9 respectively.

114 Mathematics Class X

Click here to buy other Educart books on Amazon - https://amzn.to/40U8Txx


 ANSWER SHEET

SELF PRACTICE
5. (b) less than 0 19. (a) 1.5
Explanation: We know that, Explanation: As, for any event E, 0 £ P(E) £ 1.
For an event E, 0 < P(E) < 1. 3 25

Here, = 0.6 and 25% = = 0.25
Thus, when probability is expressed in terms of 5 100
percentage, it always lies from 0 to 100. Therefore, 0.6, 0.25 and 0.3 can be the
Hence, it cannot be less than 0. probability of an event.

1 But, 1.5 is greater than 1. So, it cannot be the


9. (a) probability of an event.
6
Explanation: When a dice is thrown: 23. (a) 0 £ P(A) £ 1
Total number of outcomes Explanation: The measure of likelihood that an
= 6 i.e. {1, 2, 3, 4, 5, 6} event will occur is probability.
Odd numbers less than 3 = {1} It is referred as a number that lies between 0
\ Number of favourable outcomes = 1 and 1.
\ P(odd number less than 3) 0 indicates the impossibility and 1 indicates
the certainty.
No. of favourable outcomes
= Higher the probability of an event, more likely
Total no. of outcomes it is for the event to occur.
1 So the probability of an event lies between 0
=
6 and 1
Therefore, if P(A) denotes the probability of an
10. (c) 480 event A, then 0 ≤ P(A) ≤ 1.
Explanation: Total no. of tickets sold = 6000.
28. 0.977
Probability of winning the first prize, P(E) = 0.08. Explanation: We know that,
 P(E) = 0.08 P(E) + P ( E ) = 1
No. of tickets bought by girl ⇒ P ( E ) = 1 – P(E) = 1 – 0.023
⇒ 0.08 =
Total no. of tickets sold = 0.977
No. of tickets bought by girl 4
⇒ 0.08 = 31.
6000 9
⇒ No. of tickets bought by girl Explanation: Between 1 to 9, there are 4 prime
= 0.08 × 6000 = 480. numbers i.e. 2, 3, 5, 7.
Hence, she bought 480 tickets. 4
\ P(a prime number) =
9
6
14. (b)
23 33. 8
Explanation: Total no. of students = 23
Explanation: The Possibilities are:
No. of students from houses A, B and C = 4 + (HHH), (HTH), (HHT), (HTT), (TTT), (THT),
8 + 5 = 17 (TTH), (THH)
\ Remaining no. of students = 23 – 17 = 6 Thus, total number of outcomes in a single
throw of three coins is 8.
\ Required probability
34. (d) Assertion (A) is false but reason (R) is true.
No. of students, not from houses A, B and C
= Explanation: When a die is thrown once, total
Total no. of students possible outcomes = 6
6 and prime numbers in it are {2, 3, 5}
=
23 Total possible outcomes = 3

Probability 115

Click here to buy other Educart books on Amazon - https://amzn.to/40U8Txx


3 1 42. (C) Outcomes for number 4 to appear on
Probability of getting a prime =  either of the dice
6 2
= (1, 4), (2, 4), (3, 4), (4, 4), (5, 4), (6, 4), (4, 1),
Hence, assertion is false but reason is true.
(4, 2), (4, 3), (4, 5)
37. (b) Both assertion (A) and reason (R) are true \ Number of favourable outcomes = 11
but reason (R) is not the correct explanation
\ Probability that number 4 will appear
of assertion (A).
11
Explanation: Total number of outcomes on one of the dice = .
= (6+2+3) = 11 36
Number of white balls = 6 As, P(E) + P(Not E) = 1.
\ Probability that the number 4 will not
6
P (white marble) = appear on either of the dice
11
11 25
We know that, for any event E, P(E) = 1 – P(E), = 1 – =
36 36
Hence, both assertion and reason are true 43. Possible outcomes are 1, 2, 3, 4, 5, 6.
but reason is not the correct explanation of
assertion. Favourable outcomes are 2, 3, 5.
3 1
So, required probability = or
3 6 2
39. (B) (d)
13
44. Total number of possible outcomes = 36.
Explanation: Number of face cards in a deck
Favourable outcomes = {(1, 1), (2, 2), (3, 3), (4, 4),
= 12
(5, 5), (6, 6)}
12 3
\ P(getting a face card) = = \ Number of favourable outcomes = 6
52 13 6 1
3 \ Required probability = =

(D) (a) 36 6
26
Explanation: Number of red face cards in a 48. Probability of losing a game
deck = 6. = 1 – P(winning a game)
6 3 = 1 – 0.07 = 0.93
\ P(getting a red face card) = =
52 26
52. Total number of possible outcomes = 36
2
40. (B) (a) Only one outcome, i.e. (1, 1) has the product of
5
the two numbers as 1.
Explanation: The perfect squares in the 1
wheel are 9, 16, 25 and 36. So, the required probability is .
36
\ Number of favourable outcomes = 4
55. When a die is thrown twice,
\ Probability of a double chance of spinning

Total number of outcomes = 6 × 6 = 36
4 2
wheel = = Favourable outcomes = {(1, 1), (2, 2), (3, 3),
10 5
3 (4, 4), (5, 5), (6, 6)}
(D) (b)
10
⇒ Number of favourable outcomes = 6
Explanation: Out of the given numbers, the
\ P (getting the same number on both dice)
numbers divisible by 3 are 9, 12 and 36.
6 1
\ Probability of arrow pointing towards a = = .
number divisible by 3 36 6
3 56. When a die is thrown once,
=
10 Total number of outcomes = 6
41. (B) The preference for cricket is 25% and for
Favourable outcomes = {5, 6}
football is 20%. Therefore, preference for
either cricket or football is 25% + 20% i.e.
⇒ Number of favourable outcomes = 2
45%.
\ P(getting a number greater than 4)
That means the preference for neither
cricket nor football is (100 – 45)% i.e. 55%. 2 1
= =
\ Probability of favourite sport being 6 3
neither football nor cricket 63. The given numbers are {–3, –2, –1, 0, 1, 2, 3}
55 11 The square of these numbere are {9, 4, 1, 0,
= =
100 20 1, 4, 9}

116 Mathematics Class X

Click here to buy other Educart books on Amazon - https://amzn.to/40U8Txx


∴ Total numbers of outcomes = 7 50 1
So, probability of getting an even no. = =
The square of numbers that are less than or 100 2
equal to 1 = {1, 0, 1} Similarly, probability of getting an odd number
∴ Number of favourable outcomes = 3 1
=
P(getting a square of a number less than or 2
3 Hence, probability of getting an odd number
equal to 1) = 1
7 = probability of getting even number = .
3 2
Hence, the required probability is .
7 74. Total number of outcomes = 6
67. Given cards are numbered from 3, 4, 5, ...., 50 (A) Let the event of getting A be E1.
\ Total number of possible outcomes (Total
Then, favourable number of cases = 3
number of cards) = 48
3 1
Let E be the event of drawing a card having a \ P(E1) = =
6 2
perfect square number.
Then, the favourable outcomes = {4, 9, 16, 25, (B) Let the event of getting B be E2.
36, 49} Then, favourable number of cases = 2
\ Number of favourable outcomes = 6 2 1
\ P(E2) = =
\ P(E) =
6 1 6 3
=
48 8
76. In a single throw of a die, all possible outcomes
1
Hence, the required probability is . are 1, 2, 3, 4, 5, 6.
8
68. No (False). \Total number of possible outcomes = 6
In a family with 3 children, posssbilities can be (A) Let E1 be the event of getting a prime
are (b, b, b), (g, b, b), (b, g, b), (b, b, g), (g, g, g), number.
(g, g, b), (g, b, g), (b, g, g)
Then, the favourable outcomes are 2, 3, 5.
Where, b denotes a boy and g denotes a girl.
⇒ No. of favourable outcomes = 3
\ Number of possible cases = 8
3
\ Required probability \ P(E1) = = 1
6 2
No. of favourable outcomes
= (B) Let E2 be the event of getting a number
Total no. of outcomes
between 2 and 6.
Since, outcomes of having no girl = {(b, b, b)}
⇒ Number of favourable outcomes = 1 Then, the favourable outcomes are 3, 4, 5.
1 ⇒ No. of favourable outcomes = 3
\ P(no girl) =
8 3 1
Outcomes of having 1 girl = {(g, b, b), (b, g, b), \ P(E2) = =
6 2
(b, b, g)}
⇒ Number of favourable outcomes = 3
78. In a leap year there are 366 days.
3
\ P(1 girl) =  366 days = 52 weeks + 2 days
8
Outcomes of having 2 girls = {(g, g, b), (g, b, g)} These 2 days can be :
3 (Monday, Tuesday), (Tuesday, Wednesday),
\ P(2 girls) =
8 (Wednesday, Thursday), (Thursday, Friday),
And, outcomes of having 3 girls = {(g, g, g)} (Friday, Saturday), (Saturday, Sunday), (Sunday,
1 Monday).
\ P(3 girls) =
8 \Total number of outcomes = 7
1 Favourable outcomes = {(Monday, Tuesday),
Hence, the probability of each is not .
4 (Tuesday, Wednesday)}
71. From 1 to 100, there are 50 even and 50 odd \Number of favourable outcomes = 2
numbers.
\P(53 Tuesdays in a leap year) = 2
Total number of outcomes = 100, 7
and number of favourable outcomes of even Hence, the required probability is 2 .
number = 50 7

Probability 117

Click here to buy other Educart books on Amazon - https://amzn.to/40U8Txx


79. When two dice are thrown, then all possible 89. When two dice are thrown together,
outcomes are: Total number of outcomes = 36
(1, 1), (1, 2), (1, 3), (1, 4), (1, 5), (1, 6), Outcomes with sum more than 9
(2, 1), (2, 2), (2, 3), (2, 4), (2, 5), (2, 6) = (6, 6), (6, 5), (5, 6), (6, 4), (4, 6)
\ Total number of favrouable outcomes = 5
(3, 1), (3, 2), (3, 3), (3, 4), (3, 5), (3, 6)
(4, 1), (4, 2), (4, 3), (4, 4), (4, 5), (4, 6) 5
So, required probability = .
36
(5, 1), (5, 2), (5, 3), (5, 4), (5, 5), (5, 6)
(6, 1), (6, 2), (6, 3), (6, 4), (6, 5), (6, 6) 92. Let the total number of marbles in the jar be x.
\ Total number possible outcomes = 36 1
Given: P (blue marble) =
Let, E be the event that the product of the 5
numbers appeared is less than 18.
1
Then, favourable cases are: P (black marble) =
4
(1, 1), (1, 2), (1, 3), (1, 4), (1, 5), (1, 6), (2, 1), (2, 2),
(2, 3), (2, 4), (2, 5), (2, 6), (3, 1), (3, 2), (3, 3), (3, 4), Number of green marbles= 11
(3, 5), (4, 1), (4, 2), (4, 3), (4, 4), (5, 1), (5, 2), (5, 3), Since,
(6, 1), (6, 2). P (green marble) = 1 – [P (blue marble)
\ Number of favourable cases = 26 + P (black marble)]
1 1
\ P(E) = 26 = 13 = 1 −  + 
36 18 5 4
Hence, the required probability is 13 . 9 11
18 = 1 − =
20 20
83. Total number of outcomes = 46 But, P (green marble) =
11
(A) Let, the event of getting a prime number x
less than 10 be E1. 11 11
\ =
Then, favourable outcomes = {5, 7} x 20
\Total number of favourable outcomes = 2 or x = 20
2 1 Hence, the total number of marbles in the jar
\ P(E1) = =
46 23 is 20.

(B) Let, the event of getting a number which is 95. Number of cards removed = 4 kings + 4 jacks
a perfect square be E2. + 11 diamond (since, jack and king of diamond
Then, favourable outcomes = {9, 16, 25, 36, 49} are already removed) = 19
\Total number of favourable outcomes = 5 ∴ Total number of remaining cards
5 = 52 – 19 = 33
\ P(E2) =
46 (A) As the cards are only jacks, queens and
kings so, there will be 3 queens left as face
84. Two dice are thrown at the same time. cards.
∴ Total number of possible outcomes = 36
\ Number of face cards = 3
(A) Let E be the event of getting the same
number on both dice. 3 1
∴ P(a face card) = =
33 11
For getting the same number on both dice,
favourable outcomes are (1, 1), (2, 2), (3, 3),
Hence, the required probability is 1 .
(4, 4), (5, 5), (6, 6). 11
∴ No. of favourable outcomes = 6
(B) Number of black cards
6 1
\ P(E) = = = 26 – 2 black kings – 2 black jacks
36 6
(B) Probability of getting different numbers on = 26 – 4 = 22
both dice = 1 – Probability of getting same 22 2
number on both dice ∴ P(a black card) = =
33 3
1 5
Þ P’(E) = 1 – P(E) = 1 − = 2
6 6 Hence, the required probability is .
3

118 Mathematics Class X

Click here to buy other Educart books on Amazon - https://amzn.to/40U8Txx


97. There are 8 smaller sections, so probability \ P(E1) =
124
=
31
of the dart landing on the smaller sections 144 36
1 Hence, the probability that customer will
(1 to 8) =
16 31
buy the pen is .
And, there are 4 larger sections, so probability 36
of the dart landing on the largest sections
(B) Probability of not buying the ball pen
1
(9 to 12) = . = 1 – Probability of buying the
8 ball pen
Case 1 :
= 1 – P(E1)

There are 6 composite numbers. 31 5
= 1 – =
The probability of the dart landing on a 36 36
comporite number
Hence, the probability that the customer
 1   1 9
= 3    3    5
  will not buy the pen is .
 16   8  16 36
Case 2 :
Total number of outcomes = 1000

There are 6 even numbers. 103. (A) Let E1 be the event that the first player
The probability of the dart landing on an even wins the prize i.e. selects a card which is a
number perfect square greater than 500.
 1   1 1
∴ Favourable outcomes = {529, 576, 625,
= 4    2   
  676, 729, 784, 841, 900, 961}.
 16   8 2

∴ No. of favourable outcomes = 9
Case 3 :

9

There are 6 factors of 12 ∴ P(E1) = .
1000
The probability of the dart landing on a factor of
 1   1 7 104. (A) Total number of outcomes
12 = 5    1    = Total number of cards
 16   8  16
= 52
Thus, on comparing the above probabilities  Number of kings of red colour = 2
we may conclude that Arya has the highest 2 1
chances of winning. P(getting a king of red colour) = =
52 26
98. (A) Let E1 be the event of getting the numbers (B)  Number of spade = 13
whose sum is less than 7.
13 1
Then, favourable outcomes are (1, 1), (1, 2), P (getting a spade) = =
52 4
(1, 3), (1, 4), (1, 5), (2, 1), (2, 2), (2, 3), (2, 4),
(3, 1), (3, 2), (3, 3), (4, 1), (4, 2) and (5, 1). (C)  Number of queen of diamonds = 1
\Number of favourable outcomes = 15 1
\ P(getting a queen of diamonds) =
15 5 52
∴ P(E1) = =
36 12 105. If two unbiased coins are tossed simultaneously,
5 total possible outcomes are:
Hence, the required probability is . HH, HT, TH, TT
12
Hence, total number of outcomes = 4
No head is obtained if the event TT occurs.
99. Total number of ball pens = 144 Hence, number of favourable outcomes = 1
∴Total number of outcomes is 144. 1
Hence, P(no head) =
4
Also, the number of defective ball pens = 20
A
\ Non-defective ball pens = 144 – 20 = 124 But, P(no head) = [Given]
B 
(A) Let E1 be the event that customer will buy A 1
Þ =
a ball pen i.e., ball pen is non-defective. B 4
Total number of non-defective pens = 124 So, A = 1 and B = 4.

Probability 119

Click here to buy other Educart books on Amazon - https://amzn.to/40U8Txx


Now, (A + B)2 = (1 + 4)2 (5, 1), (5, 2), (5, 3), (5, 4), (5, 5), (5, 6),
= (5)2 (6, 1), (6, 2), (6, 3), (6, 4), (6, 5), (6, 6).
= 25 Then, the total number of outcomes = 36
So, (A + B)2 = 25.
(A) Outcomes when 5 comes up = (1, 5), (2, 5),
108. (B) Let E2 be the event that the disc drawn be (3, 5), (4, 5), (6, 5), (5, 1), (5, 2), (5, 3), (5, 4),
a number divisible by 5. (5, 5), (5, 6)

Then favourable cases are 5, 10, 15, 20, 25 \ total number of outcomes when 5 comes
..., 90. up = 11

\ Total number of favourable outcomes So, total number of outcomes when 5 will
= 18 not come up = 36 – 11 = 25
18 1 25
\ P(E2) = = \ P(that 5 will not come up either time) =
90 5 36
1
Hence, the required probability is . (B) Outcomes when the sum is not more than
5
5 = (1, 1), (1, 2), (1, 3), (1, 4), (2, 1), (2, 2), (2, 3),
(3, 1), (3, 2), (4, 1)
109. When two different dice are thrown together
by Peter, then the possible outcomes are: \ Total number of outcomes when sum is
(1, 1), (1, 2), (1, 3), (1, 4), (1, 5), (1, 6), not more than 5 = 10
(2, 1), (2, 2), (2, 3), (2, 4), (2, 5), (2, 6), \ P(that sum is not more than 5)
(3, 1), (3, 2), (3, 3), (3, 4), (3, 5), (3, 6), 10 5
= =
(4, 1), (4, 2), (4, 3), (4, 4), (4, 5), (4, 6), 36 18
(5, 1), (5, 2), (5, 3), (5, 4), (5, 5), (5, 6),
111. (B) Numbers greater than 3 = 4, 5, 6, 7, 8
(6, 1), (6, 2), (6, 3), (6, 4), (6, 5), (6, 6),
\ Number of numbers greater than 3 = 5
∴ Total number of outcomes = 36
 Favourable outcomes for getting the product 5
\ P(getting a number greater than 3) =
of numbers on the dice equal to 25 = (5, 5) 8
∴ Favourable number of outcomes = 1 5
Hence, the required probability is ·
∴ P(Peter gets the product of numbers as 25) 8
1
= ...(i) 112. Since, x = 1, 2, 3 and y = 1, 4, 9
36

\ Product xy = {1, 4, 9, 2, 8, 18, 3, 12, 27}
When a dice is thrown by Rina, then possible
outcomes are: 1, 2, 3, 4, 5, 6.
\ Total number of possible outcomes of the
∴ Total number of outcomes = 6 product xy = 9
Rina throws a die and squares the number, so
Now, outcomes of product xy having value
to get the number 25, the favourable outcome less than 9 = 1, 4, 2, 8, 3
is 5.
\ Number of favourable outcomes of product
∴ P(Rina gets the number as 5)
xy less than 9 = 5
1
= ...(ii) 5
6
\ P(xy < 9) =
9
From (i) and (ii), we get 5
P(Rina) > P(Peter) Hence, the required probability is .
9
Hence, Rina has better chance to get the number 114. For a > 1,
25. b
When a = 1, b can not take any value
110. When a dice is thrown twice, then the total
When a = 2, b can take one value i.e. 1
outcomes are:
(1, 1), (1, 2), (1, 3), (1, 4), (1, 5), (1, 6), When a = 3, b can take two values i.e. 1, 2
(2, 1), (2, 2), (2, 3), (2, 4), (2, 5), (2, 6), When a = 4, b can take three values i.e. 1, 2, 3
(3, 1), (3, 2), (3, 3), (3, 4), (3, 5), (3, 6), When a = 5, b can take four values i.e. 1, 2, 3, 4
(4, 1), (4, 2), (4, 3), (4, 4), (4, 5), (4, 6),
When a = 6, b can take five values i.e. 1, 2, 3, 4, 5

120 Mathematics Class X

Click here to buy other Educart books on Amazon - https://amzn.to/40U8Txx


Here, total number of possible outcomes is Favourable outcomes are (3, 3), (3, 3), (4, 2),
same as when we throw a dice twice. (4, 2), (5, 1), (5, 1).
\ Total possible outcomes = 36 ∴ No. of favourable outcomes = 6
a  1 + 2 + 3 + 4 + 5 6 1
\ P  > 1 = ∴ P(E5) = =
b  36 36 6
15 5 (F) Let E6 be the event of getting a sum 7.
= =
36 12 Favourable outcomes are (4, 3), (4, 3), (5, 2),
5 (5, 2), (6, 1), (6, 1).
Hence, the required probability is ·
12
∴ No. of favourable outcomes = 6
115. Possible outcomes of first dice 6 1
= {1, 2, 3, 4, 5, 6} ∴ P(E6) = =
36 6
Possible outcomes of second dice
(G) Let E7 be the event of getting a sum 8.
= {1, 1, 2, 2, 3, 3}
Favourable outcomes are (5, 3), (5, 3), (6, 2),
\Total number of outcomes on throwing them (6, 2).
together = 36.
∴ No. of favourable outcomes = 4
(A) Let E1 be the event of getting a sum 2.
Favourable outcomes are (1, 1), (1, 1). 4 1
∴ P(E7) = =

∴ No. of favourable outcomes = 2 36 9

2 1 (H) Let E8 be the event of getting a sum 9.


∴ P(E1) = =
36 18 Favourable outcomes are (6, 3), (6, 3).
(B) Let E2 be the event of getting a sum 3.

∴ No. of favourable outcomes = 2
Favourable outcomes are (1, 2), (1, 2), (2, 1),
(2, 1). 2 1
∴ No. of favourable outcomes = 4 ∴ P(E8) = =
36 18
4 1
∴ P(E2) = =
36 9 Hence, the probability of getting a sum
(C) Let E3 be the event of getting a sum 4. of numbers on the two dice as 2, 3, 4, 5,
Favourable outcomes are (1, 3), (1, 3), (2, 2), 1 1 1 1 1 1 1 1
6, 7, 8, 9 is , , , , , , ,
(2, 2), (3, 1), (3, 1). 18 9 6 6 6 6 9 18
∴ No. of favourable outcomes = 6
respectively.
6 1
∴ P(E3)= = 116. (C) Number of jacks of black colour = 2
36 6
2 1

∴ P(getting a black jack) = =
(D) Let E4 be the event of getting a sum 5. 44 22
Favourable outcomes are (2, 3), (2, 3), (4, 1),
(D) We know that jacks, queens and kings are
(4, 1) (3, 2) (3, 2).
face cards.
∴ Number of favourable outcomes = 6.
∴ Number of remaining face cards
6 1 = (2 + 2 + 2) = 6
∴ P(E4) = =
36 6
6 3
(E) Let E5 be the event of getting a sum 6.
∴ P(getting a face card) = =
44 22

Probability 121

Click here to buy other Educart books on Amazon - https://amzn.to/40U8Txx

You might also like